You are on page 1of 121

12/31/2017 Adda247 Store | Adda247 Store

Purchased by roshnipanwar025@gmail.com
Q1.
Study the following information carefully to answer the given questions.

Seven persons viz. O, P, Q, R, S, T, U visit di erent cities viz. Chandigarh, Dehradun, Jaipur, Meerut,
Pune, Shimla, Sikkim on di erent days starting from Monday to Sunday. And each of the cities is
famous for di erent industries viz. Coal, Cotton, Clothes, Paper, Plastic, Tourism and Steel & Iron.
Also each of them is of di erent age. Neither S nor T is youngest. The one who visit Jaipur is 6th
youngest. Only three persons visit the cities in between the one who visit the city which is famous
for Paper and the one who visit Dehradun. The one who is youngest, visit Chandigarh on Monday.
Pune is famous for Paper. O visits the city which is famous for Tourism. Only three persons are
older than U. The one who is 6th oldest visits Sikkim but not on Friday. R visit the city on Thursday
and is just older than S. The one who visit the city which is famous for Steel & Iron visit the city
after maximum number of Persons. P is older than only two people. Dehradun is not famous for
Steel & Iron and is not being visit on Tuesday. O is older than both Q and T but not the oldest. The
one who visit Shimla which is famous for tourism, visits it immediately after the one who visit
Pune. The city which is famous for Clothes is being visited immediately after the one which is being
famous for Cotton but not on Friday. The one who is youngest visit the city which is being famous
for Coal but not immediately after S. U neither visit Dehradun nor Shimla.
Which among the following city is famous for Plastic Industry? (2 Mark)
A. Pune
B. Shimla
C. Sikkim
D. Jaipur
E. Meerut
Your Answer: Not Attempted

Correct Answer: E. Meerut

Solution
The one who is youngest, visit Chandigarh on Monday. The one who visit the city, which is famous
for Steel & Iron, visit the city after maximum number of Persons. Means the one who visit the city
for more join
which is famous for Steel & Iron visit the city on Sunday. Only three persons visit the cities in
between the one who visit the city which https://t.me/currentAffairscLuB
is famous for Paper and the one who visit Dehradun.
Pune is famous for Paper. Dehradun is not famous for Steel & Iron and is not being visit on
Tuesday. R visit the city on Thursday. Only three persons are older than U. P is older than only two
people.

https://store.adda247.com/#!/myTestAnalysis/printsolution/mappingId=25329/packageId=359/lang=ENGLISH 1/121
12/31/2017 Adda247 Store | Adda247 Store

O is older than both Q and T but not the oldest. Neither S nor T is youngest. R visit the city on
Thursday and is just older than S. So it is clear that R is the oldest and S will be 2nd oldest and Q
will be the youngest. The one who visit Jaipur is 6th youngest. The one who visit Shimla which is
famous for tourism visits immediately after the one who visits Pune. The one who is 6th oldest
visits Sikkim but not on Friday.

The city which is famous for Clothes is being visited immediately after the one which is being
famous for Cotton but not on Friday. The one who is youngest visit the city which is being famous
for Coal but not immediately after S. U neither visit Dehradun nor Shimla. O visits the city which is
famous for Tourism.

Purchased by roshnipanwar025@gmail.com
Q2.
Study the following information carefully to answer the given questions.

for more join


Seven persons viz. O, P, Q, R, S, T, U visit di erent cities viz. Chandigarh, Dehradun, Jaipur, Meerut,
Pune, Shimla, Sikkim on di https://t.me/currentAffairscLuB
erent days starting from Monday to Sunday. And each of the cities is
famous for di erent industries viz. Coal, Cotton, Clothes, Paper, Plastic, Tourism and Steel & Iron.
Also each of them is of di erent age. Neither S nor T is youngest. The one who visit Jaipur is 6th
youngest. Only three persons visit the cities in between the one who visit the city which is famous
for Paper and the one who visit Dehradun. The one who is youngest, visit Chandigarh on Monday.
Pune is famous for Paper. O visits the city which is famous for Tourism. Only three persons are
older than U. The one who is 6th oldest visits Sikkim but not on Friday. R visit the city on Thursday
https://store.adda247.com/#!/myTestAnalysis/printsolution/mappingId=25329/packageId=359/lang=ENGLISH 2/121
12/31/2017 Adda247 Store | Adda247 Store
and is just older than S. The one who visit the city which is famous for Steel & Iron visit the city
after maximum number of Persons. P is older than only two people. Dehradun is not famous for
Steel & Iron and is not being visit on Tuesday. O is older than both Q and T but not the oldest. The
one who visit Shimla which is famous for tourism, visits it immediately after the one who visit
Pune. The city which is famous for Clothes is being visited immediately after the one which is being
famous for Cotton but not on Friday. The one who is youngest visit the city which is being famous
for Coal but not immediately after S. U neither visit Dehradun nor Shimla.
Who among the following is 3rd oldest? (2 Mark)
A. Q
B. S
C. O
D. T
E. U
Your Answer: Not Attempted

Correct Answer: C. O

Solution
The one who is youngest, visit Chandigarh on Monday. The one who visit the city, which is famous
for Steel & Iron, visit the city after maximum number of Persons. Means the one who visit the city
which is famous for Steel & Iron visit the city on Sunday. Only three persons visit the cities in
between the one who visit the city which is famous for Paper and the one who visit Dehradun.
Pune is famous for Paper. Dehradun is not famous for Steel & Iron and is not being visit on
Tuesday. R visit the city on Thursday. Only three persons are older than U. P is older than only two
people.

for more join


https://t.me/currentAffairscLuB
O is older than both Q and T but not the oldest. Neither S nor T is youngest. R visit the city on
Thursday and is just older than S. So it is clear that R is the oldest and S will be 2nd oldest and Q
will be the youngest. The one who visit Jaipur is 6th youngest. The one who visit Shimla which is
famous for tourism visits immediately after the one who visits Pune. The one who is 6th oldest
visits Sikkim but not on Friday.

https://store.adda247.com/#!/myTestAnalysis/printsolution/mappingId=25329/packageId=359/lang=ENGLISH 3/121
12/31/2017 Adda247 Store | Adda247 Store

The city which is famous for Clothes is being visited immediately after the one which is being
famous for Cotton but not on Friday. The one who is youngest visit the city which is being famous
for Coal but not immediately after S. U neither visit Dehradun nor Shimla. O visits the city which is
famous for Tourism.

Purchased by roshnipanwar025@gmail.com
Q3.
Study the following information carefully to answer the given questions.

Seven persons viz. O, P, Q, R, S, T, U visit di erent cities viz. Chandigarh, Dehradun, Jaipur, Meerut,
Pune, Shimla, Sikkim on di erent days starting from Monday to Sunday. And each of the cities is
famous for di erent industries viz. Coal, Cotton, Clothes, Paper, Plastic, Tourism and Steel & Iron.
Also each of them is of di erent age. Neither S nor T is youngest. The one who visit Jaipur is 6th
youngest. Only three persons visit the cities in between the one who visit the city which is famous
for Paper and the one who visit Dehradun. The one who is youngest, visit Chandigarh on Monday.
Pune is famous for Paper. O visits the city which is famous for Tourism. Only three persons are
older than U. The one who is 6th oldest visits Sikkim but not on Friday. R visit the city on Thursday
and is just older than S. The one who visit the city which is famous for Steel & Iron visit the city
after maximum number of Persons. P is older than only two people. Dehradun is not famous for
Steel & Iron and is not being visit on Tuesday. O is older than both Q and T but not the oldest. The
one who visit Shimla which is famous for tourism, visits it immediately after the one who visit
Pune. The city which is famous for Clothes is being visited immediately after the one which is being
famous for Cotton but not on Friday. The one who is youngest visit the city which is being famous
for more join
for Coal but not immediately after S. U neither visit Dehradun nor Shimla.
S visit the city on which of the followinghttps://t.me/currentAffairscLuB
day? (2 Mark)
A. Thursday
B. Sunday
C. Tuesday
D. Friday
E. Monday
Your Answer: Not Attempted
https://store.adda247.com/#!/myTestAnalysis/printsolution/mappingId=25329/packageId=359/lang=ENGLISH 4/121
12/31/2017 Adda247 Store | Adda247 Store

Correct Answer: D. Friday

Solution
The one who is youngest, visit Chandigarh on Monday. The one who visit the city, which is famous
for Steel & Iron, visit the city after maximum number of Persons. Means the one who visit the city
which is famous for Steel & Iron visit the city on Sunday. Only three persons visit the cities in
between the one who visit the city which is famous for Paper and the one who visit Dehradun.
Pune is famous for Paper. Dehradun is not famous for Steel & Iron and is not being visit on
Tuesday. R visit the city on Thursday. Only three persons are older than U. P is older than only two
people.

O is older than both Q and T but not the oldest. Neither S nor T is youngest. R visit the city on
Thursday and is just older than S. So it is clear that R is the oldest and S will be 2nd oldest and Q
will be the youngest. The one who visit Jaipur is 6th youngest. The one who visit Shimla which is
famous for tourism visits immediately after the one who visits Pune. The one who is 6th oldest
visits Sikkim but not on Friday.

for more join


https://t.me/currentAffairscLuB
The city which is famous for Clothes is being visited immediately after the one which is being
famous for Cotton but not on Friday. The one who is youngest visit the city which is being famous
for Coal but not immediately after S. U neither visit Dehradun nor Shimla. O visits the city which is
famous for Tourism.

Purchased by roshnipanwar025@gmail.com
https://store.adda247.com/#!/myTestAnalysis/printsolution/mappingId=25329/packageId=359/lang=ENGLISH 5/121
12/31/2017 Adda247 Store | Adda247 Store
Q4.
Study the following information carefully to answer the given questions. Seven persons viz. O, P,
Q, R, S, T, U visit di erent cities viz. Chandigarh, Dehradun, Jaipur, Meerut, Pune, Shimla, Sikkim on
di erent days starting from Monday to Sunday. And each of the cities is famous for di erent
industries viz. Coal, Cotton, Clothes, Paper, Plastic, Tourism and Steel & Iron. Also each of them is
of di erent age. Neither S nor T is youngest. The one who visit Jaipur is 6th youngest. Only three
persons visit the cities in between the one who visit the city which is famous for Paper and the one
who visit Dehradun. The one who is youngest, visit Chandigarh on Monday. Pune is famous for
Paper. O visits the city which is famous for Tourism. Only three persons are older than U. The one
who is 6th oldest visits Sikkim but not on Friday. R visit the city on Thursday and is just older than
S. The one who visit the city which is famous for Steel & Iron visit the city after maximum number
of Persons. P is older than only two people. Dehradun is not famous for Steel & Iron and is not
being visit on Tuesday. O is older than both Q and T but not the oldest. The one who visit Shimla
which is famous for tourism, visits it immediately after the one who visit Pune. The city which is
famous for Clothes is being visited immediately after the one which is being famous for Cotton but
not on Friday. The one who is youngest visit the city which is being famous for Coal but not
immediately after S. U neither visit Dehradun nor Shimla.
Who among the following visit Sikkim? (2 Mark)
A. Q
B. S
C. O
D. T
E. U
Your Answer: Not Attempted

Correct Answer: D. T

Solution
The one who is youngest, visit Chandigarh on Monday. The one who visit the city, which is famous
for Steel & Iron, visit the city after maximum number of Persons. Means the one who visit the city
which is famous for Steel & Iron visit the city on Sunday. Only three persons visit the cities in
for more join
between the one who visit the city which is famous for Paper and the one who visit Dehradun.
Pune is famous for Paper. Dehradun is not famous for Steel & Iron and is not being visit on
https://t.me/currentAffairscLuB
Tuesday. R visit the city on Thursday. Only three persons are older than U. P is older than only two
people.

https://store.adda247.com/#!/myTestAnalysis/printsolution/mappingId=25329/packageId=359/lang=ENGLISH 6/121
12/31/2017 Adda247 Store | Adda247 Store

O is older than both Q and T but not the oldest. Neither S nor T is youngest. R visit the city on
Thursday and is just older than S. So it is clear that R is the oldest and S will be 2nd oldest and Q
will be the youngest. The one who visit Jaipur is 6th youngest. The one who visit Shimla which is
famous for tourism visits immediately after the one who visits Pune. The one who is 6th oldest
visits Sikkim but not on Friday.

The city which is famous for Clothes is being visited immediately after the one which is being
famous for Cotton but not on Friday. The one who is youngest visit the city which is being famous
for Coal but not immediately after S. U neither visit Dehradun nor Shimla. O visits the city which is
famous for Tourism.

Purchased by roshnipanwar025@gmail.com
Q5.
Study the following information carefully to answer the given questions. Seven persons viz. O, P,
Q, R, S, T, U visit di erent cities viz. Chandigarh, Dehradun, Jaipur, Meerut, Pune, Shimla, Sikkim on
for more join
di erent days starting from Monday to Sunday. And each of the cities is famous for di erent
https://t.me/currentAffairscLuB
industries viz. Coal, Cotton, Clothes, Paper, Plastic, Tourism and Steel & Iron. Also each of them is
of di erent age. Neither S nor T is youngest. The one who visit Jaipur is 6th youngest. Only three
persons visit the cities in between the one who visit the city which is famous for Paper and the one
who visit Dehradun. The one who is youngest, visit Chandigarh on Monday. Pune is famous for
Paper. O visits the city which is famous for Tourism. Only three persons are older than U. The one
who is 6th oldest visits Sikkim but not on Friday. R visit the city on Thursday and is just older than
S. The one who visit the city which is famous for Steel & Iron visit the city after maximum number
https://store.adda247.com/#!/myTestAnalysis/printsolution/mappingId=25329/packageId=359/lang=ENGLISH 7/121
12/31/2017 Adda247 Store | Adda247 Store
of Persons. P is older than only two people. Dehradun is not famous for Steel & Iron and is not
being visit on Tuesday. O is older than both Q and T but not the oldest. The one who visit Shimla
which is famous for tourism, visits it immediately after the one who visit Pune. The city which is
famous for Clothes is being visited immediately after the one which is being famous for Cotton but
not on Friday. The one who is youngest visit the city which is being famous for Coal but not
immediately after S. U neither visit Dehradun nor Shimla.
Which among the following is true regarding Q? (2 Mark)
A. Chandigarh- Coal- youngest
B. Jaipur-Cotton-2nd oldest
C. Dehradun- Clothes- 4th youngest
D. Pune-Paper-6th oldest
E. Meerut-Plastic-6th youngest
Your Answer: Not Attempted

Correct Answer: A. Chandigarh- Coal- youngest

Solution
The one who is youngest, visit Chandigarh on Monday. The one who visit the city, which is famous
for Steel & Iron, visit the city after maximum number of Persons. Means the one who visit the city
which is famous for Steel & Iron visit the city on Sunday. Only three persons visit the cities in
between the one who visit the city which is famous for Paper and the one who visit Dehradun.
Pune is famous for Paper. Dehradun is not famous for Steel & Iron and is not being visit on
Tuesday. R visit the city on Thursday. Only three persons are older than U. P is older than only two
people.

for more join


https://t.me/currentAffairscLuB
O is older than both Q and T but not the oldest. Neither S nor T is youngest. R visit the city on
Thursday and is just older than S. So it is clear that R is the oldest and S will be 2nd oldest and Q
will be the youngest. The one who visit Jaipur is 6th youngest. The one who visit Shimla which is
famous for tourism visits immediately after the one who visits Pune. The one who is 6th oldest
visits Sikkim but not on Friday.

https://store.adda247.com/#!/myTestAnalysis/printsolution/mappingId=25329/packageId=359/lang=ENGLISH 8/121
12/31/2017 Adda247 Store | Adda247 Store

The city which is famous for Clothes is being visited immediately after the one which is being
famous for Cotton but not on Friday. The one who is youngest visit the city which is being famous
for Coal but not immediately after S. U neither visit Dehradun nor Shimla. O visits the city which is
famous for Tourism.

Purchased by roshnipanwar025@gmail.com
Q6.
Study the given information carefully and answer the given questions.

An input-output is given in di erent steps. Some mathematical operations are done in each step.
No mathematical operation is repeated in next step but it can be repeated with some other
mathematical operation (as multiplication can be used with subtraction in step 1 and same can be
used with addition in step 2)

As per the rules following in the steps given above, nd out in each of the following questions the
appropriate step for the given input.

Find the addition of the two numbers obtained in step 2? (2 Mark)


A. 150
B. 87
C. 76
D. 90
for more join
E. 111
Your Answer: Not Attempted https://t.me/currentAffairscLuB
Correct Answer: D. 90 <br>

Solution

https://store.adda247.com/#!/myTestAnalysis/printsolution/mappingId=25329/packageId=359/lang=ENGLISH 9/121
12/31/2017 Adda247 Store | Adda247 Store

Purchased by roshnipanwar025@gmail.com
Q7.
Study the given information carefully and answer the given questions.

An input-output is given in di erent steps. Some mathematical operations are done in each step.
No mathematical operation is repeated in next step but it can be repeated with some other
mathematical operation (as multiplication can be used with subtraction in step 1 and same can be
used with addition in step 2)

As per the rules following in the steps given above, nd out in each of the following questions the
appropriate step for the given input.

Find the di erence between sum of numbers which obtained in 1st step and sum of numbers
obtained in all other steps? (2 Mark)
A. 23.5
B. 150
C. 59.5
D. 83.5
E. 55.5
Your Answer: Not Attempted

Correct Answer: E. 55.5 <br>

Solution

Purchased by roshnipanwar025@gmail.com
Q8.
Study the given information carefully and answer the given questions.
for more join
An input-output is given in di https://t.me/currentAffairscLuB
erent steps. Some mathematical operations are done in each step.
No mathematical operation is repeated in next step but it can be repeated with some other
mathematical operation (as multiplication can be used with subtraction in step 1 and same can be
used with addition in step 2)

As per the rules following in the steps given above, nd out in each of the following questions the
appropriate step for the given input.
https://store.adda247.com/#!/myTestAnalysis/printsolution/mappingId=25329/packageId=359/lang=ENGLISH 10/121
12/31/2017 Adda247 Store | Adda247 Store

Find the multiplication of the 1st digit obtained in block-1 of step I and the 2nd digit obtained in
block-3 of step I? (2 Mark)
A. 26
B. 42
C. 18
D. 9
E. None of these
Your Answer: Not Attempted

Correct Answer: C. 18 <br>

Solution

Purchased by roshnipanwar025@gmail.com
Q9.
Study the given information carefully and answer the given questions.

An input-output is given in di erent steps. Some mathematical operations are done in each step.
No mathematical operation is repeated in next step but it can be repeated with some other
mathematical operation (as multiplication can be used with subtraction in step 1 and same can be
used with addition in step 2)

As per the rules following in the steps given above, nd out in each of the following questions the
appropriate step for the given input.

If we multiply both the digits of 2nd block of step-1 and then divide it by 1st digit of 2nd block of
step-2, then the resultant will be? (2 Mark) for more join
A. 26
B. 12 https://t.me/currentAffairscLuB
C. 10
D. 9
E. None of these
Your Answer: Not Attempted

Correct Answer: C. 10 <br>


https://store.adda247.com/#!/myTestAnalysis/printsolution/mappingId=25329/packageId=359/lang=ENGLISH 11/121
12/31/2017 Adda247 Store | Adda247 Store

Solution

Purchased by roshnipanwar025@gmail.com
Q10.
Study the given information carefully and answer the given questions.

An input-output is given in di erent steps. Some mathematical operations are done in each step.
No mathematical operation is repeated in next step but it can be repeated with some other
mathematical operation (as multiplication can be used with subtraction in step 1 and same can be
used with addition in step 2)

As per the rules following in the steps given above, nd out in each of the following questions the
appropriate step for the given input.

If we divide the number obtained in 2nd block in step 2 by the number obtained in 3rd block in
step 1, then the resultant will be? (2 Mark)
A. 6
B. 7
C. 2
D. 9
E. None of these
Your Answer: Not Attempted

Correct Answer: C. 2 <br>

Solution

for more join


https://t.me/currentAffairscLuB
Purchased by roshnipanwar025@gmail.com
Q11.
India and the UAE have discussed measures to deepen cooperation in a range of areas including
defence, security, terrorism and trade as the two countries held the second round of their
strategic dialogue in Abu Dhabi.

Which among the following is not-in-line with given statement?


https://store.adda247.com/#!/myTestAnalysis/printsolution/mappingId=25329/packageId=359/lang=ENGLISH 12/121
12/31/2017 Adda247 Store | Adda247 Store

(I)The Indian delegation at the talks held yesterday was led by Minister of State for External A airs
M J Akbar while the UAE side was headed by its Minister of State for Foreign A airs Anwar
Gargash.
(II)India ships wheat to Afghanistan via Chabahar. India on Sunday began shipment of wheat to
Afghanistan through the Iranian port of Chabahar.
(III)The UAE has raised its investment pro le in India, and cooperation has increased on security
issues, the statement said. (2 Mark)
A. Only I
B. Both II and III
C. Both I and III
D. Only II
E. None of these
Your Answer: Not Attempted

Correct Answer: D. Only II <br>

Solution

For I-This is in-line with the given statement as it states about the meeting in between Indian and
UAE delegations which is also mentioned in the given statement that India and the UAE have
discussed measures to deepen cooperation in a range of areas.
For II- This is not- in-line with the given statement as it states about shipment of wheat from India
to Afghanistan through the Iranian port of Chabahar whereas the given statement states that India
and the UAE have discussed measures to deepen cooperation in a range of areas.
For III- This is in-line with the given statement as it states that UAE has raised its investment pro le
in India which is also mentioned in the given statement that India and the UAE have discussed
measures to deepen cooperation in a range of areas.
Purchased by roshnipanwar025@gmail.com
Q12.
Read the following information carefully and answer the questions given below:
for more join
Nine family members A, C, D, G, I, K, L, M and W are going for a lunch. A and her husband reached
https://t.me/currentAffairscLuB
there by car. C’s sister-in-law, L’s wife and M’s father are already waiting in a restaurant. K is the
sister of M and likes Chinese. C who is the son of the husband of I ordered Indian food. The
grandson of L likes chocolate ice-cream. D who is the mother-in-law of C does not like the food of
that restaurant. K’s father L whose only son M came one hour late for the lunch. G is not the
youngest member in the family.

If M is married to V, how A is related to V? (1 Mark)


https://store.adda247.com/#!/myTestAnalysis/printsolution/mappingId=25329/packageId=359/lang=ENGLISH 13/121
12/31/2017 Adda247 Store | Adda247 Store
A. Cousin
B. Father-in-law
C. Sister-in-law
D. Niece
E. Can’t be determine
Your Answer: Not Attempted

Correct Answer: C. Sister-in-law <br>

Solution

Purchased by roshnipanwar025@gmail.com
Q13.
Read the following information carefully and answer the questions given below:

Nine family members A, C, D, G, I, K, L, M and W are going for a lunch. A and her husband reached
there by car. C’s sister-in-law, L’s wife and M’s father are already waiting in a restaurant. K is the
sister of M and likes Chinese. C who is the son of the husband of I ordered Indian food. The
grandson of L likes chocolate ice-cream. D who is the mother-in-law of C does not like the food of
that restaurant. K’s father L whose only son M came one hour late for the lunch. G is not the
youngest member in the family.

Who among the following is the father of C? (1 Mark)


A. M
B. G
C. D
D. I
E. Can’t be determine
Your Answer: Not Attempted

Correct Answer: B. G <br> for more join


Solution https://t.me/currentAffairscLuB

Purchased by roshnipanwar025@gmail.com
Q14.
Read the following information carefully and answer the questions given below:

https://store.adda247.com/#!/myTestAnalysis/printsolution/mappingId=25329/packageId=359/lang=ENGLISH 14/121
12/31/2017 Adda247 Store | Adda247 Store
Nine family members A, C, D, G, I, K, L, M and W are going for a lunch. A and her husband reached
there by car. C’s sister-in-law, L’s wife and M’s father are already waiting in a restaurant. K is the
sister of M and likes Chinese. C who is the son of the husband of I ordered Indian food. The
grandson of L likes chocolate ice-cream. D who is the mother-in-law of C does not like the food of
that restaurant. K’s father L whose only son M came one hour late for the lunch. G is not the
youngest member in the family.

How W is related to K? (1 Mark)


A. Niece
B. Nephew
C. Sister
D. Brother
E. Can’t be determine
Your Answer: Not Attempted

Correct Answer: B. Nephew <br>

Solution

Purchased by roshnipanwar025@gmail.com
Q15.
Modi government to reframe social welfare schemes for OBCs. In the run-up to the high stakes
elections in Gujarat and Himachal Pradesh, the Narendra Modi-led government has moved to
reframe social welfare schemes for other backward classes (OBCs).

Which among the following can be hypothesized from the given statement?

(I)The targeted schemes promise quality residential accommodation and scholarships to OBC
students.
(II) This reframe of social welfare schemes for OBCs by the Modi government is a political move to
target high stakes elections in Gujarat and Himachal Pradesh.
for more join
(III) The ministry has laid down guidelines for construction of hostels, which would be energy
e cient, economical yet good quality. (2 https://t.me/currentAffairscLuB
Mark)
A. Only (I)
B. Both (III) and (I)
C. Only (II)
D. Both (II) and (III)
E. None of these
Your Answer: Not Attempted
https://store.adda247.com/#!/myTestAnalysis/printsolution/mappingId=25329/packageId=359/lang=ENGLISH 15/121
12/31/2017 Adda247 Store | Adda247 Store

Correct Answer: C. Only (II) <br>

Solution

For I- This statement can’t be hypothesized from the given statement as nothing is mentioned
about facilities given under social welfare schemes for OBCs in the given statement.
For II- This statement can be hypothesized from the given statement as it is mentioned in the
given statement that Modi-led government has moved to reframe social welfare schemes for
other backward classes (OBCs) in the run-up to the highstakes elections in Gujarat and Himachal
Pradesh.
For III- This statement can’t be hypothesized from the given statement as nothing is mentioned
about facilities given under social welfare schemes for OBCs in the given statement.
Purchased by roshnipanwar025@gmail.com
Q16.
Read the following information carefully and answer the questions given below.

Ten friends i.e. D, R, G, S, P, T, L, C, O and Y sit in a restaurant for dinner purpose on two parallel
rows. Each row contains six seats. At row 1, S, P, O, G and Y sit, facing towards South and one seat
is vacant. Similarly T, R, C, D and L sit at row 2, facing North and again one seat is vacant. They
have di erent branded phones i.e. Jio, Iphone, LG, MI, Lava, Lenovo, Nokia, Redmi, Karbon and
Intex, but not in same order.
The one who has Lenovo sits at 2nd position of a row from left end. There are two persons sit
between D and vacant seat, which is immediate neighbor of the one who faces the one who has
Lenovo phone. The one who has Jio phone does not sit at any extreme end. The one who faces the
vacant seat of row 2, sits 2nd left of Vacant seat of row 1. There are three seats between G and Y. Y
faces the one who sits 3rd right of R who has Karbon phone. The one who has MI phone, is
immediate neighbor of Vacant seat. The one who has Redmi phone faces the one who sits 2nd left
of the one who has Intex mobile. There is no friend sit between T and the one who has Intex
mobile. Vacant seat is not at any extreme end of the row. C sits 2nd left of L. The one who has Jio
phone faces the one who sits 3rd right of P. The one who has Nokia faces the one who has LG
phone who sits 2nd right of vacant seat. O faces the vacant seat and does not have Lava phone.
for more join
https://t.me/currentAffairscLuB
Who among the following has Intex mobile phone? (1 Mark)
A. P
B. G
C. D
D. T
E. L
Your Answer: Not Attempted
https://store.adda247.com/#!/myTestAnalysis/printsolution/mappingId=25329/packageId=359/lang=ENGLISH 16/121
12/31/2017 Adda247 Store | Adda247 Store

Correct Answer: E. L <br>

Solution
It is given that the one who has Lenovo sits at 2nd position of a row from left end. There are two
persons sit between D and vacant seat, which is immediate neighbor of the one who faces the one
who has Lenovo phone.
In this condition, we can see that the one who has Lenovo phone , sits at row 1 and faces South.
But the position of Vacant seat in row 2, is not xed. So it will give two possibilities to place D and
vacant seat but in next condition, it is given that Vacant seat is not at any extreme end of the row.
So there will be only one possibility. Now, The one who faces the vacant seat of row 2, sits 2nd left
of Vacant seat of row 1. So,

Now, it is given, There are three seats between G and Y. Y faces the one who sits 3rd right of R who
has Karbon phone. The one who has Redmi phone faces the one who sits 2nd left of the one who
has Intex mobile. There is no friend sit between T and the one who has Intex mobile. Now for the
Intex mobile phone, there will be two possibilities.

But it is given that C sits 2nd left of L also C and L sit in row 2. So Case 2 will be eliminated.
Now, The one who has Jio phone faces the one who sits 3rd right of P. The one who has Jio phone
does not sit at any extreme end. For this condition, again there will be two possibilities but 2nd
possibilities again will be eliminated by 2nd condition. So,

Now, The one who has MI phone, is immediate neighbor of Vacant seat. The one who has Nokia
faces the one who has LG phone who sits 2nd right of vacant seat. O faces the vacant seat and
does not have Lava phone. Then our nal answer is
Purchased by roshnipanwar025@gmail.com
Q17.
Read the following information carefully and answer the questions given below.

Ten friends i.e. D, R, G, S, P, T, L, C, O and Y sit in a restaurant for dinner purpose on two parallel
rows. Each row contains six seats. At row 1, S, P, O, G and Y sit, facing towards South and one seat
for more join
is vacant. Similarly T, R, C, D and L sit at row 2, facing North and again one seat is vacant. They
https://t.me/currentAffairscLuB
have di erent branded phones i.e. Jio, Iphone, LG, MI, Lava, Lenovo, Nokia, Redmi, Karbon and
Intex, but not in same order.
The one who has Lenovo sits at 2nd position of a row from left end. There are two persons sit
between D and vacant seat, which is immediate neighbor of the one who faces the one who has
Lenovo phone. The one who has Jio phone does not sit at any extreme end. The one who faces the
vacant seat of row 2, sits 2nd left of Vacant seat of row 1. There are three seats between G and Y. Y
faces the one who sits 3rd right of R who has Karbon phone. The one who has MI phone, is
https://store.adda247.com/#!/myTestAnalysis/printsolution/mappingId=25329/packageId=359/lang=ENGLISH 17/121
12/31/2017 Adda247 Store | Adda247 Store
immediate neighbor of Vacant seat. The one who has Redmi phone faces the one who sits 2nd left
of the one who has Intex mobile. There is no friend sit between T and the one who has Intex
mobile. Vacant seat is not at any extreme end of the row. C sits 2nd left of L. The one who has Jio
phone faces the one who sits 3rd right of P. The one who has Nokia faces the one who has LG
phone who sits 2nd right of vacant seat. O faces the vacant seat and does not have Lava phone.

How many friends sit between the one who has Iphone and the one who faces the one who has
LG phone? (1 Mark)
A. One
B. Two
C. Three
D. Four
E. Five
Your Answer: Not Attempted

Correct Answer: A. One <br>

Solution
It is given that the one who has Lenovo sits at 2nd position of a row from left end. There are two
persons sit between D and vacant seat, which is immediate neighbor of the one who faces the one
who has Lenovo phone.
In this condition, we can see that the one who has Lenovo phone , sits at row 1 and faces South.
But the position of Vacant seat in row 2, is not xed. So it will give two possibilities to place D and
vacant seat but in next condition, it is given that Vacant seat is not at any extreme end of the row.
So there will be only one possibility. Now, The one who faces the vacant seat of row 2, sits 2nd left
of Vacant seat of row 1. So,

Now, it is given, There are three seats between G and Y. Y faces the one who sits 3rd right of R who
has Karbon phone. The one who has Redmi phone faces the one who sits 2nd left of the one who
has Intex mobile. There is no friend sit between T and the one who has Intex mobile. Now for the
Intex mobile phone, there will be two possibilities.
for more join
But it is given that C sits 2nd left of L also C and L sit in row 2. So Case 2 will be eliminated.
https://t.me/currentAffairscLuB
Now, The one who has Jio phone faces the one who sits 3rd right of P. The one who has Jio phone
does not sit at any extreme end. For this condition, again there will be two possibilities but 2nd
possibilities again will be eliminated by 2nd condition. So,

https://store.adda247.com/#!/myTestAnalysis/printsolution/mappingId=25329/packageId=359/lang=ENGLISH 18/121
12/31/2017 Adda247 Store | Adda247 Store
Now, The one who has MI phone, is immediate neighbor of Vacant seat. The one who has Nokia
faces the one who has LG phone who sits 2nd right of vacant seat. O faces the vacant seat and
does not have Lava phone. Then our nal answer is
Purchased by roshnipanwar025@gmail.com
Q18.
Read the following information carefully and answer the questions given below.

Ten friends i.e. D, R, G, S, P, T, L, C, O and Y sit in a restaurant for dinner purpose on two parallel
rows. Each row contains six seats. At row 1, S, P, O, G and Y sit, facing towards South and one seat
is vacant. Similarly T, R, C, D and L sit at row 2, facing North and again one seat is vacant. They
have di erent branded phones i.e. Jio, Iphone, LG, MI, Lava, Lenovo, Nokia, Redmi, Karbon and
Intex, but not in same order.
The one who has Lenovo sits at 2nd position of a row from left end. There are two persons sit
between D and vacant seat, which is immediate neighbor of the one who faces the one who has
Lenovo phone. The one who has Jio phone does not sit at any extreme end. The one who faces the
vacant seat of row 2, sits 2nd left of Vacant seat of row 1. There are three seats between G and Y. Y
faces the one who sits 3rd right of R who has Karbon phone. The one who has MI phone, is
immediate neighbor of Vacant seat. The one who has Redmi phone faces the one who sits 2nd left
of the one who has Intex mobile. There is no friend sit between T and the one who has Intex
mobile. Vacant seat is not at any extreme end of the row. C sits 2nd left of L. The one who has Jio
phone faces the one who sits 3rd right of P. The one who has Nokia faces the one who has LG
phone who sits 2nd right of vacant seat. O faces the vacant seat and does not have Lava phone.

Who among the following faces the one who sits immediate right of the vacant seat of row 1? (1
Mark)
A. T
B. D
C. R
D. C
E. L
Your Answer: Not Attempted
for more join
Correct Answer: B. D <br>
https://t.me/currentAffairscLuB
Solution
It is given that the one who has Lenovo sits at 2nd position of a row from left end. There are two
persons sit between D and vacant seat, which is immediate neighbor of the one who faces the one
who has Lenovo phone.
In this condition, we can see that the one who has Lenovo phone , sits at row 1 and faces South.
But the position of Vacant seat in row 2, is not xed. So it will give two possibilities to place D and
https://store.adda247.com/#!/myTestAnalysis/printsolution/mappingId=25329/packageId=359/lang=ENGLISH 19/121
12/31/2017 Adda247 Store | Adda247 Store
vacant seat but in next condition, it is given that Vacant seat is not at any extreme end of the row.
So there will be only one possibility. Now, The one who faces the vacant seat of row 2, sits 2nd left
of Vacant seat of row 1. So,

Now, it is given, There are three seats between G and Y. Y faces the one who sits 3rd right of R who
has Karbon phone. The one who has Redmi phone faces the one who sits 2nd left of the one who
has Intex mobile. There is no friend sit between T and the one who has Intex mobile. Now for the
Intex mobile phone, there will be two possibilities.

But it is given that C sits 2nd left of L also C and L sit in row 2. So Case 2 will be eliminated.
Now, The one who has Jio phone faces the one who sits 3rd right of P. The one who has Jio phone
does not sit at any extreme end. For this condition, again there will be two possibilities but 2nd
possibilities again will be eliminated by 2nd condition. So,

Now, The one who has MI phone, is immediate neighbor of Vacant seat. The one who has Nokia
faces the one who has LG phone who sits 2nd right of vacant seat. O faces the vacant seat and
does not have Lava phone. Then our nal answer is
Purchased by roshnipanwar025@gmail.com
Q19.
Read the following information carefully and answer the questions given below.

Ten friends i.e. D, R, G, S, P, T, L, C, O and Y sit in a restaurant for dinner purpose on two parallel
rows. Each row contains six seats. At row 1, S, P, O, G and Y sit, facing towards South and one seat
is vacant. Similarly T, R, C, D and L sit at row 2, facing North and again one seat is vacant. They
have di erent branded phones i.e. Jio, Iphone, LG, MI, Lava, Lenovo, Nokia, Redmi, Karbon and
Intex, but not in same order.
The one who has Lenovo sits at 2nd position of a row from left end. There are two persons sit
between D and vacant seat, which is immediate neighbor of the one who faces the one who has
Lenovo phone. The one who has Jio phone does not sit at any extreme end. The one who faces the
vacant seat of row 2, sits 2nd left of Vacant seat of row 1. There are three seats between G and Y. Y
faces the one who sits 3rd right of R who has Karbon phone. The one who has MI phone, is
immediate neighbor of Vacant seat. The one who has Redmi phone faces the one who sits 2nd left
for more join
of the one who has Intex mobile. There is no friend sit between T and the one who has Intex
https://t.me/currentAffairscLuB
mobile. Vacant seat is not at any extreme end of the row. C sits 2nd left of L. The one who has Jio
phone faces the one who sits 3rd right of P. The one who has Nokia faces the one who has LG
phone who sits 2nd right of vacant seat. O faces the vacant seat and does not have Lava phone.

If S is related to Lava and Y is related to C, then who is related to R? (1 Mark)


A. Lava
B. L
https://store.adda247.com/#!/myTestAnalysis/printsolution/mappingId=25329/packageId=359/lang=ENGLISH 20/121
12/31/2017 Adda247 Store | Adda247 Store
C. Nokia
D. O
E. D
Your Answer: Not Attempted

Correct Answer: D. O <br>

Solution
It is given that the one who has Lenovo sits at 2nd position of a row from left end. There are two
persons sit between D and vacant seat, which is immediate neighbor of the one who faces the one
who has Lenovo phone.
In this condition, we can see that the one who has Lenovo phone , sits at row 1 and faces South.
But the position of Vacant seat in row 2, is not xed. So it will give two possibilities to place D and
vacant seat but in next condition, it is given that Vacant seat is not at any extreme end of the row.
So there will be only one possibility. Now, The one who faces the vacant seat of row 2, sits 2nd left
of Vacant seat of row 1. So,

Now, it is given, There are three seats between G and Y. Y faces the one who sits 3rd right of R who
has Karbon phone. The one who has Redmi phone faces the one who sits 2nd left of the one who
has Intex mobile. There is no friend sit between T and the one who has Intex mobile. Now for the
Intex mobile phone, there will be two possibilities.

But it is given that C sits 2nd left of L also C and L sit in row 2. So Case 2 will be eliminated.
Now, The one who has Jio phone faces the one who sits 3rd right of P. The one who has Jio phone
does not sit at any extreme end. For this condition, again there will be two possibilities but 2nd
possibilities again will be eliminated by 2nd condition. So,

Now, The one who has MI phone, is immediate neighbor of Vacant seat. The one who has Nokia
faces the one who has LG phone who sits 2nd right of vacant seat. O faces the vacant seat and
does not have Lava phone. Then our nal answer is
Purchased by roshnipanwar025@gmail.com
Q20. for more join
Read the following information carefully and answer the questions given below.
https://t.me/currentAffairscLuB
Ten friends i.e. D, R, G, S, P, T, L, C, O and Y sit in a restaurant for dinner purpose on two parallel
rows. Each row contains six seats. At row 1, S, P, O, G and Y sit, facing towards South and one seat
is vacant. Similarly T, R, C, D and L sit at row 2, facing North and again one seat is vacant. They
have di erent branded phones i.e. Jio, Iphone, LG, MI, Lava, Lenovo, Nokia, Redmi, Karbon and
Intex, but not in same order.
The one who has Lenovo sits at 2nd position of a row from left end. There are two persons sit
https://store.adda247.com/#!/myTestAnalysis/printsolution/mappingId=25329/packageId=359/lang=ENGLISH 21/121
12/31/2017 Adda247 Store | Adda247 Store
between D and vacant seat, which is immediate neighbor of the one who faces the one who has
Lenovo phone. The one who has Jio phone does not sit at any extreme end. The one who faces the
vacant seat of row 2, sits 2nd left of Vacant seat of row 1. There are three seats between G and Y. Y
faces the one who sits 3rd right of R who has Karbon phone. The one who has MI phone, is
immediate neighbor of Vacant seat. The one who has Redmi phone faces the one who sits 2nd left
of the one who has Intex mobile. There is no friend sit between T and the one who has Intex
mobile. Vacant seat is not at any extreme end of the row. C sits 2nd left of L. The one who has Jio
phone faces the one who sits 3rd right of P. The one who has Nokia faces the one who has LG
phone who sits 2nd right of vacant seat. O faces the vacant seat and does not have Lava phone.

Who among the following has Iphone? (1 Mark)


A. O
B. L
C. P
D. S
E. D
Your Answer: Not Attempted

Correct Answer: A. O <br>

Solution
It is given that the one who has Lenovo sits at 2nd position of a row from left end. There are two
persons sit between D and vacant seat, which is immediate neighbor of the one who faces the one
who has Lenovo phone.
In this condition, we can see that the one who has Lenovo phone , sits at row 1 and faces South.
But the position of Vacant seat in row 2, is not xed. So it will give two possibilities to place D and
vacant seat but in next condition, it is given that Vacant seat is not at any extreme end of the row.
So there will be only one possibility. Now, The one who faces the vacant seat of row 2, sits 2nd left
of Vacant seat of row 1. So,

Now, it is given, There are three seats between G and Y. Y faces the one who sits 3rd right of R who
for more join
has Karbon phone. The one who has Redmi phone faces the one who sits 2nd left of the one who
has Intex mobile. There is no friend sit between T and the one who has Intex mobile. Now for the
https://t.me/currentAffairscLuB
Intex mobile phone, there will be two possibilities.

But it is given that C sits 2nd left of L also C and L sit in row 2. So Case 2 will be eliminated.
Now, The one who has Jio phone faces the one who sits 3rd right of P. The one who has Jio phone
does not sit at any extreme end. For this condition, again there will be two possibilities but 2nd
possibilities again will be eliminated by 2nd condition. So,

https://store.adda247.com/#!/myTestAnalysis/printsolution/mappingId=25329/packageId=359/lang=ENGLISH 22/121
12/31/2017 Adda247 Store | Adda247 Store

Now, The one who has MI phone, is immediate neighbor of Vacant seat. The one who has Nokia
faces the one who has LG phone who sits 2nd right of vacant seat. O faces the vacant seat and
does not have Lava phone. Then our nal answer is
Purchased by roshnipanwar025@gmail.com
Q21.
Read the following information carefully and answer the questions given below:

In a certain coding pattern-


“Energy and Power Sector” is coded as - “KS5, VZ5, ZE1, HS5”
“Housing with Loan Financing” is coded as – “OO1, DI9, UH1, SH7”
“Banking or Development Industry” is coded as – “YH1, WU5, LS15, RZ9”

What will be the code of “Aviation”? (1 Mark)


A. BO1
B. ZO5
C. ZO1
D. BM5
E. BM1
Your Answer: Not Attempted

Correct Answer: C. ZO1 <br>

Solution
These are the latest pattern of coding-decoding questions. In these questions we are applying
following concept:-

Purchased by roshnipanwar025@gmail.com
Q22.
Read the following information carefully and answer the questions given below:

In a certain coding pattern-


for more join
“Energy and Power Sector” is coded as - “KS5, VZ5, ZE1, HS5”
“Housing with Loan Financing” is codedhttps://t.me/currentAffairscLuB
as – “OO1, DI9, UH1, SH7”
“Banking or Development Industry” is coded as – “YH1, WU5, LS15, RZ9”

What can be the code of “Health Sector”? (1 Mark)


A. SG1 HS5
B. SI1 HS3
C. II5 SH5
https://store.adda247.com/#!/myTestAnalysis/printsolution/mappingId=25329/packageId=359/lang=ENGLISH 23/121
12/31/2017 Adda247 Store | Adda247 Store
D. SG1 TI5
E. SG1 TI1
Your Answer: Not Attempted

Correct Answer: B. SI1 HS3 <br>

Solution
These are the latest pattern of coding-decoding questions. In these questions we are applying
following concept:-

Purchased by roshnipanwar025@gmail.com
Q23.
Read the following information carefully and answer the questions given below:

In a certain coding pattern-


“Energy and Power Sector” is coded as - “KS5, VZ5, ZE1, HS5”
“Housing with Loan Financing” is coded as – “OO1, DI9, UH1, SH7”
“Banking or Development Industry” is coded as – “YH1, WU5, LS15, RZ9”

What will be the code of “Agriculture Industry”? (1 Mark)


A. BF1 RZ5
B. ZF1 JZ1
C. BD1 JZ5
D. ZF1 RZ4
E. ZF5 RZ9
Your Answer: Not Attempted

Correct Answer: D. ZF1 RZ4 <br>

Solution
These are the latest pattern of coding-decoding questions. In these questions we are applying
following concept:- for more join
https://t.me/currentAffairscLuB
Purchased by roshnipanwar025@gmail.com
Q24.
Read the following information carefully and answer the questions given below:

In a certain coding pattern-


“Energy and Power Sector” is coded as - “KS5, VZ5, ZE1, HS5”
“Housing with Loan Financing” is coded as – “OO1, DI9, UH1, SH7”
https://store.adda247.com/#!/myTestAnalysis/printsolution/mappingId=25329/packageId=359/lang=ENGLISH 24/121
12/31/2017 Adda247 Store | Adda247 Store
“Banking or Development Industry” is coded as – “YH1, WU5, LS15, RZ9”

What can be the code of “Oil Gas”? (1 Mark)


A. PM9 HT1
B. LM1 TT5
C. PK9 TT1
D. LM9 TT1
E. LM9 HT1
Your Answer: Not Attempted

Correct Answer: D. LM9 TT1 <br>

Solution
These are the latest pattern of coding-decoding questions. In these questions we are applying
following concept:-

Purchased by roshnipanwar025@gmail.com
Q25.
Read the following information carefully and answer the questions given below:

In a certain coding pattern-


“Energy and Power Sector” is coded as - “KS5, VZ5, ZE1, HS5”
“Housing with Loan Financing” is coded as – “OO1, DI9, UH1, SH7”
“Banking or Development Industry” is coded as – “YH1, WU5, LS15, RZ9”

What will be the code of “Road Transportation”? (1 Mark)


A. IE1 GO1
B. EI1 UO5
C. SI1 UO5
D. EU1 GO5
E. EI5 UO5
Your Answer: Not Attempted for more join
Correct Answer: A. IE1 GO1 <br> https://t.me/currentAffairscLuB
Solution
These are the latest pattern of coding-decoding questions. In these questions we are applying
following concept:-

Purchased by roshnipanwar025@gmail.com
https://store.adda247.com/#!/myTestAnalysis/printsolution/mappingId=25329/packageId=359/lang=ENGLISH 25/121
12/31/2017 Adda247 Store | Adda247 Store
Q26.

A. 011001
B. 100110
C. 101010
D. 000000
E. 111111
Your Answer: Not Attempted

Correct Answer: B. 100110 <br>

Solution

represents 011001. For nding one’s complement we change each 1 to 0 and each 0 to 1. Hence
One’s complement of 011001 will be 100110.

Purchased by roshnipanwar025@gmail.com
Q27.
Read the following information carefully and answer the questions given below:

Rahul starts walking from point A towards east direction. After walking 10km he reached to point
B. From there he turned towards his right and walk 3km to reach point C. Then he turns to his
right and walks 5km to reach point D. Now he turns to his south and reached to point E after
walking 3km. Now he turns to east direction and walks 8km and reached to point F. Then he turns
90° in anticlockwise direction and walks 5km to reach point G. From there he turns 90° in
clockwise direction and walks 3km to reach point H. Then he turns to north direction and walks
5km and reached point I. He nally turns to his left and walk 6km to reach point J.
If Rahul turns to his left from his nal position and walks in the same direction then at which point
he will reach rst? (1 Mark)
A. C
B. G
C. I for more join
D. H
E. B https://t.me/currentAffairscLuB
Your Answer: Not Attempted

Correct Answer: E. B

https://store.adda247.com/#!/myTestAnalysis/printsolution/mappingId=25329/packageId=359/lang=ENGLISH 26/121
12/31/2017 Adda247 Store | Adda247 Store
Solution

Purchased by roshnipanwar025@gmail.com
Q28.
Read the following information carefully and answer the questions given below:

Rahul starts walking from point A towards east direction. After walking 10km he reached to point
B. From there he turned towards his right and walk 3km to reach point C. Then he turns to his
right and walks 5km to reach point D. Now he turns to his south and reached to point E after
walking 3km. Now he turns to east direction and walks 8km and reached to point F. Then he turns
90° in anticlockwise direction and walks 5km to reach point G. From there he turns 90° in
clockwise direction and walks 3km to reach point H. Then he turns to north direction and walks
5km and reached point I. He nally turns to his left and walk 6km to reach point J.

In which direction is point D with respect to point I? (1 Mark)


A. South
B. North-east
C. South-west
D. North
E. None of these
Your Answer: Not Attempted

Correct Answer: C. South-west <br>

Solution

Purchased by roshnipanwar025@gmail.com
Q29.
Disaster management systems can safeguard India’s heritage from climate changes. India is a
signatory to the 2015 Sendai Framework for Disaster Risk Reduction, which recognizes that the
for more join
State has the primary role in reducing disaster risk, terrorism related issues and identi es heritage
as a priority area. https://t.me/currentAffairscLuB
Which of the following negates the given statement?

(I)The authority’s report also has detailed guidelines for museums on the systems that need to be
put in place to tackle di erent kinds of threats.
(II)The rst-of-its-kind analysis by the National Disaster Management Authority (NDMA) of the
https://store.adda247.com/#!/myTestAnalysis/printsolution/mappingId=25329/packageId=359/lang=ENGLISH 27/121
12/31/2017 Adda247 Store | Adda247 Store
threats shows that India’s museums face lists terrorism; climate-related events such as oods,
earthquakes and tsunamis; and vandalism.
(III)Climate-related issues such as oods, earthquakes are not a matter of concern for Indian
Heritage. (2 Mark)
A. Only (III)
B. Only (II)
C. Only (I)
D. Both (II) and (III)
E. None of these
Your Answer: Not Attempted

Correct Answer: A. Only (III) <br>

Solution

For I- This statement supports the given statement as it describes that the authority’s report also
has detailed guidelines for museums to tackle di erent kinds of threats as given in the statement
that Disaster management systems can safeguard India’s heritage.
For II- This statement supports the given statement as it describes that analysis of NDMA shows
that India’s museums face lists terrorism and climate-related events which is also mentioned in
the given statement.
For III- This statement does not support or negates the given statement as it describes that
Climate-related issues are not a matter of concern for Indian Heritage whereas the given
statement states that India’s Disaster management systems can safeguard its’ heritage from
climate changes.
Purchased by roshnipanwar025@gmail.com
Q30.
Statement-
Central Pollution Control Board (CPCB) data for all four months shows the average levels of PM 2.5
(tiny breathable particles) and the air quality index (AQI) values at most places in Delhi improved
over 2016.
for more join
Which of the following statements negates the above statement?
https://t.me/currentAffairscLuB
(i) This does not mean Delhi's air was `clean' but only `less dirty'.
(ii) The head of Regional Weather Forecasting Centre said “Weather played a major role in
reducing pollution during the cold months this year.”
(iii) In February 2017, the average PM2.5 concentration at Anand Vihar was found to be 322 gm3,
which is more than ve times the acceptable limit of 60 gm3 whereas the average PM2.5
concentration at Anand Vihar was found to be 260gm3. (2 Mark)
https://store.adda247.com/#!/myTestAnalysis/printsolution/mappingId=25329/packageId=359/lang=ENGLISH 28/121
12/31/2017 Adda247 Store | Adda247 Store
A. Only (ii)
B. Both (ii) and (iii)
C. All (i), (ii) and (iii)
D. Only (iii)
E. Both (i) and (iii)
Your Answer: Not Attempted

Correct Answer: D. Only (iii) <br>

Solution

Statement (i) is suggesting that air is less dirty and therefore strengthen the given statement.
Statement (iii) shows a data that proves that air quality is ve times worse than the desired levels
and also pollution level rises from 2016. So, (iii) negates the given statement. Whereas according
to (ii) air pollution has reduced which clearly strengthen the given statement.

Purchased by roshnipanwar025@gmail.com
Q31.
Study the following information carefully to answer the given questions.

Eight persons K, L, M, N, O, P, Q, R are sitting around a square table in such a way that four of
them are sitting at the corners while four are sitting at the middle side of the table. The ones
sitting at the corner are facing opposite to the center while the ones sitting at the middle are
facing towards the center of the table. Each of them is of di erent age viz. 5, 7, 8, 9, 10, 12, 15, 18
and also likes di erent color viz. Black, Blue, Green, Pink, Purple, Red, White, Yellow. Both who
likes Red and Purple are immediate neighbours of O. L sits third to the left of M whose age is an
even number. The one who faces L likes Green color. There is a person who is sitting at the corner
of square table whose age is twice of the person who sits to the immediate left of that cornered
sitting person. Q’s age is 5 and P’s age is a prime number. K sits opposite to N who likes Blue color.
The one who likes Black color sits second to the right of the one who likes White color. R’s age is
twice of Q’s age but both are not an immediate neighbour of each other. O does not like Green
color. The one who likes Red is an immediate neighbour of the one who likes Green but he is not
for more join
M. P sits second to the left of R. N sits second to the left of the one who likes Pink color. M’s age is
addition of the ages of P and the one who https://t.me/currentAffairscLuB
likes Purple color. K’s age is multiple of Q’s age. The one
whose age is equal to the sum of the ages the persons who likes Black and Yellow, sits at the
corner.
Who among the following sits at the corner? (1 Mark)
A. R
B. O
C. L
https://store.adda247.com/#!/myTestAnalysis/printsolution/mappingId=25329/packageId=359/lang=ENGLISH 29/121
12/31/2017 Adda247 Store | Adda247 Store
D. P
E. K
Your Answer: Not Attempted

Correct Answer: E. K

Solution
The one who faces L likes Green color. L sits third to the left of M whose age is an even number. K
sits opposite to N who likes Blue color.

The one who likes Red is an immediate neighbour of the one who likes Green but he is not M.
Both who likes Red and Purple are immediate neighbours of O. O does not like Green color.

As there is no place for O so case:2 will be eliminated. Now with case:1.

P sits second to the left of R. Q’s age is 5 and P’s age is a prime number. R’s age is twice of Q’s age
but both are not an immediate neighbour of each other. N sits second to the left of the one who
likes Pink color. M’s age is addition of the ages of P and the one who likes Purple color. K’s age is
multiple of Q’s age. The one whose age is twice of himself sits to the immediate left of him. The
one who likes Black color sits second to the right of the one who likes White color. N likes Blue
color. The one whose age is equal to the sum of the ages of the persons who likes Black and
Yellow color, sits at the corner.

Purchased by roshnipanwar025@gmail.com
Q32.
Study the following information carefully to answer the given questions.

Eight persons K, L, M, N, O, P, Q, R are sitting around a square table in such a way that four of
them are sitting at the corners while four are sitting at the middle side of the table. The ones
sitting at the corner are facing opposite to the center while the ones sitting at the middle are
facing towards the center of the table. Each of them is of di erent age viz. 5, 7, 8, 9, 10, 12, 15, 18
and also likes di erent color viz. Black, Blue, Green, Pink, Purple, Red, White, Yellow. Both who
for more join
likes Red and Purple are immediate neighbours of O. L sits third to the left of M whose age is an
https://t.me/currentAffairscLuB
even number. The one who faces L likes Green color. There is a person who is sitting at the corner
of square table whose age is twice of the person who sits to the immediate left of that cornered
sitting person. Q’s age is 5 and P’s age is a prime number. K sits opposite to N who likes Blue color.
The one who likes Black color sits second to the right of the one who likes White color. R’s age is
twice of Q’s age but both are not an immediate neighbour of each other. O does not like Green
color. The one who likes Red is an immediate neighbour of the one who likes Green but he is not
M. P sits second to the left of R. N sits second to the left of the one who likes Pink color. M’s age is
https://store.adda247.com/#!/myTestAnalysis/printsolution/mappingId=25329/packageId=359/lang=ENGLISH 30/121
12/31/2017 Adda247 Store | Adda247 Store
addition of the ages of P and the one who likes Purple color. K’s age is multiple of Q’s age. The one
whose age is equal to the sum of the ages the persons who likes Black and Yellow, sits at the
corner.
Who among the following likes Pink color? (1 Mark)
A. K
B. L
C. Q
D. M
E. P
Your Answer: Not Attempted

Correct Answer: D. M

Solution
The one who faces L likes Green color. L sits third to the left of M whose age is an even number. K
sits opposite to N who likes Blue color.

The one who likes Red is an immediate neighbour of the one who likes Green but he is not M.
Both who likes Red and Purple are immediate neighbours of O. O does not like Green color.

As there is no place for O so case:2 will be eliminated. Now with case:1.

P sits second to the left of R. Q’s age is 5 and P’s age is a prime number. R’s age is twice of Q’s age
but both are not an immediate neighbour of each other. N sits second to the left of the one who
likes Pink color. M’s age is addition of the ages of P and the one who likes Purple color. K’s age is
multiple of Q’s age. The one whose age is twice of himself sits to the immediate left of him. The
one who likes Black color sits second to the right of the one who likes White color. N likes Blue
color. The one whose age is equal to the sum of the ages of the persons who likes Black and
Yellow color, sits at the corner.

Purchased by roshnipanwar025@gmail.com
Q33. for more join
Study the following information carefully to answer the given questions.
https://t.me/currentAffairscLuB
Eight persons K, L, M, N, O, P, Q, R are sitting around a square table in such a way that four of
them are sitting at the corners while four are sitting at the middle side of the table. The ones
sitting at the corner are facing opposite to the center while the ones sitting at the middle are
facing towards the center of the table. Each of them is of di erent age viz. 5, 7, 8, 9, 10, 12, 15, 18
and also likes di erent color viz. Black, Blue, Green, Pink, Purple, Red, White, Yellow. Both who
likes Red and Purple are immediate neighbours of O. L sits third to the left of M whose age is an
https://store.adda247.com/#!/myTestAnalysis/printsolution/mappingId=25329/packageId=359/lang=ENGLISH 31/121
12/31/2017 Adda247 Store | Adda247 Store
even number. The one who faces L likes Green color. There is a person who is sitting at the corner
of square table whose age is twice of the person who sits to the immediate left of that cornered
sitting person. Q’s age is 5 and P’s age is a prime number. K sits opposite to N who likes Blue color.
The one who likes Black color sits second to the right of the one who likes White color. R’s age is
twice of Q’s age but both are not an immediate neighbour of each other. O does not like Green
color. The one who likes Red is an immediate neighbour of the one who likes Green but he is not
M. P sits second to the left of R. N sits second to the left of the one who likes Pink color. M’s age is
addition of the ages of P and the one who likes Purple color. K’s age is multiple of Q’s age. The one
whose age is equal to the sum of the ages the persons who likes Black and Yellow, sits at the
corner.
What is the age of L? (1 Mark)
A. 7 year
B. 5 year
C. 15 year
D. 12 year
E. 9 year
Your Answer: Not Attempted

Correct Answer: E. 9 year

Solution
The one who faces L likes Green color. L sits third to the left of M whose age is an even number. K
sits opposite to N who likes Blue color.

The one who likes Red is an immediate neighbour of the one who likes Green but he is not M.
Both who likes Red and Purple are immediate neighbours of O. O does not like Green color.

As there is no place for O so case:2 will be eliminated. Now with case:1.

P sits second to the left of R. Q’s age is 5 and P’s age is a prime number. R’s age is twice of Q’s age
but both are not an immediate neighbour of each other. N sits second to the left of the one who
for more join
likes Pink color. M’s age is addition of the ages of P and the one who likes Purple color. K’s age is
multiple of Q’s age. The one whose age is twice of himself sits to the immediate left of him. The
https://t.me/currentAffairscLuB
one who likes Black color sits second to the right of the one who likes White color. N likes Blue
color. The one whose age is equal to the sum of the ages of the persons who likes Black and
Yellow color, sits at the corner.

Purchased by roshnipanwar025@gmail.com
Q34.

https://store.adda247.com/#!/myTestAnalysis/printsolution/mappingId=25329/packageId=359/lang=ENGLISH 32/121
12/31/2017 Adda247 Store | Adda247 Store
Study the following information carefully to answer the given questions.

Eight persons K, L, M, N, O, P, Q, R are sitting around a square table in such a way that four of
them are sitting at the corners while four are sitting at the middle side of the table. The ones
sitting at the corner are facing opposite to the center while the ones sitting at the middle are
facing towards the center of the table. Each of them is of di erent age viz. 5, 7, 8, 9, 10, 12, 15, 18
and also likes di erent color viz. Black, Blue, Green, Pink, Purple, Red, White, Yellow. Both who
likes Red and Purple are immediate neighbours of O. L sits third to the left of M whose age is an
even number. The one who faces L likes Green color. There is a person who is sitting at the corner
of square table whose age is twice of the person who sits to the immediate left of that cornered
sitting person. Q’s age is 5 and P’s age is a prime number. K sits opposite to N who likes Blue color.
The one who likes Black color sits second to the right of the one who likes White color. R’s age is
twice of Q’s age but both are not an immediate neighbour of each other. O does not like Green
color. The one who likes Red is an immediate neighbour of the one who likes Green but he is not
M. P sits second to the left of R. N sits second to the left of the one who likes Pink color. M’s age is
addition of the ages of P and the one who likes Purple color. K’s age is multiple of Q’s age. The one
whose age is equal to the sum of the ages the persons who likes Black and Yellow, sits at the
corner.
Who sits third to the right of the one who likes Green color? (1 Mark)
A. R
B. N
C. L
D. P
E. K
Your Answer: Not Attempted

Correct Answer: B. N

Solution
The one who faces L likes Green color. L sits third to the left of M whose age is an even number. K
sits opposite to N who likes Blue color.
for more join
The one who likes Red is an immediate neighbour of the one who likes Green but he is not M.
https://t.me/currentAffairscLuB
Both who likes Red and Purple are immediate neighbours of O. O does not like Green color.

As there is no place for O so case:2 will be eliminated. Now with case:1.

P sits second to the left of R. Q’s age is 5 and P’s age is a prime number. R’s age is twice of Q’s age
but both are not an immediate neighbour of each other. N sits second to the left of the one who

https://store.adda247.com/#!/myTestAnalysis/printsolution/mappingId=25329/packageId=359/lang=ENGLISH 33/121
12/31/2017 Adda247 Store | Adda247 Store
likes Pink color. M’s age is addition of the ages of P and the one who likes Purple color. K’s age is
multiple of Q’s age. The one whose age is twice of himself sits to the immediate left of him. The
one who likes Black color sits second to the right of the one who likes White color. N likes Blue
color. The one whose age is equal to the sum of the ages of the persons who likes Black and
Yellow color, sits at the corner.

Purchased by roshnipanwar025@gmail.com
Q35.
Study the following information carefully to answer the given questions.

Eight persons K, L, M, N, O, P, Q, R are sitting around a square table in such a way that four of
them are sitting at the corners while four are sitting at the middle side of the table. The ones
sitting at the corner are facing opposite to the center while the ones sitting at the middle are
facing towards the center of the table. Each of them is of di erent age viz. 5, 7, 8, 9, 10, 12, 15, 18
and also likes di erent color viz. Black, Blue, Green, Pink, Purple, Red, White, Yellow. Both who
likes Red and Purple are immediate neighbours of O. L sits third to the left of M whose age is an
even number. The one who faces L likes Green color. There is a person who is sitting at the corner
of square table whose age is twice of the person who sits to the immediate left of that cornered
sitting person. Q’s age is 5 and P’s age is a prime number. K sits opposite to N who likes Blue color.
The one who likes Black color sits second to the right of the one who likes White color. R’s age is
twice of Q’s age but both are not an immediate neighbour of each other. O does not like Green
color. The one who likes Red is an immediate neighbour of the one who likes Green but he is not
M. P sits second to the left of R. N sits second to the left of the one who likes Pink color. M’s age is
addition of the ages of P and the one who likes Purple color. K’s age is multiple of Q’s age. The one
whose age is equal to the sum of the ages the persons who likes Black and Yellow, sits at the
corner.
Which of the following combination is true? (1 Mark)
A. K- Black-7
B. L- Green-15
C. R- Yellow-10
D. M-Pink-18
E. P-Purple-12 for more join
Your Answer: Not Attempted
https://t.me/currentAffairscLuB
Correct Answer: C. R- Yellow-10

Solution
The one who faces L likes Green color. L sits third to the left of M whose age is an even number. K
sits opposite to N who likes Blue color.

https://store.adda247.com/#!/myTestAnalysis/printsolution/mappingId=25329/packageId=359/lang=ENGLISH 34/121
12/31/2017 Adda247 Store | Adda247 Store
The one who likes Red is an immediate neighbour of the one who likes Green but he is not M.
Both who likes Red and Purple are immediate neighbours of O. O does not like Green color.

As there is no place for O so case:2 will be eliminated. Now with case:1.

P sits second to the left of R. Q’s age is 5 and P’s age is a prime number. R’s age is twice of Q’s age
but both are not an immediate neighbour of each other. N sits second to the left of the one who
likes Pink color. M’s age is addition of the ages of P and the one who likes Purple color. K’s age is
multiple of Q’s age. The one whose age is twice of himself sits to the immediate left of him. The
one who likes Black color sits second to the right of the one who likes White color. N likes Blue
color. The one whose age is equal to the sum of the ages of the persons who likes Black and
Yellow color, sits at the corner.

Purchased by roshnipanwar025@gmail.com
Q36.
Statement-
India's space agency has successfully launched its heaviest rocket. The 640-tonne rocket blasted
o from a launching site o the Bay of Bengal in Sriharikota. Media coverage of the launch has
been euphoric, and often colourful, with news channels comparing the rocket to the weight of 200
elephants, or ve jumbo jets.

What is the motive of the TV channels in comparing the weight of the rocket with that of
elephants?

(i) Such comparisons highlight the importance of the launch for the country, which is aggressively
competing to get a bigger share of the global commercial satellite launch market.
(ii) This was done to promote tourism as India is famous for its elephants.
(iii) Comparison shows that India have a capability to send 200 elephants to the space. (2 Mark)
A. Only (i)
B. Both (ii) and (iii)
C. (i), (ii) and (iii)
D. Only (iii) for more join
E. None of these.
Your Answer: Not Attempted https://t.me/currentAffairscLuB
Correct Answer: A. Only (i) <br>

Solution

Clearly highlighting this fact will help showcase the strengths of our space agency. Elephant was
https://store.adda247.com/#!/myTestAnalysis/printsolution/mappingId=25329/packageId=359/lang=ENGLISH 35/121
12/31/2017 Adda247 Store | Adda247 Store
only used as an example to highlight the importance of launch. So (ii) and (iii) are not true.
Purchased by roshnipanwar025@gmail.com
Q37.
In which format records are arranged in Hierarchical Database Model? (1 Mark)
A. Array
B. List
C. Links
D. Tree
E. None of these
Your Answer: Not Attempted

Correct Answer: D. Tree <br>

Solution

A hierarchical database model is a data model in which the data is organized into a tree-like
structure. The data is stored as records which are connected to one another through links. A
record is a collection of elds, with each eld containing only one value.
Purchased by roshnipanwar025@gmail.com
Q38.
In each of the following questions, two statements numbered I and II are given. There may be
cause and e ect relationship between the two statements. These two statements may be the
e ect of the same cause or independent causes. These statements may be independent causes
without having any relationship. Read both the statements in each question and mark your
answer as-

Statement I-Infosys, among India’s largest software services companies, said it plans to hire 10,000
American workers over the next two years.
Statement II-In lling these jobs, Infosys will hire experienced technology professionals and recent
graduates from major universities, and local and community colleges, and the talent pools will be
created in the future. (1 Mark)
A. If statements I is the cause and statement II is its e ect.
for more join
B. If statements II is the cause and statement I is its e ect.
https://t.me/currentAffairscLuB
C. If both the statements I and II are independent causes.
D. If both the statements I and II are e ects of independent causes.
E. If both the statements I and II are e ects of some common cause.
Your Answer: Not Attempted

Correct Answer: A. If statements I is the cause and statement II is its e ect. <br>

https://store.adda247.com/#!/myTestAnalysis/printsolution/mappingId=25329/packageId=359/lang=ENGLISH 36/121
12/31/2017 Adda247 Store | Adda247 Store
Solution

It is clear from the above that statement I is the cause and II is it’s e ect as this will be the cause
that Infosys is planning to hire workers and for that it will hire graduates and will create talent
pools will be it’s e ect.
Purchased by roshnipanwar025@gmail.com
Q39.
In each of the following questions, two statements numbered I and II are given. There may be
cause and e ect relationship between the two statements. These two statements may be the
e ect of the same cause or independent causes. These statements may be independent causes
without having any relationship. Read both the statements in each question and mark your
answer as-

Statement I- Aadhar has become mandatory for ling income tax returns with PAN card.
Statement II-PAN has become more and more susceptible to duplicity and fraud. (1 Mark)
A. If statements I is the cause and statement II is its e ect.
B. If statements II is the cause and statement I is its e ect.
C. If both the statements I and II are independent causes.
D. If both the statements I and II are e ects of independent causes.
E. If both the statements I and II are e ects of some common cause.
Your Answer: Not Attempted

Correct Answer: B. If statements II is the cause and statement I is its e ect. <br>

Solution

It is clear from the above statement that II is cause and I is it’s e ect because the PAN has become
more vulnerable that is why Aadhar has become mandatory for ling Income Tax.
Purchased by roshnipanwar025@gmail.com
Q40.
Statement-
According to the latest research-“Few people openly admit to holding racist beliefs on the basis of
for more join
color but many psychologists claim most of us are nonetheless unintentionally racist. We hold,
what are called "implicit biases". https://t.me/currentAffairscLuB
One the basis of the information provided in the above statement, which of the following de nes
the term “implicit bias” most accurately?

https://store.adda247.com/#!/myTestAnalysis/printsolution/mappingId=25329/packageId=359/lang=ENGLISH 37/121
12/31/2017 Adda247 Store | Adda247 Store
(i) A slight automatic preference for white people over black people.
(ii) Preference for white people over black people.
(iii) Discriminating against a particular community on the basis of colour. (1 Mark)
A. Only (i)
B. Both (ii) and (iii)
C. Only (ii)
D. Only (iii)
E. None of these.
Your Answer: Not Attempted

Correct Answer: A. Only (i) <br>

Solution

(i) most accurately de nes term “implicit bias” as the word automatic was used which is similar to
the term “unintentionally racist” used by researchers.
Purchased by roshnipanwar025@gmail.com
Q41.
Read the following information carefully and answer the questions given below:

Eight persons A, B, C, D, E, F, G, H are sitting around a circular table in such a way that no two
successive people are sitting together in an alphabetical order (for ex- A can’t sit with B, B can’t sit
with C etc.).Each of them belongs to di erent cities viz. Chennai, Goa, Jaipur, Meerut, Mumbai,
Lucknow, Pune and Varanasi. And each of them also likes di erent natural sites viz. Beach,
Fountain, Forest, Garden, Mountain, River, Tree, Water fall.
*Note- Some of them are facing inside while some are facing outside. Not more than two people
sitting together are facing same direction.
The one who belongs to Varanasi sits second to the left of F. B likes Fountain and sits opposite to
the person who belongs to Meerut. The one who likes trees sits second to the left of H. G does not
like Trees. Only one person sits between the one who belongs to Pune and the one who likes
Water fall. Neither F nor G belongs to Pune. E belongs to Mumbai. G sits third to the right of C. The
one who likes Mountain and the one who likes River are sitting adjacent to each other and facing
for more join
opposite direction to each other. F sits to the immediate left of C and faces opposite direction of C.
H belong to Varanasi and is an immediate https://t.me/currentAffairscLuB
neighbour of E. The one who sits second to the right of
F belongs to Chennai. The one who likes beach sits third to the left of A, who is facing opposite
direction F. The one who likes Forest sits to the immediate left of the one who belongs to Pune.
The one who belongs to Jaipur and the one who belongs to Goa are immediate neighbour of each
other. The one who likes river and the one who likes Garden faces same direction. Both D and B
faces same direction but opposite to the one who belongs to Meerut. The one who belongs to
Lucknow likes Trees. B does not belong to Pune. The one who likes Beach faces opposite direction
https://store.adda247.com/#!/myTestAnalysis/printsolution/mappingId=25329/packageId=359/lang=ENGLISH 38/121
12/31/2017 Adda247 Store | Adda247 Store
of the one who belongs to Jaipur. The one who likes Trees does not face outside of the center.

Who among the following belongs to Lucknow? (1 Mark)


A. C
B. E
C. D
D. F
E. B
Your Answer: Not Attempted

Correct Answer: D. F <br>

Solution
G sits third to the right of C. F sits to the immediate left of C and faces opposite direction of C. The
one who belongs to Varanasi sits second to the left of F. H belong to Varanasi and is an immediate
neighbour of E. B likes Fountain and sits opposite to the person who belongs to Meerut. As A can’t
sit near to B so A sits near to F. E belongs to Mumbai. The one who likes trees sits second to the
left of H. G does not like Trees. The one who sits second to the right of F belongs to Chennai. The
one who likes beach sits third to the left of A, who is facing opposite direction F.

The one who likes Trees does not face outside of the center. So from this condition case-1 will be
eliminated. B does not belong to Pune. Only one person sits between the one who belongs to
Pune and the one who likes Water fall. Neither F nor G belongs to Pune. So A belongs to Pune. The
one who likes Mountain and the one who likes River are sitting adjacent to each other and facing
opposite direction to each other. The one who likes Forest sits to the immediate left of the one
who belongs to Pune. The one who belongs to Jaipur and the one who belongs to Goa are
immediate neighbour of each other. The one who likes river and the one who likes Garden faces
same direction. Both D and B faces same direction but opposite to the one who belongs to
Meerut. The one who belongs to Lucknow likes Trees. The one who likes Beach faces opposite
direction of the one who belongs to Jaipur.

for more join


Purchased by roshnipanwar025@gmail.com
Q42. https://t.me/currentAffairscLuB
Read the following information carefully and answer the questions given below:

Eight persons A, B, C, D, E, F, G, H are sitting around a circular table in such a way that no two
successive people are sitting together in an alphabetical order (for ex- A can’t sit with B, B can’t sit
with C etc.).Each of them belongs to di erent cities viz. Chennai, Goa, Jaipur, Meerut, Mumbai,
Lucknow, Pune and Varanasi. And each of them also likes di erent natural sites viz. Beach,
https://store.adda247.com/#!/myTestAnalysis/printsolution/mappingId=25329/packageId=359/lang=ENGLISH 39/121
12/31/2017 Adda247 Store | Adda247 Store
Fountain, Forest, Garden, Mountain, River, Tree, Water fall.
*Note- Some of them are facing inside while some are facing outside. Not more than two people
sitting together are facing same direction.
The one who belongs to Varanasi sits second to the left of F. B likes Fountain and sits opposite to
the person who belongs to Meerut. The one who likes trees sits second to the left of H. G does not
like Trees. Only one person sits between the one who belongs to Pune and the one who likes
Water fall. Neither F nor G belongs to Pune. E belongs to Mumbai. G sits third to the right of C. The
one who likes Mountain and the one who likes River are sitting adjacent to each other and facing
opposite direction to each other. F sits to the immediate left of C and faces opposite direction of C.
H belong to Varanasi and is an immediate neighbour of E. The one who sits second to the right of
F belongs to Chennai. The one who likes beach sits third to the left of A, who is facing opposite
direction F. The one who likes Forest sits to the immediate left of the one who belongs to Pune.
The one who belongs to Jaipur and the one who belongs to Goa are immediate neighbour of each
other. The one who likes river and the one who likes Garden faces same direction. Both D and B
faces same direction but opposite to the one who belongs to Meerut. The one who belongs to
Lucknow likes Trees. B does not belong to Pune. The one who likes Beach faces opposite direction
of the one who belongs to Jaipur. The one who likes Trees does not face outside of the center.

E likes which natural site? (1 Mark)


A. River
B. Tree
C. Beach
D. Mountain
E. Water fall
Your Answer: Not Attempted

Correct Answer: D. Mountain <br>

Solution
G sits third to the right of C. F sits to the immediate left of C and faces opposite direction of C. The
one who belongs to Varanasi sits second to the left of F. H belong to Varanasi and is an immediate
for more join
neighbour of E. B likes Fountain and sits opposite to the person who belongs to Meerut. As A can’t
sit near to B so A sits near to F. E belongs to Mumbai. The one who likes trees sits second to the
https://t.me/currentAffairscLuB
left of H. G does not like Trees. The one who sits second to the right of F belongs to Chennai. The
one who likes beach sits third to the left of A, who is facing opposite direction F.

The one who likes Trees does not face outside of the center. So from this condition case-1 will be
eliminated. B does not belong to Pune. Only one person sits between the one who belongs to
Pune and the one who likes Water fall. Neither F nor G belongs to Pune. So A belongs to Pune. The

https://store.adda247.com/#!/myTestAnalysis/printsolution/mappingId=25329/packageId=359/lang=ENGLISH 40/121
12/31/2017 Adda247 Store | Adda247 Store
one who likes Mountain and the one who likes River are sitting adjacent to each other and facing
opposite direction to each other. The one who likes Forest sits to the immediate left of the one
who belongs to Pune. The one who belongs to Jaipur and the one who belongs to Goa are
immediate neighbour of each other. The one who likes river and the one who likes Garden faces
same direction. Both D and B faces same direction but opposite to the one who belongs to
Meerut. The one who belongs to Lucknow likes Trees. The one who likes Beach faces opposite
direction of the one who belongs to Jaipur.

Purchased by roshnipanwar025@gmail.com
Q43.
Read the following information carefully and answer the questions given below:

Eight persons A, B, C, D, E, F, G, H are sitting around a circular table in such a way that no two
successive people are sitting together in an alphabetical order (for ex- A can’t sit with B, B can’t sit
with C etc.).Each of them belongs to di erent cities viz. Chennai, Goa, Jaipur, Meerut, Mumbai,
Lucknow, Pune and Varanasi. And each of them also likes di erent natural sites viz. Beach,
Fountain, Forest, Garden, Mountain, River, Tree, Water fall.
*Note- Some of them are facing inside while some are facing outside. Not more than two people
sitting together are facing same direction.
The one who belongs to Varanasi sits second to the left of F. B likes Fountain and sits opposite to
the person who belongs to Meerut. The one who likes trees sits second to the left of H. G does not
like Trees. Only one person sits between the one who belongs to Pune and the one who likes
Water fall. Neither F nor G belongs to Pune. E belongs to Mumbai. G sits third to the right of C. The
one who likes Mountain and the one who likes River are sitting adjacent to each other and facing
opposite direction to each other. F sits to the immediate left of C and faces opposite direction of C.
H belong to Varanasi and is an immediate neighbour of E. The one who sits second to the right of
F belongs to Chennai. The one who likes beach sits third to the left of A, who is facing opposite
direction F. The one who likes Forest sits to the immediate left of the one who belongs to Pune.
The one who belongs to Jaipur and the one who belongs to Goa are immediate neighbour of each
other. The one who likes river and the one who likes Garden faces same direction. Both D and B
faces same direction but opposite to the one who belongs to Meerut. The one who belongs to
for more join
Lucknow likes Trees. B does not belong to Pune. The one who likes Beach faces opposite direction
https://t.me/currentAffairscLuB
of the one who belongs to Jaipur. The one who likes Trees does not face outside of the center.

Who sits third to the right of G? (1 Mark)


A. The one who belongs to Pune
B. The one who belongs to Varanasi
C. The one who likes Garden
D. Both (a) and (c)
https://store.adda247.com/#!/myTestAnalysis/printsolution/mappingId=25329/packageId=359/lang=ENGLISH 41/121
12/31/2017 Adda247 Store | Adda247 Store
E. The one who likes Forest
Your Answer: Not Attempted

Correct Answer: D. Both (a) and (c) <br>

Solution
G sits third to the right of C. F sits to the immediate left of C and faces opposite direction of C. The
one who belongs to Varanasi sits second to the left of F. H belong to Varanasi and is an immediate
neighbour of E. B likes Fountain and sits opposite to the person who belongs to Meerut. As A can’t
sit near to B so A sits near to F. E belongs to Mumbai. The one who likes trees sits second to the
left of H. G does not like Trees. The one who sits second to the right of F belongs to Chennai. The
one who likes beach sits third to the left of A, who is facing opposite direction F.

The one who likes Trees does not face outside of the center. So from this condition case-1 will be
eliminated. B does not belong to Pune. Only one person sits between the one who belongs to
Pune and the one who likes Water fall. Neither F nor G belongs to Pune. So A belongs to Pune. The
one who likes Mountain and the one who likes River are sitting adjacent to each other and facing
opposite direction to each other. The one who likes Forest sits to the immediate left of the one
who belongs to Pune. The one who belongs to Jaipur and the one who belongs to Goa are
immediate neighbour of each other. The one who likes river and the one who likes Garden faces
same direction. Both D and B faces same direction but opposite to the one who belongs to
Meerut. The one who belongs to Lucknow likes Trees. The one who likes Beach faces opposite
direction of the one who belongs to Jaipur.

Purchased by roshnipanwar025@gmail.com
Q44.
Read the following information carefully and answer the questions given below:

Eight persons A, B, C, D, E, F, G, H are sitting around a circular table in such a way that no two
successive people are sitting together in an alphabetical order (for ex- A can’t sit with B, B can’t sit
with C etc.).Each of them belongs to di erent cities viz. Chennai, Goa, Jaipur, Meerut, Mumbai,
for more join
Lucknow, Pune and Varanasi. And each of them also likes di erent natural sites viz. Beach,
https://t.me/currentAffairscLuB
Fountain, Forest, Garden, Mountain, River, Tree, Water fall.
*Note- Some of them are facing inside while some are facing outside. Not more than two people
sitting together are facing same direction.
The one who belongs to Varanasi sits second to the left of F. B likes Fountain and sits opposite to
the person who belongs to Meerut. The one who likes trees sits second to the left of H. G does not
like Trees. Only one person sits between the one who belongs to Pune and the one who likes
Water fall. Neither F nor G belongs to Pune. E belongs to Mumbai. G sits third to the right of C. The
https://store.adda247.com/#!/myTestAnalysis/printsolution/mappingId=25329/packageId=359/lang=ENGLISH 42/121
12/31/2017 Adda247 Store | Adda247 Store
one who likes Mountain and the one who likes River are sitting adjacent to each other and facing
opposite direction to each other. F sits to the immediate left of C and faces opposite direction of C.
H belong to Varanasi and is an immediate neighbour of E. The one who sits second to the right of
F belongs to Chennai. The one who likes beach sits third to the left of A, who is facing opposite
direction F. The one who likes Forest sits to the immediate left of the one who belongs to Pune.
The one who belongs to Jaipur and the one who belongs to Goa are immediate neighbour of each
other. The one who likes river and the one who likes Garden faces same direction. Both D and B
faces same direction but opposite to the one who belongs to Meerut. The one who belongs to
Lucknow likes Trees. B does not belong to Pune. The one who likes Beach faces opposite direction
of the one who belongs to Jaipur. The one who likes Trees does not face outside of the center.

Who likes Forest? (1 Mark)


A. C
B. E
C. D
D. F
E. B
Your Answer: Not Attempted

Correct Answer: C. D <br>

Solution
G sits third to the right of C. F sits to the immediate left of C and faces opposite direction of C. The
one who belongs to Varanasi sits second to the left of F. H belong to Varanasi and is an immediate
neighbour of E. B likes Fountain and sits opposite to the person who belongs to Meerut. As A can’t
sit near to B so A sits near to F. E belongs to Mumbai. The one who likes trees sits second to the
left of H. G does not like Trees. The one who sits second to the right of F belongs to Chennai. The
one who likes beach sits third to the left of A, who is facing opposite direction F.

The one who likes Trees does not face outside of the center. So from this condition case-1 will be
eliminated. B does not belong to Pune. Only one person sits between the one who belongs to
for more join
Pune and the one who likes Water fall. Neither F nor G belongs to Pune. So A belongs to Pune. The
one who likes Mountain and the one who likes River are sitting adjacent to each other and facing
https://t.me/currentAffairscLuB
opposite direction to each other. The one who likes Forest sits to the immediate left of the one
who belongs to Pune. The one who belongs to Jaipur and the one who belongs to Goa are
immediate neighbour of each other. The one who likes river and the one who likes Garden faces
same direction. Both D and B faces same direction but opposite to the one who belongs to
Meerut. The one who belongs to Lucknow likes Trees. The one who likes Beach faces opposite

https://store.adda247.com/#!/myTestAnalysis/printsolution/mappingId=25329/packageId=359/lang=ENGLISH 43/121
12/31/2017 Adda247 Store | Adda247 Store
direction of the one who belongs to Jaipur.

Purchased by roshnipanwar025@gmail.com
Q45.
Read the following information carefully and answer the questions given below:

Eight persons A, B, C, D, E, F, G, H are sitting around a circular table in such a way that no two
successive people are sitting together in an alphabetical order (for ex- A can’t sit with B, B can’t sit
with C etc.).Each of them belongs to di erent cities viz. Chennai, Goa, Jaipur, Meerut, Mumbai,
Lucknow, Pune and Varanasi. And each of them also likes di erent natural sites viz. Beach,
Fountain, Forest, Garden, Mountain, River, Tree, Water fall.
*Note- Some of them are facing inside while some are facing outside. Not more than two people
sitting together are facing same direction.
The one who belongs to Varanasi sits second to the left of F. B likes Fountain and sits opposite to
the person who belongs to Meerut. The one who likes trees sits second to the left of H. G does not
like Trees. Only one person sits between the one who belongs to Pune and the one who likes
Water fall. Neither F nor G belongs to Pune. E belongs to Mumbai. G sits third to the right of C. The
one who likes Mountain and the one who likes River are sitting adjacent to each other and facing
opposite direction to each other. F sits to the immediate left of C and faces opposite direction of C.
H belong to Varanasi and is an immediate neighbour of E. The one who sits second to the right of
F belongs to Chennai. The one who likes beach sits third to the left of A, who is facing opposite
direction F. The one who likes Forest sits to the immediate left of the one who belongs to Pune.
The one who belongs to Jaipur and the one who belongs to Goa are immediate neighbour of each
other. The one who likes river and the one who likes Garden faces same direction. Both D and B
faces same direction but opposite to the one who belongs to Meerut. The one who belongs to
Lucknow likes Trees. B does not belong to Pune. The one who likes Beach faces opposite direction
of the one who belongs to Jaipur. The one who likes Trees does not face outside of the center.

Which among the following combination is true regarding A? (1 Mark)


A. River-Varanasi
B. Garden-Pune for more join
C. Beach-Chennai
D. Mountain-Jaipur https://t.me/currentAffairscLuB
E. Water fall-Meerut
Your Answer: Not Attempted

Correct Answer: B. Garden-Pune <br>

https://store.adda247.com/#!/myTestAnalysis/printsolution/mappingId=25329/packageId=359/lang=ENGLISH 44/121
12/31/2017 Adda247 Store | Adda247 Store
Solution
G sits third to the right of C. F sits to the immediate left of C and faces opposite direction of C. The
one who belongs to Varanasi sits second to the left of F. H belong to Varanasi and is an immediate
neighbour of E. B likes Fountain and sits opposite to the person who belongs to Meerut. As A can’t
sit near to B so A sits near to F. E belongs to Mumbai. The one who likes trees sits second to the
left of H. G does not like Trees. The one who sits second to the right of F belongs to Chennai. The
one who likes beach sits third to the left of A, who is facing opposite direction F.

The one who likes Trees does not face outside of the center. So from this condition case-1 will be
eliminated. B does not belong to Pune. Only one person sits between the one who belongs to
Pune and the one who likes Water fall. Neither F nor G belongs to Pune. So A belongs to Pune. The
one who likes Mountain and the one who likes River are sitting adjacent to each other and facing
opposite direction to each other. The one who likes Forest sits to the immediate left of the one
who belongs to Pune. The one who belongs to Jaipur and the one who belongs to Goa are
immediate neighbour of each other. The one who likes river and the one who likes Garden faces
same direction. Both D and B faces same direction but opposite to the one who belongs to
Meerut. The one who belongs to Lucknow likes Trees. The one who likes Beach faces opposite
direction of the one who belongs to Jaipur.

Purchased by roshnipanwar025@gmail.com
Q46.
Working e ciency of A is 20% more than that of B. B can complete a work ‘X’ in 36 days.
B and C together started to complete the work ‘X’ and after 10 days they both left the work and
then remaining work is done by A alone in 15 days.

A and C together started to complete another work ‘Y’ and after working for 12 days they both left
the work. Remaining work is done by B alone in 16 days. D rst completed work ‘X’ and then
completed work ‘Y’ in total 38 days.
It is given that e ciency of all, in completing work ‘X’ and work ‘Y’ is same.

A, B and C working together completed 1/3 rd of work ‘X’, and then A and C are replaced by D. Now
for more join
remaining of work ‘X’ is completed by B and D together. For how many days B worked? (2 Mark)
A. 12 days https://t.me/currentAffairscLuB
B. 10 days
C. 15 days
D. 4 days
E. None of these
Your Answer: Not Attempted

https://store.adda247.com/#!/myTestAnalysis/printsolution/mappingId=25329/packageId=359/lang=ENGLISH 45/121
12/31/2017 Adda247 Store | Adda247 Store
Correct Answer: A. 12 days <br>

Solution

Purchased by roshnipanwar025@gmail.com
Q47.
Working e ciency of A is 20% more than that of B. B can complete a work ‘X’ in 36 days.
B and C together started to complete the work ‘X’ and after 10 days they both left the work and
then remaining work is done by A alone in 15 days.

A and C together started to complete another work ‘Y’ and after working for 12 days they both left
the work. Remaining work is done by B alone in 16 days. D rst completed work ‘X’ and then
completed work ‘Y’ in total 38 days.
It is given that e ciency of all, in completing work ‘X’ and work ‘Y’ is same.

A, C and D working simultaneously completed work ‘X’ in ‘n’ days and A, B, C and D working
simultaneously
completed work ‘Y’ in ‘m’ days. Find the value of (m+n). (2 Mark)
A. 15 days
B. 10 days
C. 12 days
D. 8 days
E. None of these
Your Answer: Not Attempted

Correct Answer: E. None of these <br>

Solution

Purchased by roshnipanwar025@gmail.com
Q48.
for more join
Working e https://t.me/currentAffairscLuB
ciency of A is 20% more than that of B. B can complete a work ‘X’ in 36 days.
B and C together started to complete the work ‘X’ and after 10 days they both left the work and
then remaining work is done by A alone in 15 days.

A and C together started to complete another work ‘Y’ and after working for 12 days they both left
the work. Remaining work is done by B alone in 16 days. D rst completed work ‘X’ and then
completed work ‘Y’ in total 38 days.
https://store.adda247.com/#!/myTestAnalysis/printsolution/mappingId=25329/packageId=359/lang=ENGLISH 46/121
12/31/2017 Adda247 Store | Adda247 Store
It is given that e ciency of all, in completing work ‘X’ and work ‘Y’ is same.

A person E starts the work ‘X’ and leave after 12 days, then B and C complete the remaining work
in 8 days. What is the ratio of number of days taken by A and E together to complete the work ‘X’
to the number of days taken by D, B and C together to complete the both work ‘X’ and ‘Y’ . (2 Mark)
A. 3 : 5
B. 5 : 3
C. 8 : 7
D. 1 : 2
E. None of these
Your Answer: Not Attempted

Correct Answer: A. 3 : 5 <br>

Solution

Purchased by roshnipanwar025@gmail.com
Q49.
Given below is the table which shows ve di erent schemes and rate of simple interest (S.I.) and
rate of compound interest (C.I.) o ered on these schemes.

Note : All the interest is calculated annually.

If a sum is invested in scheme B at C.I, then amount obtained after 2 year from this scheme is 1.44
times the sum invested. Rate of simple interest for scheme A is half of the rate of compound
interest for scheme B. Find out the interest earned when 8000 was invested for 2 years in Scheme
A at S.I and in Scheme B at C.I for 2 years. (1 Mark)
A. 5120
B. 5000 for more join
C. 4800
D. Can’t be determined https://t.me/currentAffairscLuB
E. None of these
Your Answer: Not Attempted

Correct Answer: A. 5120 <br>

https://store.adda247.com/#!/myTestAnalysis/printsolution/mappingId=25329/packageId=359/lang=ENGLISH 47/121
12/31/2017 Adda247 Store | Adda247 Store
Solution

Purchased by roshnipanwar025@gmail.com
Q50.
Given below is the table which shows ve di erent schemes and rate of simple interest (S.I.) and
rate of compound interest (C.I.) o ered on these schemes.

Note : All the interest is calculated annually.

A man invested 10,000 in scheme D at S.I. for 6 years, the interest he obtained is divided into equal
halves and invested in two di erent schemes i.e. scheme B and scheme C for 4 year each at S.I. If
the ratio of interest obtained in both scheme is 3 : 2, then nd out the rate of interest in C scheme.
(2 Mark)
A. 10%
B. 8%
C. 11%
D. 5%
E. None of these
Your Answer: Not Attempted

Correct Answer: B. 8% <br>

Solution

Purchased by roshnipanwar025@gmail.com
Q51.
Given below is the table which shows ve di erent schemes and rate of simple interest (S.I.) and
rate of compound interest (C.I.) o ered on these schemes.
for more join
https://t.me/currentAffairscLuB
Note : All the interest is calculated annually.

A sum is invested in scheme E at S.I. for 2 year and then whole amount obtained is invested at C.I.
in same scheme for 2 more years. If same sum would have been invested in scheme D for 4 year
with S.I. then, what would have been the ration of amount obtained from scheme E to the amount
https://store.adda247.com/#!/myTestAnalysis/printsolution/mappingId=25329/packageId=359/lang=ENGLISH 48/121
12/31/2017 Adda247 Store | Adda247 Store
obtained from scheme D. (2 Mark)
A. 27 : 25
B. 21 : 23
C. 40 : 49
D. Can’t be determined
E. None of these
Your Answer: Not Attempted

Correct Answer: A. 27 : 25 <br>

Solution

Purchased by roshnipanwar025@gmail.com
Q52.
Given below is the table which shows ve di erent schemes and rate of simple interest (S.I.) and
rate of compound interest (C.I.) o ered on these schemes.

Note : All the interest is calculated annually.

A man invests equal sum in two di erent schemes, D and E at S.I. for 4 year each. The total
interest he got is invested in the scheme A for 3 year at C.I. Due to some reason instead of getting
interest from scheme A , the scheme is opped and sum invested in scheme A is depreciated each
year with same rate, and he got Rs. 778688 after 3 year. Find the amount he invested in both
scheme initially. (2 Mark)
A. 30,00,000
B. 40,00,000
C. 20,00,000
D. 10,00,000 for more join
E. None of these
Your Answer: Not Attempted https://t.me/currentAffairscLuB
Correct Answer: C. 20,00,000 <br>

Solution

https://store.adda247.com/#!/myTestAnalysis/printsolution/mappingId=25329/packageId=359/lang=ENGLISH 49/121
12/31/2017 Adda247 Store | Adda247 Store
Purchased by roshnipanwar025@gmail.com
Q53.
Given below is the table which shows ve di erent schemes and rate of simple interest (S.I.) and
rate of compound interest (C.I.) o ered on these schemes.

Note : All the interest is calculated annually.

A sum is invested in scheme C for 5 years at S.I. and then the amount received from it is invested
in same scheme for 2 years at C.I. Total amount received after 7 years is194% more than the sum
invested initially. Find out the rate of interest in scheme C for S.I. (2 Mark)
A. 10%
B. 12.5%
C. 15%
D. 5%
E. None of these
Your Answer: Not Attempted

Correct Answer: A. 10% <br>

Solution

Purchased by roshnipanwar025@gmail.com
Q54.
In the given questions, two quantities are given, one as Quantity I and another as Quantity II. You
have to determine relationship between two quantities and choose the appropriate option

In a two digit number, digit at unit place exceeds, the digit in its tens place by 2 and the product of
the required number with the sum of its digit is equal to 144.

Quantity I: Value of two digit number


for more join
Quantity II: 26 (1 Mark) https://t.me/currentAffairscLuB
A. Quantity I > Quantity II
B. Quantity I < Quantity II
C. Quantity I ≥ Quantity II
D. Quantity I ≤ Quantity II
E. Quantity I = Quantity II or No relation
Your Answer: Not Attempted
https://store.adda247.com/#!/myTestAnalysis/printsolution/mappingId=25329/packageId=359/lang=ENGLISH 50/121
12/31/2017 Adda247 Store | Adda247 Store

Correct Answer: B. Quantity I &lt; Quantity II <br>

Solution

Purchased by roshnipanwar025@gmail.com
Q55.
In the given questions, two quantities are given, one as Quantity I and another as Quantity II. You
have to determine relationship between two quantities and choose the appropriate option

Quantity I : Days after which A and B meet. A and B set out to meet each other from two places
165 km apart. A travels 15 km the rst day, 14 km second day, 13 km the third day and so on, B
travels 10 km the rst, 12 km the second day, 14 km the third day and so on.
Quantity II: Number of days required to complete the whole work if A, B and C can complete a
piece of work in 10, 12 and 15 days respectively. A left the work 5 days before the work was
completed and B left 2 days after A had left. (2 Mark)
A. Quantity I > Quantity II
B. Quantity I < Quantity II
C. Quantity I ≥ Quantity II
D. Quantity I ≤ Quantity II
E. Quantity I = Quantity II or No relation
Your Answer: Not Attempted

Correct Answer: B. Quantity I &lt; Quantity II <br>

Solution

Purchased by roshnipanwar025@gmail.com
Q56.
for more join
In the given questions, two quantities are given, one as Quantity I and another as Quantity II. You
https://t.me/currentAffairscLuB
have to determine relationship between two quantities and choose the appropriate option

Quantity I: Present age of Randy, if 10 years are subtracted from the present age of Randy, then
you would get twelve times of the present age of his grandson Sandy and Sandy is 19 years
younger to Sundar whose age is 24.
Quantity II: Average age of the remaining persons in the group if average age of group of 14
persons is 27 years and 9 months. Two persons, each 42 years old, left the group. (1 Mark)
https://store.adda247.com/#!/myTestAnalysis/printsolution/mappingId=25329/packageId=359/lang=ENGLISH 51/121
12/31/2017 Adda247 Store | Adda247 Store
A. Quantity I > Quantity II
B. Quantity I < Quantity II
C. Quantity I ≥ Quantity II
D. Quantity I ≤ Quantity II
E. Quantity I = Quantity II or No relation
Your Answer: Not Attempted

Correct Answer: A. Quantity I &gt; Quantity II <br>

Solution

Purchased by roshnipanwar025@gmail.com
Q57.
In the given questions, two quantities are given, one as Quantity I and another as Quantity II. You
have to determine relationship between two quantities and choose the appropriate option

Quantity I: Percentage pro t earned by the shopkeeper if at the time of selling and purchasing he
uses weights 10% less and 20% more per kilogram respectively and pro esses to all goods at 5%
pro t.
Quantity II: ‘x’ ; A book was sold for a certain sum and there was a loss of 20%. Had it been sold for
Rs 12 more, there would have been a gain of 30%. ‘x’ would be value of pro t percent if the book
were sold for Rs 4.8 more than what it was sold for. (1 Mark)
A. Quantity I > Quantity II
B. Quantity I < Quantity II
C. Quantity I ≥ Quantity II
D. Quantity I ≤ Quantity II
E. Quantity I = Quantity II or No relation
Your Answer: Not Attempted

Correct Answer: A. Quantity I &gt; Quantity II <br>


for more join
Solution
https://t.me/currentAffairscLuB

Purchased by roshnipanwar025@gmail.com
Q58.
In the given questions, two quantities are given, one as Quantity I and another as Quantity II. You
have to determine relationship between two quantities and choose the appropriate option
https://store.adda247.com/#!/myTestAnalysis/printsolution/mappingId=25329/packageId=359/lang=ENGLISH 52/121
12/31/2017 Adda247 Store | Adda247 Store

A group consist of 4 couples in which each of the 4 persons have one wife
Quantity I : Number of ways in which they could be arranged in a straight line such that the men
and women occupy alternate positions
Quantity II: Eight times the number of ways in which they be seated around circular table such that
men and women occupy alternate position. (2 Mark)
A. Quantity I > Quantity II
B. Quantity I < Quantity II
C. Quantity I ≥ Quantity II
D. Quantity I ≤ Quantity II
E. Quantity I = Quantity II or No relation
Your Answer: Not Attempted

Correct Answer: E. Quantity I = Quantity II or No relation <br>

Solution

Purchased by roshnipanwar025@gmail.com
Q59.
Ramesh and Suresh decided to meet at a common point at the same time in the river. Ramesh
had to travel 42 km upstream in the river and Suresh had to travel less distance downstream than
that of Ramesh to meet at a common point. They both sets o in their respective boats at the
same time and speed of Ramesh’s boat is 20 km/hr more than the speed of Suresh boat. It is given
that Suresh covers 280 km upstream in 35 hours.
Find the speed of stream of river? (2 Mark)
A. 6 km/hr
B. 8 km/hr
C. 5 km/hr
D. 10 km/hr
E. 4 km/hr
Your Answer: Not Attempted for more join
Correct Answer: C. 5 km/hr <br> https://t.me/currentAffairscLuB
Solution

Purchased by roshnipanwar025@gmail.com
Q60.
https://store.adda247.com/#!/myTestAnalysis/printsolution/mappingId=25329/packageId=359/lang=ENGLISH 53/121
12/31/2017 Adda247 Store | Adda247 Store
Ramesh and Suresh decided to meet at a common point at the same time in the river. Ramesh
had to travel 42 km upstream in the river and Suresh had to travel less distance downstream than
that of Ramesh to meet at a common point. They both sets o in their respective boats at the
same time and speed of Ramesh’s boat is 20 km/hr more than the speed of Suresh boat. It is given
that Suresh covers 280 km upstream in 35 hours.
After meeting, if they decided to return to their original places but Ramesh travelled for 19 km and
Suresh travelled for 16 km, then what is the sum of time taken by both in covering these
distances? (1 Mark)
A. 150 min
B. 120 min
C. 180 min
D. 90 min
E. 60 min
Your Answer: Not Attempted

Correct Answer: A. 150 min <br>

Solution

Purchased by roshnipanwar025@gmail.com
Q61.
Data show the di erent kind of solids in a toy shop. Shopkeeper or (toymaker) makes di erent
types of toys by joining these solids. Some values are missing, you have to calculate these values if
rquired to answer the question.

A toymaker makes a toy in which a cone is mounted on the base of a hemisphere. If the total
surface area of the toy is then nd the volume of the toy? (2 Mark)
A.
B. for more join
C.
D. https://t.me/currentAffairscLuB
E.
Your Answer: Not Attempted

Correct Answer: A. <img


src="https://d37ohbyy0w8xn8.cloudfront.net/1510829576497image8.png"> <br>

https://store.adda247.com/#!/myTestAnalysis/printsolution/mappingId=25329/packageId=359/lang=ENGLISH 54/121
12/31/2017 Adda247 Store | Adda247 Store
Solution

Purchased by roshnipanwar025@gmail.com
Q62.
Data show the di erent kind of solids in a toy shop. Shopkeeper or (toymaker) makes di erent
types of toys by joining these solids. Some values are missing, you have to calculate these values if
rquired to answer the question.

Toymaker mounted the cube on the cylinder such that cylinder top is exactly in the middle of the
face of the cube. Find the total surface of the toy formed, if the height of formed toy is twice the
height of cylinder and curved surface area of cylinder is 66 times the height of cylinder (2 Mark)
A. 3125 cm²
B. 2794.5 cm²
C. 4112 cm²
D. 5123 cm²
E. None of these
Your Answer: Not Attempted

Correct Answer: E. None of these <br>

Solution

Purchased by roshnipanwar025@gmail.com
Q63.
Data show the di erent kind of solids in a toy shop. Shopkeeper or (toymaker) makes di erent
types of toys by joining these solids. Some values are missing, you have to calculate these values if
rquired to answer the question.

for more join


https://t.me/currentAffairscLuB
If given sphere is cut into two hemisphere and these hemispheres are mounted on both ends of
the cylinder, then nd out the ratio of volumes of toy formed by joining both hemispheres on
cylinder, cylinder and sphere. (2 Mark)
A. 7 : 6 : 13
B. 6 : 13 : 7
C. 13 : 6 : 7
https://store.adda247.com/#!/myTestAnalysis/printsolution/mappingId=25329/packageId=359/lang=ENGLISH 55/121
12/31/2017 Adda247 Store | Adda247 Store
D. 13 : 7 : 6
E. None of these
Your Answer: Not Attempted

Correct Answer: C. 13 : 6 : 7 <br>

Solution

Purchased by roshnipanwar025@gmail.com
Q64.
Data show the di erent kind of solids in a toy shop. Shopkeeper or (toymaker) makes di erent
types of toys by joining these solids. Some values are missing, you have to calculate these values if
rquired to answer the question.

Volume of the cuboid is approximately what percent more or less than the volume of cone if slant
height of cone is 25 cm and the breadth of the cuboid is 25% of the height of cone. (2 Mark)
A. 7%
B. 11%
C. 14%
D. 17%
E. 21%
Your Answer: Not Attempted

Correct Answer: D. 17% <br>

Solution

Purchased by roshnipanwar025@gmail.com
Q65.
for more join
A solid right circular cylinder has radiushttps://t.me/currentAffairscLuB
r and height 5r. A solid right circular cone is carved out
from one end of the base of cylinder. If base radius of cone is r and height is 2√2 r then, nd the
ratio between total surface area of cone to the total surface area of remaining part of cylinder. (2
Mark)
A. 3 : 5
B. 4 : 7
C. 2 : 7
https://store.adda247.com/#!/myTestAnalysis/printsolution/mappingId=25329/packageId=359/lang=ENGLISH 56/121
12/31/2017 Adda247 Store | Adda247 Store
D. 3 : 4
E. 1 : 3
Your Answer: Not Attempted

Correct Answer: C. 2 : 7 <br>

Solution

Purchased by roshnipanwar025@gmail.com
Q66.
In a circus there are two Jokers Sunty and Bunty who juggle the balls. Each one has di erent
number of balls of di erent color (Red, Green and White). No one have the same number of balls
of same color. Ratio of total number of balls having Sunty to that of Bunty is 2 : 3. If each Joker
choose one ball from their respective set of balls then probability of getting Green ball are same
for both Jokers. The number of white balls with Shunty is equal to the di erence between the
number of Green and Red balls with him. The number of Red and White balls with Bunty is more
than the number of green balls with him. Shunty has Red ball greater than Green ball which is
equal to the number of white ball of Bunty. Total number of Green balls is 15 and total number of
white balls is 8.
When Shunty picks 3 balls randomly, what is the probability of getting all green balls from the set?
(2 Mark)
A. 5/91
B. 13/95
C. 6/95
D. 17/91
E. None of these
Your Answer: Not Attempted

Correct Answer: A. 5/91 <br>


for more join
Solution
https://t.me/currentAffairscLuB

Purchased by roshnipanwar025@gmail.com
Q67.
In a circus there are two Jokers Sunty and Bunty who juggle the balls. Each one has di erent
number of balls of di erent color (Red, Green and White). No one have the same number of balls
https://store.adda247.com/#!/myTestAnalysis/printsolution/mappingId=25329/packageId=359/lang=ENGLISH 57/121
12/31/2017 Adda247 Store | Adda247 Store
of same color. Ratio of total number of balls having Sunty to that of Bunty is 2 : 3. If each Joker
choose one ball from their respective set of balls then probability of getting Green ball are same
for both Jokers. The number of white balls with Shunty is equal to the di erence between the
number of Green and Red balls with him. The number of Red and White balls with Bunty is more
than the number of green balls with him. Shunty has Red ball greater than Green ball which is
equal to the number of white ball of Bunty. Total number of Green balls is 15 and total number of
white balls is 8.
Bunty choose 3 ball for juggling, what is the probability that all the chosen balls are of di erent
color. (2 Mark)
A. 19/38
B. 9/38
C. 5/38
D. 3/38
E. None of these
Your Answer: Not Attempted

Correct Answer: B. 9/38 <br>

Solution

Purchased by roshnipanwar025@gmail.com
Q68.
In a circus there are two Jokers Sunty and Bunty who juggle the balls. Each one has di erent
number of balls of di erent color (Red, Green and White). No one have the same number of balls
of same color. Ratio of total number of balls having Sunty to that of Bunty is 2 : 3. If each Joker
choose one ball from their respective set of balls then probability of getting Green ball are same
for both Jokers. The number of white balls with Shunty is equal to the di erence between the
number of Green and Red balls with him. The number of Red and White balls with Bunty is more
than the number of green balls with him. Shunty has Red ball greater than Green ball which is
equal to the number of white ball of Bunty. Total number of Green balls is 15 and total number of
white balls is 8. for more join
Shunty is good juggler as compare to Bunty, and chances of performance being performed by
https://t.me/currentAffairscLuB
Shunty is 60%. What is the probability that Shunty performed with 4 balls in which in which atleast
2 balls are Red. (2 Mark)
A. 101/325
B. 309/715
C. 103/325
D. Can’t be determined
E. None of these
https://store.adda247.com/#!/myTestAnalysis/printsolution/mappingId=25329/packageId=359/lang=ENGLISH 58/121
12/31/2017 Adda247 Store | Adda247 Store
Your Answer: Not Attempted

Correct Answer: B. 309/715 <br>

Solution

Purchased by roshnipanwar025@gmail.com
Q69.
Some amount was lent at 6% per annum simple interest. After one year, Rs. 6800 is repaid and the
rest of the amount is repaid at 5% per annum. If the second year’s interest is 11/20 of rst year’s
interest, then nd what amount of money was lent out. (1 Mark)
A. 16,800
B. 17,000
C. 16,500
D. 18,000
E. 15,500
Your Answer: Not Attempted

Correct Answer: B. 17,000 <br>

Solution

Purchased by roshnipanwar025@gmail.com
Q70.
Anuj, Bibhuti and Chandu can lay 432 m of wires together in 8 days. In a day, Chandu can lay as
many more meters of wire than Bibhuti as Bibhuti can lay more than Anuj. Chandu’s 5 days of
work is equivalent to Anuj’s 7 days of work.How many meters of wire can Anuj alone lay in a day?
(2 Mark)
A. 9 m
B. 15 m
C. 18 m
for more join
D. 21m https://t.me/currentAffairscLuB
E. none of these
Your Answer: Not Attempted

Correct Answer: B. 15 m <br>

https://store.adda247.com/#!/myTestAnalysis/printsolution/mappingId=25329/packageId=359/lang=ENGLISH 59/121
12/31/2017 Adda247 Store | Adda247 Store
Solution

Purchased by roshnipanwar025@gmail.com
Q71.
The following questions are accompanied by three statements A, B and C. You have to determine
which statement(s) is/are necessary/su cient to answer the question.

What is the 10th term of an arithmetic progression?

A. Seventh term of the AP is 32 more than the third term of the AP.
B. The average of rst ve terms of the AP is 20.
C. The sum of third and fth terms of the AP is 56. (1 Mark)
A. Only A and B together
B. Only A and C together
C. Only B and C together
D. Any two of the three together
E. All the three together
Your Answer: Not Attempted

Correct Answer: D. Any two of the three together <br>

Solution

Purchased by roshnipanwar025@gmail.com
Q72.
The following questions are accompanied by three statements A, B and C. You have to determine
which statement(s) is/are necessary/su cient to answer the question.

What is the ratio of speed of the boat in still water and speed of the stream?

for more join


A. The boat takes 2.5 hours in travelling 45 km upstream.
https://t.me/currentAffairscLuB
B. The ratio of downstream and upstream speeds is 5 : 3.
C. The boat goes 45 km downstream in 1.5 hours. (1 Mark)
A. All the three together are not su cient
B. Only B alone
C. All the three together
D. Only A and C together
E. Either B alone or A and C together
https://store.adda247.com/#!/myTestAnalysis/printsolution/mappingId=25329/packageId=359/lang=ENGLISH 60/121
12/31/2017 Adda247 Store | Adda247 Store
Your Answer: Not Attempted

Correct Answer: E. Either B alone or A and C together <br>

Solution

Purchased by roshnipanwar025@gmail.com
Q73.
The following questions are accompanied by three statements A, B and C. You have to determine
which statement(s) is/are necessary/su cient to answer the question.

What is the rate of interest per annum?

A. Di erence between compound interest and simple interest for 2 years on the same amount at
same rate of interest is Rs.60.
B. Simple interest for ve years on the same amount at the same rate is one-fourth of the principal
amount.
C. Compound interest for three years on Rs.16000 at the same rate is Rs.2522. (2 Mark)
A. Only B alone
B. Only C alone
C. Only A alone
D. Any one of the three alone
E. Either B alone or C alone
Your Answer: Not Attempted

Correct Answer: E. Either B alone or C alone <br>

Solution

Purchased by roshnipanwar025@gmail.com
Q74.
for more join
https://t.me/currentAffairscLuB
The following questions are accompanied by three statements A, B and C. You have to determine
which statement(s) is/are necessary/su cient to answer the question.

There are some boys and girls in a class. How many girls are there in the class?

A. Ratio of number of boys and girls in the class is 5 : 3.

https://store.adda247.com/#!/myTestAnalysis/printsolution/mappingId=25329/packageId=359/lang=ENGLISH 61/121
12/31/2017 Adda247 Store | Adda247 Store
B. If one student is selected at random from the class, the probability of the selected student being
a boy is 5/8.
C. Number of boys in the class is 40 more than the number of girls in the class. (1 Mark)
A. Any two of the three together
B. Only A and C together
C. Either A and C together or B and C together
D. Only A and B together
E. Only B and C together
Your Answer: Not Attempted

Correct Answer: C. Either A and C together or B and C together <br>

Solution

Purchased by roshnipanwar025@gmail.com
Q75.
The following questions are accompanied by three statements A, B and C. You have to determine
which statement(s) is/are necessary/su cient to answer the question.

What is the pro t/loss % earned by the shopkeeper after selling an article?
A. After selling 4 articles, shopkeeper had a pro t equal to the selling price of one such article.
B. If the shopkeeper had sold one article at Rs.60 less than the original selling price, he would have
had pro t.
C. If he had sold one article at 25% less than the original selling price, he would have had no pro t
or loss. (2 Mark)
A. Either A alone or C alone
B. Only A alone
C. Only B alone
D. Only C alone
E. Any one of three alone
Your Answer: Not Attempted for more join
https://t.me/currentAffairscLuB
Correct Answer: A. Either A alone or C alone <br>

Solution

Purchased by roshnipanwar025@gmail.com
Q76.
https://store.adda247.com/#!/myTestAnalysis/printsolution/mappingId=25329/packageId=359/lang=ENGLISH 62/121
12/31/2017 Adda247 Store | Adda247 Store
Read the following and answer the questions that follow.

Two friends Shayam and Kailash own two versions of a car. Shayam owns the diesel version of the
car, while Kailash owns the petrol version. Kailash’s car gives an average that is 20% higher than
Shayam’s (in terms of litres per kilometer). It is known that petrol costs 60% of its price higher than
diesel.

The ratio of the cost per kilometer of Kailash’s car to Shayam’s car is (2 Mark)
A. 3 : 1
B. 1 : 3
C. 1.92 : 1
D. 2 : 1
E. Cannot be determined
Your Answer: Not Attempted

Correct Answer: A. 3 : 1 <br>

Solution

Purchased by roshnipanwar025@gmail.com
Q77.
Read the following and answer the questions that follow.

Two friends Shayam and Kailash own two versions of a car. Shayam owns the diesel version of the
car, while Kailash owns the petrol version. Kailash’s car gives an average that is 20% higher than
Shayam’s (in terms of litres per kilometer). It is known that petrol costs 60% of its price higher than
diesel.

If Shyam’s car gives an average of 20 km per litre and diesel cost Rs. 12.5 per litre, then the
di erence in the cost of travel per kilometer between the two cars is (2 Mark)
A. Rs. 4.3 for more join
B. Rs. 3.5
C. Rs. 2.5 https://t.me/currentAffairscLuB
D. Rs. 3
E. Rs. 1.25
Your Answer: Not Attempted

Correct Answer: E. Rs. 1.25 <br>

https://store.adda247.com/#!/myTestAnalysis/printsolution/mappingId=25329/packageId=359/lang=ENGLISH 63/121
12/31/2017 Adda247 Store | Adda247 Store
Solution

Purchased by roshnipanwar025@gmail.com
Q78.
Study the following bar graph carefully and answer the following question

Total investment (in thousand) of Abhimanyu and Gaurav in 6 di erent schemes (A, B, C, D, E, F)
and percentage of Abhimanyu share in total investment

If Scheme A o ers simple interest of R% percent per annum and share of interest earned by
Abhimanyu in scheme A is 1100 then nd the value of R% if investment is made for 2 year. (2
Mark)
A.
B.
C.
D.
E. None of these
Your Answer: Not Attempted

Correct Answer: A. <img


src="https://d37ohbyy0w8xn8.cloudfront.net/1510829576511image17.png"> <br>

Solution

Purchased by roshnipanwar025@gmail.com
Q79.
Study the following bar graph carefully and answer the following question
for more join
https://t.me/currentAffairscLuB
Total investment (in thousand) of Abhimanyu and Gaurav in 6 di erent schemes (A, B, C, D, E, F)
and percentage of Abhimanyu share in total investment

Average of investment made by Abhimanyu in scheme B and C together is what % more or less
than average of investment made by Gaurav in scheme A and F together (Approximately) (1 Mark)
https://store.adda247.com/#!/myTestAnalysis/printsolution/mappingId=25329/packageId=359/lang=ENGLISH 64/121
12/31/2017 Adda247 Store | Adda247 Store
A. 22%
B. 18%
C. 24%
D. 37%
E. 27%
Your Answer: Not Attempted

Correct Answer: E. 27%

Solution

Purchased by roshnipanwar025@gmail.com
Q80.
Study the following bar graph carefully and answer the following question

Total investment (in thousand) of Abhimanyu and Gaurav in 6 di erent schemes (A, B, C, D, E, F)
and percentage of Abhimanyu share in total investment

If scheme B and C o ers simple interest at the rate of 10% and respectively. Then nd the total
interest obtained from scheme B and C in 3 years. Gaurav invested in scheme B for 2 years and
Abhimanyu invested in Schemes C for 2 years. (2 Mark)
A. 45275
B. 43340
C. 38270
D. 32350
E. None of these
Your Answer: Not Attempted
for more join
Correct Answer: E. None of these <br>
https://t.me/currentAffairscLuB
Solution

Purchased by roshnipanwar025@gmail.com
Q81.

https://store.adda247.com/#!/myTestAnalysis/printsolution/mappingId=25329/packageId=359/lang=ENGLISH 65/121
12/31/2017 Adda247 Store | Adda247 Store
Each passage is followed by questions based on its content. After reading passage, choose the
best answer to each question. Answer all questions following a passage on the basis of what is
stated or implied in that passage.

Paragraph 1: Surprisingly enough, modern historians have rarely interested themselves in the
history of the American South in the period before the South began to become self-consciously
and distinctively “Southern” — the decades after 1815. Consequently, the cultural history of
Britain’s North American empire in the seventeenth and eighteenth centuries has been written
almost as if the Southern colonies had never existed. The American culture that emerged during
the Colonial and Revolutionary eras has been depicted as having been simply an extension of New
England Puritan culture. However, Professor Davis has recently argued that the South stood apart
from the rest of American society during this early period, following its own unique pattern of
cultural development. The case for Southern distinctiveness rests upon two related premises: rst,
that the cultural similarities among the ve Southern colonies were far more impressive than the
di erences, and second, that what made those colonies alike also made them di erent from the
other colonies. The rst, for which Davis o ers an enormous amount of evidence, can be accepted
without major reservations; the second is far more problematic.
Paragraph 2: What makes the second premise problematic is the use of the Puritan colonies as a
basis for comparison. Quite properly, Davis decries the excessive in uence ascribed by historians
to the Puritans in the formation of American culture. Yet Davis inadvertently adds weight to such
ascriptions by using the Puritans as the standard against which to assess the achievements and
contributions of Southern colonials. Throughout, Davis focuses on the important, and undeniable,
di erences between the Southern and Northern colonies in motives for and patterns of early
settlement, in attitudes toward nature and Native Americans, and in the degree of receptivity to
metropolitan cultural in uences.
Paragraph 3: However, recent scholarship has strongly suggested that those aspects of early New
England culture that seem to have been most distinctly Puritan, such as the strong religious
orientation and the communal impulse, were not even typical of New England as a whole, but
were largely con ned to the two colonies of America. Thus, what in contrast to the Puritan
(Northern) colonies appears to Davis to be peculiarly Southern — acquisitiveness, a strong interest
in politics and the law, and a tendency to cultivate metropolitan cultural models — was not only
for more join
more typically English than the cultural patterns exhibited by Puritan Massachusetts and
Connecticut, but also almost certainly characteristic of most other early modern British colonies
https://t.me/currentAffairscLuB
from Barbados north to Rhode Island and New Hampshire. Within the larger framework of
American colonial life, then, not the Southern but the Northern colonies appear to have been
distinctive, and even they seem to have been rapidly assimilating to the dominant cultural
patterns by the last Colonial period.

The author is primarily concerned with (1.5 Mark)

https://store.adda247.com/#!/myTestAnalysis/printsolution/mappingId=25329/packageId=359/lang=ENGLISH 66/121
12/31/2017 Adda247 Store | Adda247 Store
A. refuting a claim about the in uence of Puritan culture on the early American South.
B. refuting a thesis about the distinctiveness of the culture of the early American South.
C. refuting the two premises that underlie Davis-discussion of the culture of the American South.
D. challenging the hypothesis that early American culture was homogeneous in nature.
E. challenging the in uence of Puritan culture that early American culture was homogeneous in
nature.
Your Answer: Not Attempted

Correct Answer: B. refuting a thesis about the distinctiveness of the culture of the early American
South. <br>

Solution
The author is primarily concerned with refuting a thesis about the distinctiveness of the culture of
the early American South, hence option (b) is the answer. (It is mentioned in the last line of the
passage). Option (a) is incorrect as the keyword claim is not mentioned in the passage. Similarly,
options (c) and (d) are incorrect. Hence, the answer is option (b).
Purchased by roshnipanwar025@gmail.com
Q82.
Each passage is followed by questions based on its content. After reading passage, choose the
best answer to each question. Answer all questions following a passage on the basis of what is
stated or implied in that passage.

Paragraph 1: Surprisingly enough, modern historians have rarely interested themselves in the
history of the American South in the period before the South began to become self-consciously
and distinctively “Southern” — the decades after 1815. Consequently, the cultural history of
Britain’s North American empire in the seventeenth and eighteenth centuries has been written
almost as if the Southern colonies had never existed. The American culture that emerged during
the Colonial and Revolutionary eras has been depicted as having been simply an extension of New
England Puritan culture. However, Professor Davis has recently argued that the South stood apart
from the rest of American society during this early period, following its own unique pattern of
cultural development. The case for Southern distinctiveness rests upon two related premises: rst,
that the cultural similarities among the ve Southern colonies were far more impressive than the
for more join
di erences, and second, that what made those colonies alike also made them di erent from the
https://t.me/currentAffairscLuB
other colonies. The rst, for which Davis o ers an enormous amount of evidence, can be accepted
without major reservations; the second is far more problematic.
Paragraph 2: What makes the second premise problematic is the use of the Puritan colonies as a
basis for comparison. Quite properly, Davis decries the excessive in uence ascribed by historians
to the Puritans in the formation of American culture. Yet Davis inadvertently adds weight to such
ascriptions by using the Puritans as the standard against which to assess the achievements and
contributions of Southern colonials. Throughout, Davis focuses on the important, and undeniable,
https://store.adda247.com/#!/myTestAnalysis/printsolution/mappingId=25329/packageId=359/lang=ENGLISH 67/121
12/31/2017 Adda247 Store | Adda247 Store
di erences between the Southern and Northern colonies in motives for and patterns of early
settlement, in attitudes toward nature and Native Americans, and in the degree of receptivity to
metropolitan cultural in uences.
Paragraph 3: However, recent scholarship has strongly suggested that those aspects of early New
England culture that seem to have been most distinctly Puritan, such as the strong religious
orientation and the communal impulse, were not even typical of New England as a whole, but
were largely con ned to the two colonies of America. Thus, what in contrast to the Puritan
(Northern) colonies appears to Davis to be peculiarly Southern — acquisitiveness, a strong interest
in politics and the law, and a tendency to cultivate metropolitan cultural models — was not only
more typically English than the cultural patterns exhibited by Puritan Massachusetts and
Connecticut, but also almost certainly characteristic of most other early modern British colonies
from Barbados north to Rhode Island and New Hampshire. Within the larger framework of
American colonial life, then, not the Southern but the Northern colonies appear to have been
distinctive, and even they seem to have been rapidly assimilating to the dominant cultural
patterns by the last Colonial period.

The passage implies that the attitudes toward Native Americans that prevailed in the Southern
colonies (1.5 Mark)
A. were in con ict with the cosmopolitan outlook of the South.
B. derived from Southerners' strong interest in the law.
C. weremodeled after those that prevailed in the North.
D. di ered from those that prevailed in the Puritan colonies.
E. were modeled after the cosmopolitan outlook in the North.
Your Answer: Not Attempted

Correct Answer: D. di ered from those that prevailed in the Puritan colonies. <br>

Solution
The passage implies that the attitudes Native Americans that prevailed in the Southern colonies
di ered from those that prevailed in the Puritan colonies, hence answer is option (d). It is
mentioned in the passage about the attitudes towards Native Americans that prevailed in the
for more join
Southern colonies. Other options are incorrect as per the passage given. Hence, the answer is
option (d).
https://t.me/currentAffairscLuB
Purchased by roshnipanwar025@gmail.com
Q83.
Each passage is followed by questions based on its content. After reading passage, choose the
best answer to each question. Answer all questions following a passage on the basis of what is
stated or implied in that passage.

Paragraph 1: Surprisingly enough, modern historians have rarely interested themselves in the
https://store.adda247.com/#!/myTestAnalysis/printsolution/mappingId=25329/packageId=359/lang=ENGLISH 68/121
12/31/2017 Adda247 Store | Adda247 Store
history of the American South in the period before the South began to become self-consciously
and distinctively “Southern” — the decades after 1815. Consequently, the cultural history of
Britain’s North American empire in the seventeenth and eighteenth centuries has been written
almost as if the Southern colonies had never existed. The American culture that emerged during
the Colonial and Revolutionary eras has been depicted as having been simply an extension of New
England Puritan culture. However, Professor Davis has recently argued that the South stood apart
from the rest of American society during this early period, following its own unique pattern of
cultural development. The case for Southern distinctiveness rests upon two related premises: rst,
that the cultural similarities among the ve Southern colonies were far more impressive than the
di erences, and second, that what made those colonies alike also made them di erent from the
other colonies. The rst, for which Davis o ers an enormous amount of evidence, can be accepted
without major reservations; the second is far more problematic.
Paragraph 2: What makes the second premise problematic is the use of the Puritan colonies as a
basis for comparison. Quite properly, Davis decries the excessive in uence ascribed by historians
to the Puritans in the formation of American culture. Yet Davis inadvertently adds weight to such
ascriptions by using the Puritans as the standard against which to assess the achievements and
contributions of Southern colonials. Throughout, Davis focuses on the important, and undeniable,
di erences between the Southern and Northern colonies in motives for and patterns of early
settlement, in attitudes toward nature and Native Americans, and in the degree of receptivity to
metropolitan cultural in uences.
Paragraph 3: However, recent scholarship has strongly suggested that those aspects of early New
England culture that seem to have been most distinctly Puritan, such as the strong religious
orientation and the communal impulse, were not even typical of New England as a whole, but
were largely con ned to the two colonies of America. Thus, what in contrast to the Puritan
(Northern) colonies appears to Davis to be peculiarly Southern — acquisitiveness, a strong interest
in politics and the law, and a tendency to cultivate metropolitan cultural models — was not only
more typically English than the cultural patterns exhibited by Puritan Massachusetts and
Connecticut, but also almost certainly characteristic of most other early modern British colonies
from Barbados north to Rhode Island and New Hampshire. Within the larger framework of
American colonial life, then, not the Southern but the Northern colonies appear to have been
distinctive, and even they seem to have been rapidly assimilating to the dominant cultural
patterns by the last Colonial period. for more join
https://t.me/currentAffairscLuB
According to the author, the depiction of American culture during the Colonial and Revolutionary
ears as an extension of New England Puritan culture re ects the (1.5 Mark)
A. fact that historians have overestimated the importance of the Puritans in the development of
American culture.
B. fact that early American culture was deeply in uenced by the strong religious orientation of the
colonists.

https://store.adda247.com/#!/myTestAnalysis/printsolution/mappingId=25329/packageId=359/lang=ENGLISH 69/121
12/31/2017 Adda247 Store | Adda247 Store
C. extent to which Massachusetts and Connecticut served as cultural models for the other
American colonies.
D. extent to which colonial America resisted assimilating cultural patterns that were typically
English.
E. fact that early American culture fact that early American cultureagreement
Your Answer: Not Attempted

Correct Answer: A. fact that historians have overestimated the importance of the Puritans in the
development of American culture. <br>

Solution
As it is given that – The American culture that emerged during the Colonial and Revolutionary eras
has been depicted as having been simply an extension of New England Puritan culture, therefore,
the answer is option (a), which is clearly mentioned in the passage. Option (b), is incorrect, as it is
not re ecting the culture. Similarly, options (c) and (d) are incorrect. Hence, the answer is option
(a).
Purchased by roshnipanwar025@gmail.com
Q84.
Each passage is followed by questions based on its content. After reading passage, choose the
best answer to each question. Answer all questions following a passage on the basis of what is
stated or implied in that passage.

Paragraph 1: Surprisingly enough, modern historians have rarely interested themselves in the
history of the American South in the period before the South began to become self-consciously
and distinctively “Southern” — the decades after 1815. Consequently, the cultural history of
Britain’s North American empire in the seventeenth and eighteenth centuries has been written
almost as if the Southern colonies had never existed. The American culture that emerged during
the Colonial and Revolutionary eras has been depicted as having been simply an extension of New
England Puritan culture. However, Professor Davis has recently argued that the South stood apart
from the rest of American society during this early period, following its own unique pattern of
cultural development. The case for Southern distinctiveness rests upon two related premises: rst,
that the cultural similarities among the ve Southern colonies were far more impressive than the
for more join
di erences, and second, that what made those colonies alike also made them di erent from the
https://t.me/currentAffairscLuB
other colonies. The rst, for which Davis o ers an enormous amount of evidence, can be accepted
without major reservations; the second is far more problematic.
Paragraph 2: What makes the second premise problematic is the use of the Puritan colonies as a
basis for comparison. Quite properly, Davis decries the excessive in uence ascribed by historians
to the Puritans in the formation of American culture. Yet Davis inadvertently adds weight to such
ascriptions by using the Puritans as the standard against which to assess the achievements and
contributions of Southern colonials. Throughout, Davis focuses on the important, and undeniable,
https://store.adda247.com/#!/myTestAnalysis/printsolution/mappingId=25329/packageId=359/lang=ENGLISH 70/121
12/31/2017 Adda247 Store | Adda247 Store
di erences between the Southern and Northern colonies in motives for and patterns of early
settlement, in attitudes toward nature and Native Americans, and in the degree of receptivity to
metropolitan cultural in uences.
Paragraph 3: However, recent scholarship has strongly suggested that those aspects of early New
England culture that seem to have been most distinctly Puritan, such as the strong religious
orientation and the communal impulse, were not even typical of New England as a whole, but
were largely con ned to the two colonies of America. Thus, what in contrast to the Puritan
(Northern) colonies appears to Davis to be peculiarly Southern — acquisitiveness, a strong interest
in politics and the law, and a tendency to cultivate metropolitan cultural models — was not only
more typically English than the cultural patterns exhibited by Puritan Massachusetts and
Connecticut, but also almost certainly characteristic of most other early modern British colonies
from Barbados north to Rhode Island and New Hampshire. Within the larger framework of
American colonial life, then, not the Southern but the Northern colonies appear to have been
distinctive, and even they seem to have been rapidly assimilating to the dominant cultural
patterns by the last Colonial period.

The author of the passage is in agreement with which of the following elements of Davis’ book?

I. Davis' claim that acquisitiveness was a characteristic unique to the South during the Colonial
period.
II. Davis' argument that there were signi cant di erences between Puritan and Southern culture
during the Colonial period.
III. Davis' thesis that the Southern colonies shared a common culture. (1.5 Mark)
A. I only
B. II only
C. III only
D. II and III only
E. I and II only
Your Answer: Not Attempted

Correct Answer: D. II and III only <br>


for more join
Solution
https://t.me/currentAffairscLuB
The answer is option (d) as information II and III are mentioned in the passage, whereas
information I is not given in the passage. So, options (a), (b) and (c) are incorrect. Hence, the
answer is option (d).
Purchased by roshnipanwar025@gmail.com
Q85.
Each passage is followed by questions based on its content. After reading passage, choose the
best answer to each question. Answer all questions following a passage on the basis of what is
https://store.adda247.com/#!/myTestAnalysis/printsolution/mappingId=25329/packageId=359/lang=ENGLISH 71/121
12/31/2017 Adda247 Store | Adda247 Store
stated or implied in that passage.

Paragraph 1: Surprisingly enough, modern historians have rarely interested themselves in the
history of the American South in the period before the South began to become self-consciously
and distinctively “Southern” — the decades after 1815. Consequently, the cultural history of
Britain’s North American empire in the seventeenth and eighteenth centuries has been written
almost as if the Southern colonies had never existed. The American culture that emerged during
the Colonial and Revolutionary eras has been depicted as having been simply an extension of New
England Puritan culture. However, Professor Davis has recently argued that the South stood apart
from the rest of American society during this early period, following its own unique pattern of
cultural development. The case for Southern distinctiveness rests upon two related premises: rst,
that the cultural similarities among the ve Southern colonies were far more impressive than the
di erences, and second, that what made those colonies alike also made them di erent from the
other colonies. The rst, for which Davis o ers an enormous amount of evidence, can be accepted
without major reservations; the second is far more problematic.
Paragraph 2: What makes the second premise problematic is the use of the Puritan colonies as a
basis for comparison. Quite properly, Davis decries the excessive in uence ascribed by historians
to the Puritans in the formation of American culture. Yet Davis inadvertently adds weight to such
ascriptions by using the Puritans as the standard against which to assess the achievements and
contributions of Southern colonials. Throughout, Davis focuses on the important, and undeniable,
di erences between the Southern and Northern colonies in motives for and patterns of early
settlement, in attitudes toward nature and Native Americans, and in the degree of receptivity to
metropolitan cultural in uences.
Paragraph 3: However, recent scholarship has strongly suggested that those aspects of early New
England culture that seem to have been most distinctly Puritan, such as the strong religious
orientation and the communal impulse, were not even typical of New England as a whole, but
were largely con ned to the two colonies of America. Thus, what in contrast to the Puritan
(Northern) colonies appears to Davis to be peculiarly Southern — acquisitiveness, a strong interest
in politics and the law, and a tendency to cultivate metropolitan cultural models — was not only
more typically English than the cultural patterns exhibited by Puritan Massachusetts and
Connecticut, but also almost certainly characteristic of most other early modern British colonies
for more join
from Barbados north to Rhode Island and New Hampshire. Within the larger framework of
American colonial life, then, not the Southern but the Northern colonies appear to have been
https://t.me/currentAffairscLuB
distinctive, and even they seem to have been rapidly assimilating to the dominant cultural
patterns by the last Colonial period.

The passage suggests that by the late Colonial period the tendency to cultivate metropolitan
cultural models was a cultural pattern that was (1.5 Mark)
A. dying out as Puritan in uence began to grow.

https://store.adda247.com/#!/myTestAnalysis/printsolution/mappingId=25329/packageId=359/lang=ENGLISH 72/121
12/31/2017 Adda247 Store | Adda247 Store
B. self-consciously and distinctively Southern.
C. spreading to Massachusetts and Connecticut.
D. more characteristic of the Southern colonies than of England.
E. in uenced by the late Colonial perioddistinctively
Your Answer: Not Attempted

Correct Answer: C. spreading to Massachusetts and Connecticut. <br>

Solution
The answer is option (c) that is spreading to Massachusetts and Connecticut. Other options are
incorrect as per the condition given in the question above. Hence, the answer is option (c).
Purchased by roshnipanwar025@gmail.com
Q86.
Each passage is followed by questions based on its content. After reading passage, choose the
best answer to each question. Answer all questions following a passage on the basis of what is
stated or implied in that passage.

Paragraph 1: Surprisingly enough, modern historians have rarely interested themselves in the
history of the American South in the period before the South began to become self-consciously
and distinctively “Southern” — the decades after 1815. Consequently, the cultural history of
Britain’s North American empire in the seventeenth and eighteenth centuries has been written
almost as if the Southern colonies had never existed. The American culture that emerged during
the Colonial and Revolutionary eras has been depicted as having been simply an extension of New
England Puritan culture. However, Professor Davis has recently argued that the South stood apart
from the rest of American society during this early period, following its own unique pattern of
cultural development. The case for Southern distinctiveness rests upon two related premises: rst,
that the cultural similarities among the ve Southern colonies were far more impressive than the
di erences, and second, that what made those colonies alike also made them di erent from the
other colonies. The rst, for which Davis o ers an enormous amount of evidence, can be accepted
without major reservations; the second is far more problematic.
Paragraph 2: What makes the second premise problematic is the use of the Puritan colonies as a
basis for comparison. Quite properly, Davis decries the excessive in uence ascribed by historians
for more join
to the Puritans in the formation of American culture. Yet Davis inadvertently adds weight to such
ascriptions by using the Puritans as thehttps://t.me/currentAffairscLuB
standard against which to assess the achievements and
contributions of Southern colonials. Throughout, Davis focuses on the important, and undeniable,
di erences between the Southern and Northern colonies in motives for and patterns of early
settlement, in attitudes toward nature and Native Americans, and in the degree of receptivity to
metropolitan cultural in uences.
Paragraph 3: However, recent scholarship has strongly suggested that those aspects of early New
England culture that seem to have been most distinctly Puritan, such as the strong religious
https://store.adda247.com/#!/myTestAnalysis/printsolution/mappingId=25329/packageId=359/lang=ENGLISH 73/121
12/31/2017 Adda247 Store | Adda247 Store
orientation and the communal impulse, were not even typical of New England as a whole, but
were largely con ned to the two colonies of America. Thus, what in contrast to the Puritan
(Northern) colonies appears to Davis to be peculiarly Southern — acquisitiveness, a strong interest
in politics and the law, and a tendency to cultivate metropolitan cultural models — was not only
more typically English than the cultural patterns exhibited by Puritan Massachusetts and
Connecticut, but also almost certainly characteristic of most other early modern British colonies
from Barbados north to Rhode Island and New Hampshire. Within the larger framework of
American colonial life, then, not the Southern but the Northern colonies appear to have been
distinctive, and even they seem to have been rapidly assimilating to the dominant cultural
patterns by the last Colonial period.

Which of the following statements could most logically follow the last sentence of the passage?
(1.5 Mark)
A. Thus, had more attention been paid to the evidence, Davis would not have been tempted to
argue that the culture of the South diverged greatly from Puritan culture in the seventeenth
century.
B. Thus, convergence, not divergence, seems to have characterized the cultural development of
the American colonies in the eighteenth century.
C. Thus, without the cultural diversity represented by the America South, the culture of colonial
America would certainly have been homogeneous in nature.
D. Thus, the contribution of Southern colonials to American culture was certainly overshadowed
by that of the Puritans.
E. Thus, the contribution of Southern colonials have been temptedprimarily in American South
Your Answer: Not Attempted

Correct Answer: B. Thus, convergence, not divergence, seems to have characterized the cultural
development of the American colonies in the eighteenth century. <br>

Solution
Here option B most logically follows the last sentence of the passage and so option (b) is the
answer. It is given that – the larger framework of American colonial life, then, not the Southern but
for more join
the Northern colonies appear to have been distinctive, and even they seem to have been rapidly
assimilating to the dominant cultural patterns by the last colonial period. Options A, C and D are
https://t.me/currentAffairscLuB
incorrect, as per the passage is concerned. Hence, the answer is option B.
Purchased by roshnipanwar025@gmail.com
Q87.
Each passage is followed by questions based on its content. After reading passage, choose the
best answer to each question. Answer all questions following a passage on the basis of what is
stated or implied in that passage.

https://store.adda247.com/#!/myTestAnalysis/printsolution/mappingId=25329/packageId=359/lang=ENGLISH 74/121
12/31/2017 Adda247 Store | Adda247 Store
Paragraph 1: Surprisingly enough, modern historians have rarely interested themselves in the
history of the American South in the period before the South began to become self-consciously
and distinctively “Southern” — the decades after 1815. Consequently, the cultural history of
Britain’s North American empire in the seventeenth and eighteenth centuries has been written
almost as if the Southern colonies had never existed. The American culture that emerged during
the Colonial and Revolutionary eras has been depicted as having been simply an extension of New
England Puritan culture. However, Professor Davis has recently argued that the South stood apart
from the rest of American society during this early period, following its own unique pattern of
cultural development. The case for Southern distinctiveness rests upon two related premises: rst,
that the cultural similarities among the ve Southern colonies were far more impressive than the
di erences, and second, that what made those colonies alike also made them di erent from the
other colonies. The rst, for which Davis o ers an enormous amount of evidence, can be accepted
without major reservations; the second is far more problematic.
Paragraph 2: What makes the second premise problematic is the use of the Puritan colonies as a
basis for comparison. Quite properly, Davis decries the excessive in uence ascribed by historians
to the Puritans in the formation of American culture. Yet Davis inadvertently adds weight to such
ascriptions by using the Puritans as the standard against which to assess the achievements and
contributions of Southern colonials. Throughout, Davis focuses on the important, and undeniable,
di erences between the Southern and Northern colonies in motives for and patterns of early
settlement, in attitudes toward nature and Native Americans, and in the degree of receptivity to
metropolitan cultural in uences.
Paragraph 3: However, recent scholarship has strongly suggested that those aspects of early New
England culture that seem to have been most distinctly Puritan, such as the strong religious
orientation and the communal impulse, were not even typical of New England as a whole, but
were largely con ned to the two colonies of America. Thus, what in contrast to the Puritan
(Northern) colonies appears to Davis to be peculiarly Southern — acquisitiveness, a strong interest
in politics and the law, and a tendency to cultivate metropolitan cultural models — was not only
more typically English than the cultural patterns exhibited by Puritan Massachusetts and
Connecticut, but also almost certainly characteristic of most other early modern British colonies
from Barbados north to Rhode Island and New Hampshire. Within the larger framework of
American colonial life, then, not the Southern but the Northern colonies appear to have been
for more join
distinctive, and even they seem to have been rapidly assimilating to the dominant cultural
patterns by the last Colonial period.
https://t.me/currentAffairscLuB
Which of the following is SIMILAR in meaning of the word ‘ Consequently’ as used in the passage?
(1.5 Mark)
A. accordingly
B. alike
C. consensus

https://store.adda247.com/#!/myTestAnalysis/printsolution/mappingId=25329/packageId=359/lang=ENGLISH 75/121
12/31/2017 Adda247 Store | Adda247 Store
D. su cient
E. frequent
Your Answer: Not Attempted

Correct Answer: A. accordingly <br>

Solution
Consequently means as a result hence accordingly is the word which is most similar in meaning.
Purchased by roshnipanwar025@gmail.com
Q88.
Each passage is followed by questions based on its content. After reading passage, choose the
best answer to each question. Answer all questions following a passage on the basis of what is
stated or implied in that passage.

Paragraph 1: Surprisingly enough, modern historians have rarely interested themselves in the
history of the American South in the period before the South began to become self-consciously
and distinctively “Southern” — the decades after 1815. Consequently, the cultural history of
Britain’s North American empire in the seventeenth and eighteenth centuries has been written
almost as if the Southern colonies had never existed. The American culture that emerged during
the Colonial and Revolutionary eras has been depicted as having been simply an extension of New
England Puritan culture. However, Professor Davis has recently argued that the South stood apart
from the rest of American society during this early period, following its own unique pattern of
cultural development. The case for Southern distinctiveness rests upon two related premises: rst,
that the cultural similarities among the ve Southern colonies were far more impressive than the
di erences, and second, that what made those colonies alike also made them di erent from the
other colonies. The rst, for which Davis o ers an enormous amount of evidence, can be accepted
without major reservations; the second is far more problematic.
Paragraph 2: What makes the second premise problematic is the use of the Puritan colonies as a
basis for comparison. Quite properly, Davis decries the excessive in uence ascribed by historians
to the Puritans in the formation of American culture. Yet Davis inadvertently adds weight to such
ascriptions by using the Puritans as the standard against which to assess the achievements and
contributions of Southern colonials. Throughout, Davis focuses on the important, and undeniable,
for more join
di erences between the Southern and Northern colonies in motives for and patterns of early
https://t.me/currentAffairscLuB
settlement, in attitudes toward nature and Native Americans, and in the degree of receptivity to
metropolitan cultural in uences.
Paragraph 3: However, recent scholarship has strongly suggested that those aspects of early New
England culture that seem to have been most distinctly Puritan, such as the strong religious
orientation and the communal impulse, were not even typical of New England as a whole, but
were largely con ned to the two colonies of America. Thus, what in contrast to the Puritan
(Northern) colonies appears to Davis to be peculiarly Southern — acquisitiveness, a strong interest
https://store.adda247.com/#!/myTestAnalysis/printsolution/mappingId=25329/packageId=359/lang=ENGLISH 76/121
12/31/2017 Adda247 Store | Adda247 Store
in politics and the law, and a tendency to cultivate metropolitan cultural models — was not only
more typically English than the cultural patterns exhibited by Puritan Massachusetts and
Connecticut, but also almost certainly characteristic of most other early modern British colonies
from Barbados north to Rhode Island and New Hampshire. Within the larger framework of
American colonial life, then, not the Southern but the Northern colonies appear to have been
distinctive, and even they seem to have been rapidly assimilating to the dominant cultural
patterns by the last Colonial period.

Which of the following is SIMILAR in meaning of the word ‘ Ascribed’ as used in the passage? (1.5
Mark)
A. illness
B. aspire
C. severance
D. attire
E. attribute
Your Answer: Not Attempted

Correct Answer: E. attribute <br>

Solution
Ascribed-attribute, regard something as being due to (a cause),regard a text, quotation, or work of
art as being produced by or belonging to (a particular person or period),regard a quality as
belonging to.
Purchased by roshnipanwar025@gmail.com
Q89.
Each passage is followed by questions based on its content. After reading passage, choose the
best answer to each question. Answer all questions following a passage on the basis of what is
stated or implied in that passage.

Paragraph 1: Surprisingly enough, modern historians have rarely interested themselves in the
history of the American South in the period before the South began to become self-consciously
and distinctively “Southern” — the decades after 1815. Consequently, the cultural history of
for more join
Britain’s North American empire in the seventeenth and eighteenth centuries has been written
almost as if the Southern colonies had https://t.me/currentAffairscLuB
never existed. The American culture that emerged during
the Colonial and Revolutionary eras has been depicted as having been simply an extension of New
England Puritan culture. However, Professor Davis has recently argued that the South stood apart
from the rest of American society during this early period, following its own unique pattern of
cultural development. The case for Southern distinctiveness rests upon two related premises: rst,
that the cultural similarities among the ve Southern colonies were far more impressive than the
di erences, and second, that what made those colonies alike also made them di erent from the
https://store.adda247.com/#!/myTestAnalysis/printsolution/mappingId=25329/packageId=359/lang=ENGLISH 77/121
12/31/2017 Adda247 Store | Adda247 Store
other colonies. The rst, for which Davis o ers an enormous amount of evidence, can be accepted
without major reservations; the second is far more problematic.
Paragraph 2: What makes the second premise problematic is the use of the Puritan colonies as a
basis for comparison. Quite properly, Davis decries the excessive in uence ascribed by historians
to the Puritans in the formation of American culture. Yet Davis inadvertently adds weight to such
ascriptions by using the Puritans as the standard against which to assess the achievements and
contributions of Southern colonials. Throughout, Davis focuses on the important, and undeniable,
di erences between the Southern and Northern colonies in motives for and patterns of early
settlement, in attitudes toward nature and Native Americans, and in the degree of receptivity to
metropolitan cultural in uences.
Paragraph 3: However, recent scholarship has strongly suggested that those aspects of early New
England culture that seem to have been most distinctly Puritan, such as the strong religious
orientation and the communal impulse, were not even typical of New England as a whole, but
were largely con ned to the two colonies of America. Thus, what in contrast to the Puritan
(Northern) colonies appears to Davis to be peculiarly Southern — acquisitiveness, a strong interest
in politics and the law, and a tendency to cultivate metropolitan cultural models — was not only
more typically English than the cultural patterns exhibited by Puritan Massachusetts and
Connecticut, but also almost certainly characteristic of most other early modern British colonies
from Barbados north to Rhode Island and New Hampshire. Within the larger framework of
American colonial life, then, not the Southern but the Northern colonies appear to have been
distinctive, and even they seem to have been rapidly assimilating to the dominant cultural
patterns by the last Colonial period.

Which of the following is OPPOSITE in meaning of the word ‘ Enormous’ as used in the passage?
(1.5 Mark)
A. facilitate
B. liable
C. miniscule
D. enlist
E. industry
Your Answer: Not Attempted
for more join
Correct Answer: C. miniscule <br>
https://t.me/currentAffairscLuB
Solution
Enormous -very large in size, quantity, or extent.
Purchased by roshnipanwar025@gmail.com
Q90.
Each passage is followed by questions based on its content. After reading passage, choose the
best answer to each question. Answer all questions following a passage on the basis of what is
https://store.adda247.com/#!/myTestAnalysis/printsolution/mappingId=25329/packageId=359/lang=ENGLISH 78/121
12/31/2017 Adda247 Store | Adda247 Store
stated or implied in that passage.

Paragraph 1: Surprisingly enough, modern historians have rarely interested themselves in the
history of the American South in the period before the South began to become self-consciously
and distinctively “Southern” — the decades after 1815. Consequently, the cultural history of
Britain’s North American empire in the seventeenth and eighteenth centuries has been written
almost as if the Southern colonies had never existed. The American culture that emerged during
the Colonial and Revolutionary eras has been depicted as having been simply an extension of New
England Puritan culture. However, Professor Davis has recently argued that the South stood apart
from the rest of American society during this early period, following its own unique pattern of
cultural development. The case for Southern distinctiveness rests upon two related premises: rst,
that the cultural similarities among the ve Southern colonies were far more impressive than the
di erences, and second, that what made those colonies alike also made them di erent from the
other colonies. The rst, for which Davis o ers an enormous amount of evidence, can be accepted
without major reservations; the second is far more problematic.
Paragraph 2: What makes the second premise problematic is the use of the Puritan colonies as a
basis for comparison. Quite properly, Davis decries the excessive in uence ascribed by historians
to the Puritans in the formation of American culture. Yet Davis inadvertently adds weight to such
ascriptions by using the Puritans as the standard against which to assess the achievements and
contributions of Southern colonials. Throughout, Davis focuses on the important, and undeniable,
di erences between the Southern and Northern colonies in motives for and patterns of early
settlement, in attitudes toward nature and Native Americans, and in the degree of receptivity to
metropolitan cultural in uences.
Paragraph 3: However, recent scholarship has strongly suggested that those aspects of early New
England culture that seem to have been most distinctly Puritan, such as the strong religious
orientation and the communal impulse, were not even typical of New England as a whole, but
were largely con ned to the two colonies of America. Thus, what in contrast to the Puritan
(Northern) colonies appears to Davis to be peculiarly Southern — acquisitiveness, a strong interest
in politics and the law, and a tendency to cultivate metropolitan cultural models — was not only
more typically English than the cultural patterns exhibited by Puritan Massachusetts and
Connecticut, but also almost certainly characteristic of most other early modern British colonies
for more join
from Barbados north to Rhode Island and New Hampshire. Within the larger framework of
American colonial life, then, not the Southern but the Northern colonies appear to have been
https://t.me/currentAffairscLuB
distinctive, and even they seem to have been rapidly assimilating to the dominant cultural
patterns by the last Colonial period.

Which of the following is OPPOSITE in meaning of the word ‘ Peculiarly’ as used in the passage?
(1.5 Mark)
A. particular

https://store.adda247.com/#!/myTestAnalysis/printsolution/mappingId=25329/packageId=359/lang=ENGLISH 79/121
12/31/2017 Adda247 Store | Adda247 Store
B. demand
C. residual
D. common
E. penury
Your Answer: Not Attempted

Correct Answer: D. common <br>

Solution
Peculiarly- more than usually; especially, in an unusual way; oddly, used to emphasize restriction
to an individual or group.
Purchased by roshnipanwar025@gmail.com
Q91.
Read the following passage carefully and answer the questions given below it.

One likes to think that one's attitudes, beliefs, and related behaviour form a consistent pattern.
Incongruity that is detected results in a sense of imbalance or dissonance, which the person then
seeks to correct. The motivating e ects of the need to correct incongruity, imbalance, or
dissonance has been the occasion for several theories. We may select for consideration the theory
proposed by Festinger which treats cognitive dissonance and its reduction. The kind of
disagreement or disharmony with, which Festinger has been chie y concerned is that which
occurs after a decision has been made, after one is committed to a course of action; under such
circumstances, there is often some lack of harmony between what one does and what one
believes, and there is pressure to change either one's behaviour or one's beliefs. For example, if a
regular smoker reads about the relationship between smoking and lung cancer, the habitual
action and the new information are dissonant. If the decision is made to continue smoking, the
dissonance will be reduced by disbelieving the information about the relationship between
smoking and lung cancer; if the decision is made to give up smoking, the information on the
linkage between smoking and lung cancer will be accepted. The fact that this information also
a ected the decision is not important here. As Festinger and others have shown, the weighing of
alternatives is more realistic prior to the decision; after the decision, the pressure is great to bring
belief and action into balance. for more join
The theory goes on to make some non-obvious predictions; for example, in some cases, failure of
https://t.me/currentAffairscLuB
expectations instead of destroying belief may strengthen it. This was illustrated by the study of a
group of people who expected to be saved from a prophesied disastrous ood by the intervention
of a heavenly being. The theory predicted that when the long-awaited day arrived and the
prophecy failed (no ood), those who had the social support of the other believers would indeed
proselyte for their beliefs with new enthusiasm; while those who had to face the crisis alone would
have their faith weakened. These predicted results did indeed occur, the rationalization for the
group of disappointed believers who faced failure together being that God had postponed his
https://store.adda247.com/#!/myTestAnalysis/printsolution/mappingId=25329/packageId=359/lang=ENGLISH 80/121
12/31/2017 Adda247 Store | Adda247 Store
vengeance because of their faith.
The tendency to be consistent is but one aspect of how self-perception in uences motivation.
Earlier illustrations of human motivation might also be reinterpreted in these terms. For example,
the success motivation and the avoidance of failure are also concerned with how a person sees
himself. R.W. White, for example, reinterprets many motives concerned with curiosity, and desire
for knowledge and for achievement as though they are all concerned with one's sense of
competence as a person who is e ective in relation to the environment. In another sense, the
person likes to develop his potentials to the full, to be as complete a person as he can. For such a
pervasive type of motive, the expression self-actualization was coined, originally by Carl Jung, one
of Freud's followers who later developed a system of his own. By self-actualization, he meant the
development of full individuality, with all parts somehow in harmony. The term and closely related
ones (productive orientation, creative becoming, etc.) have been used by man psychologists who
criticize contemporary motivational theory as being too narrow concerned with short episodes of
choice and behaviour rather than with the more profound and pervasive aspects of individual
hopes and aspirations.

Which of the following situations is most likely to give rise to cognitive dissonance? (1 Mark)
A. Cricket fans watching their team lose
B. An antique collector being told by an expert that the vase he has paid Rs. 3,000 for is worth
Rs.100.
C. Student failing an exam
D. Man cutting himself shaving
E. All of the above
Your Answer: Not Attempted

Correct Answer: B. An antique collector being told by an expert that the vase he has paid Rs. 3,000
for is worth Rs.100. <br>

Solution
All the other options are a situation which a person has witnessed and has a proof to its occurence
whereas in option (b) a person has to believe something he hasn't witnessed, hence, there is an
ample scope for ambiguity. for more join
Purchased by roshnipanwar025@gmail.com
Q92. https://t.me/currentAffairscLuB
Read the following passage carefully and answer the questions given below it.

One likes to think that one's attitudes, beliefs, and related behaviour form a consistent pattern.
Incongruity that is detected results in a sense of imbalance or dissonance, which the person then
seeks to correct. The motivating e ects of the need to correct incongruity, imbalance, or
dissonance has been the occasion for several theories. We may select for consideration the theory
https://store.adda247.com/#!/myTestAnalysis/printsolution/mappingId=25329/packageId=359/lang=ENGLISH 81/121
12/31/2017 Adda247 Store | Adda247 Store
proposed by Festinger which treats cognitive dissonance and its reduction. The kind of
disagreement or disharmony with, which Festinger has been chie y concerned is that which
occurs after a decision has been made, after one is committed to a course of action; under such
circumstances, there is often some lack of harmony between what one does and what one
believes, and there is pressure to change either one's behaviour or one's beliefs. For example, if a
regular smoker reads about the relationship between smoking and lung cancer, the habitual
action and the new information are dissonant. If the decision is made to continue smoking, the
dissonance will be reduced by disbelieving the information about the relationship between
smoking and lung cancer; if the decision is made to give up smoking, the information on the
linkage between smoking and lung cancer will be accepted. The fact that this information also
a ected the decision is not important here. As Festinger and others have shown, the weighing of
alternatives is more realistic prior to the decision; after the decision, the pressure is great to bring
belief and action into balance.
The theory goes on to make some non-obvious predictions; for example, in some cases, failure of
expectations instead of destroying belief may strengthen it. This was illustrated by the study of a
group of people who expected to be saved from a prophesied disastrous ood by the intervention
of a heavenly being. The theory predicted that when the long-awaited day arrived and the
prophecy failed (no ood), those who had the social support of the other believers would indeed
proselyte for their beliefs with new enthusiasm; while those who had to face the crisis alone would
have their faith weakened. These predicted results did indeed occur, the rationalization for the
group of disappointed believers who faced failure together being that God had postponed his
vengeance because of their faith.
The tendency to be consistent is but one aspect of how self-perception in uences motivation.
Earlier illustrations of human motivation might also be reinterpreted in these terms. For example,
the success motivation and the avoidance of failure are also concerned with how a person sees
himself. R.W. White, for example, reinterprets many motives concerned with curiosity, and desire
for knowledge and for achievement as though they are all concerned with one's sense of
competence as a person who is e ective in relation to the environment. In another sense, the
person likes to develop his potentials to the full, to be as complete a person as he can. For such a
pervasive type of motive, the expression self-actualization was coined, originally by Carl Jung, one
of Freud's followers who later developed a system of his own. By self-actualization, he meant the
for more join
development of full individuality, with all parts somehow in harmony. The term and closely related
ones (productive orientation, creative becoming, etc.) have been used by man psychologists who
https://t.me/currentAffairscLuB
criticize contemporary motivational theory as being too narrow concerned with short episodes of
choice and behaviour rather than with the more profound and pervasive aspects of individual
hopes and aspirations.

With which of the following statements would Jung be most likely to agree? (1 Mark)
A. Parents should not allow their children to smoke

https://store.adda247.com/#!/myTestAnalysis/printsolution/mappingId=25329/packageId=359/lang=ENGLISH 82/121
12/31/2017 Adda247 Store | Adda247 Store
B. Parents should force their children to learn music
C. Parents should give their children complete freedom
D. Parents should encourage their children to pursue any interests the children might have
E. None of these
Your Answer: Not Attempted

Correct Answer: D. Parents should encourage their children to pursue any interests the children
might have <br>

Solution
Refer to the last paragraph-2nd last line, ‘By self-actualisation…. harmony.’ Hence, option (d) is the
correct choice.
Purchased by roshnipanwar025@gmail.com
Q93.
Read the following passage carefully and answer the questions given below it.

One likes to think that one's attitudes, beliefs, and related behaviour form a consistent pattern.
Incongruity that is detected results in a sense of imbalance or dissonance, which the person then
seeks to correct. The motivating e ects of the need to correct incongruity, imbalance, or
dissonance has been the occasion for several theories. We may select for consideration the theory
proposed by Festinger which treats cognitive dissonance and its reduction. The kind of
disagreement or disharmony with, which Festinger has been chie y concerned is that which
occurs after a decision has been made, after one is committed to a course of action; under such
circumstances, there is often some lack of harmony between what one does and what one
believes, and there is pressure to change either one's behaviour or one's beliefs. For example, if a
regular smoker reads about the relationship between smoking and lung cancer, the habitual
action and the new information are dissonant. If the decision is made to continue smoking, the
dissonance will be reduced by disbelieving the information about the relationship between
smoking and lung cancer; if the decision is made to give up smoking, the information on the
linkage between smoking and lung cancer will be accepted. The fact that this information also
a ected the decision is not important here. As Festinger and others have shown, the weighing of
alternatives is more realistic prior to the decision; after the decision, the pressure is great to bring
for more join
belief and action into balance.
https://t.me/currentAffairscLuB
The theory goes on to make some non-obvious predictions; for example, in some cases, failure of
expectations instead of destroying belief may strengthen it. This was illustrated by the study of a
group of people who expected to be saved from a prophesied disastrous ood by the intervention
of a heavenly being. The theory predicted that when the long-awaited day arrived and the
prophecy failed (no ood), those who had the social support of the other believers would indeed
proselyte for their beliefs with new enthusiasm; while those who had to face the crisis alone would
have their faith weakened. These predicted results did indeed occur, the rationalization for the
https://store.adda247.com/#!/myTestAnalysis/printsolution/mappingId=25329/packageId=359/lang=ENGLISH 83/121
12/31/2017 Adda247 Store | Adda247 Store
group of disappointed believers who faced failure together being that God had postponed his
vengeance because of their faith.
The tendency to be consistent is but one aspect of how self-perception in uences motivation.
Earlier illustrations of human motivation might also be reinterpreted in these terms. For example,
the success motivation and the avoidance of failure are also concerned with how a person sees
himself. R.W. White, for example, reinterprets many motives concerned with curiosity, and desire
for knowledge and for achievement as though they are all concerned with one's sense of
competence as a person who is e ective in relation to the environment. In another sense, the
person likes to develop his potentials to the full, to be as complete a person as he can. For such a
pervasive type of motive, the expression self-actualization was coined, originally by Carl Jung, one
of Freud's followers who later developed a system of his own. By self-actualization, he meant the
development of full individuality, with all parts somehow in harmony. The term and closely related
ones (productive orientation, creative becoming, etc.) have been used by man psychologists who
criticize contemporary motivational theory as being too narrow concerned with short episodes of
choice and behaviour rather than with the more profound and pervasive aspects of individual
hopes and aspirations.

The passage probably comes from (1 Mark)


A. the introduction to a book
B. the rst chapter of a book
C. middle of a text book
D. an article in a news weekly
E. None of these
Your Answer: Not Attempted

Correct Answer: D. an article in a news weekly <br>

Solution
The language used by the writer is simple and lucid enough for an average reader to understand
the text, much like that of a magazine.In addition the other three options donot seem to be
relevant in the given context. Hence, option (d) is the correct choice.
for more join
Purchased by roshnipanwar025@gmail.com
Q94.
https://t.me/currentAffairscLuB
Read the following passage carefully and answer the questions given below it.

One likes to think that one's attitudes, beliefs, and related behaviour form a consistent pattern.
Incongruity that is detected results in a sense of imbalance or dissonance, which the person then
seeks to correct. The motivating e ects of the need to correct incongruity, imbalance, or
dissonance has been the occasion for several theories. We may select for consideration the theory
proposed by Festinger which treats cognitive dissonance and its reduction. The kind of
https://store.adda247.com/#!/myTestAnalysis/printsolution/mappingId=25329/packageId=359/lang=ENGLISH 84/121
12/31/2017 Adda247 Store | Adda247 Store
disagreement or disharmony with, which Festinger has been chie y concerned is that which
occurs after a decision has been made, after one is committed to a course of action; under such
circumstances, there is often some lack of harmony between what one does and what one
believes, and there is pressure to change either one's behaviour or one's beliefs. For example, if a
regular smoker reads about the relationship between smoking and lung cancer, the habitual
action and the new information are dissonant. If the decision is made to continue smoking, the
dissonance will be reduced by disbelieving the information about the relationship between
smoking and lung cancer; if the decision is made to give up smoking, the information on the
linkage between smoking and lung cancer will be accepted. The fact that this information also
a ected the decision is not important here. As Festinger and others have shown, the weighing of
alternatives is more realistic prior to the decision; after the decision, the pressure is great to bring
belief and action into balance.
The theory goes on to make some non-obvious predictions; for example, in some cases, failure of
expectations instead of destroying belief may strengthen it. This was illustrated by the study of a
group of people who expected to be saved from a prophesied disastrous ood by the intervention
of a heavenly being. The theory predicted that when the long-awaited day arrived and the
prophecy failed (no ood), those who had the social support of the other believers would indeed
proselyte for their beliefs with new enthusiasm; while those who had to face the crisis alone would
have their faith weakened. These predicted results did indeed occur, the rationalization for the
group of disappointed believers who faced failure together being that God had postponed his
vengeance because of their faith.
The tendency to be consistent is but one aspect of how self-perception in uences motivation.
Earlier illustrations of human motivation might also be reinterpreted in these terms. For example,
the success motivation and the avoidance of failure are also concerned with how a person sees
himself. R.W. White, for example, reinterprets many motives concerned with curiosity, and desire
for knowledge and for achievement as though they are all concerned with one's sense of
competence as a person who is e ective in relation to the environment. In another sense, the
person likes to develop his potentials to the full, to be as complete a person as he can. For such a
pervasive type of motive, the expression self-actualization was coined, originally by Carl Jung, one
of Freud's followers who later developed a system of his own. By self-actualization, he meant the
development of full individuality, with all parts somehow in harmony. The term and closely related
for more join
ones (productive orientation, creative becoming, etc.) have been used by man psychologists who
criticize contemporary motivational theory as being too narrow concerned with short episodes of
https://t.me/currentAffairscLuB
choice and behaviour rather than with the more profound and pervasive aspects of individual
hopes and aspirations.

The best title for this passage would be (1 Mark)


A. Self-actualization
B. Self-reference in Human Motivation

https://store.adda247.com/#!/myTestAnalysis/printsolution/mappingId=25329/packageId=359/lang=ENGLISH 85/121
12/31/2017 Adda247 Store | Adda247 Store
C. The Reduction of Cognitive Dissonance
D. Cognitive Dissonance and the Self
E. None of these
Your Answer: Not Attempted

Correct Answer: D. Cognitive Dissonance and the Self <br>

Solution
The main idea of the passage revolves around cognitive dissonance and the self. Hence option (d)
is the correct choice.
Purchased by roshnipanwar025@gmail.com
Q95.
Read the following passage carefully and answer the questions given below it.

One likes to think that one's attitudes, beliefs, and related behaviour form a consistent pattern.
Incongruity that is detected results in a sense of imbalance or dissonance, which the person then
seeks to correct. The motivating e ects of the need to correct incongruity, imbalance, or
dissonance has been the occasion for several theories. We may select for consideration the theory
proposed by Festinger which treats cognitive dissonance and its reduction. The kind of
disagreement or disharmony with, which Festinger has been chie y concerned is that which
occurs after a decision has been made, after one is committed to a course of action; under such
circumstances, there is often some lack of harmony between what one does and what one
believes, and there is pressure to change either one's behaviour or one's beliefs. For example, if a
regular smoker reads about the relationship between smoking and lung cancer, the habitual
action and the new information are dissonant. If the decision is made to continue smoking, the
dissonance will be reduced by disbelieving the information about the relationship between
smoking and lung cancer; if the decision is made to give up smoking, the information on the
linkage between smoking and lung cancer will be accepted. The fact that this information also
a ected the decision is not important here. As Festinger and others have shown, the weighing of
alternatives is more realistic prior to the decision; after the decision, the pressure is great to bring
belief and action into balance.
The theory goes on to make some non-obvious predictions; for example, in some cases, failure of
for more join
expectations instead of destroying belief may strengthen it. This was illustrated by the study of a
https://t.me/currentAffairscLuB
group of people who expected to be saved from a prophesied disastrous ood by the intervention
of a heavenly being. The theory predicted that when the long-awaited day arrived and the
prophecy failed (no ood), those who had the social support of the other believers would indeed
proselyte for their beliefs with new enthusiasm; while those who had to face the crisis alone would
have their faith weakened. These predicted results did indeed occur, the rationalization for the
group of disappointed believers who faced failure together being that God had postponed his
vengeance because of their faith.
https://store.adda247.com/#!/myTestAnalysis/printsolution/mappingId=25329/packageId=359/lang=ENGLISH 86/121
12/31/2017 Adda247 Store | Adda247 Store
The tendency to be consistent is but one aspect of how self-perception in uences motivation.
Earlier illustrations of human motivation might also be reinterpreted in these terms. For example,
the success motivation and the avoidance of failure are also concerned with how a person sees
himself. R.W. White, for example, reinterprets many motives concerned with curiosity, and desire
for knowledge and for achievement as though they are all concerned with one's sense of
competence as a person who is e ective in relation to the environment. In another sense, the
person likes to develop his potentials to the full, to be as complete a person as he can. For such a
pervasive type of motive, the expression self-actualization was coined, originally by Carl Jung, one
of Freud's followers who later developed a system of his own. By self-actualization, he meant the
development of full individuality, with all parts somehow in harmony. The term and closely related
ones (productive orientation, creative becoming, etc.) have been used by man psychologists who
criticize contemporary motivational theory as being too narrow concerned with short episodes of
choice and behaviour rather than with the more profound and pervasive aspects of individual
hopes and aspirations.

Which of the following statements would the author disagree with? (1 Mark)
A. The tendency to be consistent is the only aspect of how self-perception in uences motivation
B. The motivating e ects of the need to correct incongruity have been the occasion for several
theories
C. By self-actualization, Carl Jung meant the development of full individuality, with all parts
somehow in harmony
D. Both (A) and (B)
E. None of the above
Your Answer: Not Attempted

Correct Answer: A. The tendency to be consistent is the only aspect of how self-perception
in uences motivation <br>

Solution
As per the statement "The tendency to be consistent is but one aspect of how self-perception
in uences motivation." it is clear that option (a) is correct.
for more join
Purchased by roshnipanwar025@gmail.com
Q96.
https://t.me/currentAffairscLuB
Select the phrase/connector (STARTERS) from the given three options which can be used to form a
single sentence from the two sentences given below, implying the same meaning as expressed in
the statement sentences.

(1)MC Mary Kom has been described as the Muhammad Ali of women’s boxing.
(2)It is a tri e unfair because she has her own claim to fame - the indefatigable MC Mary Kom, the
woman from Manipur who attained global glory.
https://store.adda247.com/#!/myTestAnalysis/printsolution/mappingId=25329/packageId=359/lang=ENGLISH 87/121
12/31/2017 Adda247 Store | Adda247 Store

(I)Though MC Mary Kom has been…


(II)Despite being the indefatigable…
(III)Considering it a tri e unfair… (1 Mark)
A. Only (I) is correct
B. Only (III) is correct
C. Both (I) and (II) are correct
D. Both (I) and (III) are correct
E. All are correct
Your Answer: Not Attempted

Correct Answer: A. Only (I) is correct <br>

Solution
Only the rst starter can be used to frame a logical and comprehensive sentence connecting both
the sentences given as the question. The other two starters are incorrect owing to their varying
meanings.
(I)Though MC Mary Kom has been described as the Muhammad Ali of women’s boxing, it is a tri e
unfair because she has her own claim to fame - the indefatigable MC Mary Kom, the woman from
Manipur who attained global glory.
Purchased by roshnipanwar025@gmail.com
Q97.
Select the phrase/connector (STARTERS) from the given three options which can be used to form a
single sentence from the two sentences given below, implying the same meaning as expressed in
the statement sentences.

(1) It would be a lengthy a air to list the successes of MC Mary Kom.


(2)What matters is how she adapted herself to varying demands, including “gaining weight”
because a few competitions did not cater to her natural weight-category.

(I)To list the successes of MC Mary Kom…


(II)While it would be a lengthy a air… for more join
(III)Adapting herself to varying demands… (1 Mark)
A. Only (I) is correct https://t.me/currentAffairscLuB
B. Only (II) is correct
C. Both (I) and (II) are correct
D. Both (I) and (III) are correct
E. All are correct
Your Answer: Not Attempted

https://store.adda247.com/#!/myTestAnalysis/printsolution/mappingId=25329/packageId=359/lang=ENGLISH 88/121
12/31/2017 Adda247 Store | Adda247 Store
Correct Answer: C. Both (I) and (II) are correct <br>

Solution
Both the starters (I) and (II) can be used to connect the given set of two sentences to form a
meaningful statement. However, the third starter is incorrect as it would alter the meaning of the
actual sentence.
(I)To list the successes of MC Mary Kom would be a lengthy a air, what matters is how she
adapted herself to varying demands, including “gaining weight” because a few competitions did
not cater to her natural weight-category.
(II)While it would be a lengthy a air to list the successes of MC Mary Kom, what matters is how she
adapted herself to varying demands, including “gaining weight” because a few competitions did
not cater to her natural weight-category.
Purchased by roshnipanwar025@gmail.com
Q98.
Select the phrase/connector (STARTERS) from the given three options which can be used to form a
single sentence from the two sentences given below, implying the same meaning as expressed in
the statement sentences.

(1)The Congress was an umbrella organization with many ideologies coexisting together.
(2)However, it lacked a single focus.

(I)Being an umbrella organization…


(II)In view of the fact that the Congress…
(III)As the Congress was an umbrella… (1 Mark)
A. Only (I) is correct
B. Only (II) is correct
C. Both (I) and (II) are correct
D. Both (I) and (III) are correct
E. All are correct
Your Answer: Not Attempted

Correct Answer: E. All are correct <br> for more join


Solution https://t.me/currentAffairscLuB
All the three starters are correct enough to form three logical sentences without altering the
meaning of the actual sentence.
(I)Being an umbrella organization with many ideologies coexisting together, the Congress lacked a
single focus.
(II)In view of the fact that the Congress was an umbrella organization with many ideologies

https://store.adda247.com/#!/myTestAnalysis/printsolution/mappingId=25329/packageId=359/lang=ENGLISH 89/121
12/31/2017 Adda247 Store | Adda247 Store
coexisting together, it lacked a single focus.
(III) As the Congress was an umbrella organization with many ideologies coexisting together, it
lacked a single focus.
Purchased by roshnipanwar025@gmail.com
Q99.
Select the phrase/connector (STARTERS) from the given three options which can be used to form a
single sentence from the two sentences given below, implying the same meaning as expressed in
the statement sentences.

(1)Deen Dayal identi es honesty to be a principle.


(2)The task is to make it a policy for which the beginning has to be made by reforming an
overdeveloped state that is free of corruption, with a high level of human development indices,
and adhering to a “world minimum” that Sakharov spoke of.

(I)With the task to make a policy for…


(II)While Deen Dayal identi es honesty…
(III)Owing to the fact that reforming an… (1 Mark)
A. Only (I) is correct
B. Only (II) is correct
C. Both (I) and (II) are correct
D. Both (I) and (III) are correct
E. All are correct
Your Answer: Not Attempted

Correct Answer: B. Only (II) is correct <br>

Solution
Only the second starter can be used to frame a logical and comprehensive sentence connecting
both the sentences given as the question. The other two starters are incorrect owing to their
varying meanings.
(II)While Deen Dayal identi es honesty to be a principle, the task is to make it a policy for which
for more join
the beginning has to be made by reforming an overdeveloped state that is free of corruption, with
https://t.me/currentAffairscLuB
a high level of human development indices, and adhering to a “world minimum” that Sakharov
spoke of.
Purchased by roshnipanwar025@gmail.com
Q100.
Select the phrase/connector (STARTERS) from the given three options which can be used to form a
single sentence from the two sentences given below, implying the same meaning as expressed in

https://store.adda247.com/#!/myTestAnalysis/printsolution/mappingId=25329/packageId=359/lang=ENGLISH 90/121
12/31/2017 Adda247 Store | Adda247 Store
the statement sentences.

(1)Deen Dayal emphasised the relationship of political ideologies to the factors of time and place.
(2)He discounted the universal validity of Western ideologies.

(I)The factors of time and place…


(II)For all that emphasis on…
(III)Discounting the universal validity… (1 Mark)
A. Only (I) is correct
B. Only (III) is correct
C. Both (I) and (II) are correct
D. Both (I) and (III) are correct
E. All are correct
Your Answer: Not Attempted

Correct Answer: B. Only (III) is correct <br>

Solution
Only the third starter can be used to frame a logical and comprehensive sentence connecting both
the sentences given as the question. The other two starters are incorrect owing to their varying
meanings.
(III)Discounting the universal validity of Western ideologies, Deen Dayal emphasised the
relationship of political ideologies to the factors of time and place.
Purchased by roshnipanwar025@gmail.com
Q101.
Each of the following questions has a paragraph from which one sentence has been deleted. From
the given options, choose the one that completes the paragraph in the most appropriate way.

The Supreme Court of India is facing its worst crisis of credibility since the Emergency. With an
occasional exception, the quality of the court’s reasoning, the inconstancy of its judgment, the
abdication of its constitutional role in some cases, and its overreach in others, are already denting
its authority. _________________________________________. It will also create the conditions under which
for more join
it will be easier to legitimise diluting judicial independence. (1 Mark)
A. But Justice Chelameswar’s order setting https://t.me/currentAffairscLuB
up a ve-judge bench also made the judiciary
vulnerable.
B. But consider the di erent ways in which the judiciary has now rendered itself vulnerable.
C. But the institutional crisis that the Supreme Court has now created for itself will puncture more
holes in the authority that it so valiantly tried to exert.
D. But a lot of care needs to be exercised so that the anti-corruption measures taken do not
undermine the independence of the judiciary.
https://store.adda247.com/#!/myTestAnalysis/printsolution/mappingId=25329/packageId=359/lang=ENGLISH 91/121
12/31/2017 Adda247 Store | Adda247 Store
E. But a clamour for reforms that undermine independence in the name of accountability will be a
natural consequence of the current chain of events.
Your Answer: Not Attempted

Correct Answer: C. But the institutional crisis that the Supreme Court has now created for itself will
puncture more holes in the authority that it so valiantly tried to exert. <br>

Solution
Read the paragraph carefully, it is about the crisis of credibility that the Supreme Court of India is
facing since the Emergency. Among the given options, there is only sentence (c) that ts perfectly
into the blank space. The sentence adds meaning to the paragraph and at the same time it
correctly follows the rst and the second sentences of the paragraph. Other options are irrelevant
in the context of the paragraph. Hence (c) is the correct choice.
Purchased by roshnipanwar025@gmail.com
Q102.
Each of the following questions has a paragraph from which one sentence has been deleted. From
the given options, choose the one that completes the paragraph in the most appropriate way.

Like all modernists, Nehru too was a child of science and its grand promise of the light of reason,
its explanatory capacity and its ability to expand the zones of certainty in an otherwise uncertain
world. With “scienti c temper” — “a process of thinking, a method of acting and associating with
our fellow men” — he sought to ght the “dead weight of past” and give India the “garb of
modernity”. In Nehru’s romance with science, I guess, there was idealism.
___________________________________________. In fact, Nehru’s ideal science person was more like the
courageous/idealist doctor Satyajit Ray projected in his lm Ganashatru (Enemy of people) — a
doctor who could dare to ght the temple authority by showing that the outbreak of jaundice in
the locality is because of the contaminated water in the temple that the devotees are taking
mindlessly. (1 Mark)
A. Even though he loved industrial development and technological miracles like big dams, his
“science” was still not like technocratic rationality, the way global capitalism has reduced it into a
principle of corporate trade and reckless domination of people and nature.
B. A “secular” Nehru, I believe, was more sensitive to the neness of spiritual wonder than what
for more join
these days the zealots and state-sponsored celebrity babas are doing.
C. His scholarship, his passion, and his https://t.me/currentAffairscLuB
wonder I miss in a political culture that taps the simulation
of 24X7 television channels and networking of social media, and projects the dramaturgical
performance of narcissistic leaders without philosophic depth and real substance.
D. The “elitism” of Nehru notwithstanding, his sensitivity, I would say, opens our eyes to seeing
beyond loud modernity, loud nationalism and popular culture.
E. And despite my unease with scientism and soulless atheism, I continue to be fascinated by
secular Nehru’s openness and philosophic perplexity.
https://store.adda247.com/#!/myTestAnalysis/printsolution/mappingId=25329/packageId=359/lang=ENGLISH 92/121
12/31/2017 Adda247 Store | Adda247 Store
Your Answer: Not Attempted

Correct Answer: A. Even though he loved industrial development and technological miracles like
big dams, his “science” was still not like technocratic rationality, the way global capitalism has
reduced it into a principle of corporate trade and reckless domination of people and nature. <br>

Solution
The paragraph is about Nehru’s way of seeing the world, his ideas of science and idealism
associated with it. The last sentence of the paragraph gives an apt evidence that the sentence
preceding it would be option (a). Moreover, among the given options, only statement (a) ts
perfectly into the paragraph as it continues on the same theme as the paragraph. Other options
are irrelevant in the context of the paragraph. Hence (a) is the correct choice.
Purchased by roshnipanwar025@gmail.com
Q103.
Each of the following questions has a paragraph from which one sentence has been deleted. From
the given options, choose the one that completes the paragraph in the most appropriate way.

Under the second phase of Regional Connectivity Scheme (RCS), which aims to provide air
connectivity to the hinterland to provide an impetus to the economic growth of regional centres,
including unconnected towns and cities, the government decided to keep the focus on ‘priority
areas’ including the North-East, Jammu & Kashmir and other hilly regions of the country. In the list
of routes awarded in the rst phase of the scheme, announced in March, only six airports in the
North-East — Shillong, Dimapur, Imphal, Silchar, Aizawl and Agartala — were connected.
_____________________________________ (1 Mark)
A. The other airports are under the aegis of either the AAI, or respective state governments.
B. The window for air operators to bid for routes under the second phase of RCS started on
August 24, four days before Indian and Chinese troops disengaged at Doklam to end a border
stando that lasted more than two months.
C. The airports and helipads in Arunachal Pradesh fall under the category of ‘unserved’ airports,
meaning they have no civilian ights landing there.
D. The Tezu airport in Arunachal Pradesh has also been upgraded by the AAI with a renovated
terminal building at a cost of Rs 96.50 crore.
for more join
E. The rst round did not involve routes to Jammu & Kashmir.
Your Answer: Not Attempted https://t.me/currentAffairscLuB
Correct Answer: E. The rst round did not involve routes to Jammu &amp; Kashmir. <br>

Solution
The given paragraph is about the rst and the second phases of Regional Connectivity Scheme
(RCS), which aims to provide air connectivity to the hinterland to provide an impetus to the
https://store.adda247.com/#!/myTestAnalysis/printsolution/mappingId=25329/packageId=359/lang=ENGLISH 93/121
12/31/2017 Adda247 Store | Adda247 Store
economic growth of regional centres, including unconnected towns and cities. Read the last two
sentences of the paragraph carefully, it gives a direct clue that option (e) provides the most
appropriate conclusion to the paragraph as it maintains the ow of the theme. Moreover, it carries
a logical sequence to other sentences given in the paragraph. Other options are not relevant
enough to be the part of this particular paragraph. Hence (e) is the correct choice.
Purchased by roshnipanwar025@gmail.com
Q104.
Each of the following questions has a paragraph from which one sentence has been deleted. From
the given options, choose the one that completes the paragraph in the most appropriate way.

The quad’s signi cance, however, lies in the future. That President Donald Trump referred to the
region as “Indo-Paci c” is a re ection of the emerging expectations from India amidst the
increasing unsustainability of the present Asian order. The rise of China, the expansion of its
military capabilities and Beijing’s assertiveness on territorial disputes has ended the prolonged
tranquility in the region. China’s pressure on its neighbours is weakening the unity of the ASEAN
and limiting the options of most countries — big and small — in the region. The US, which has long
dominated Asian order, as well as many of China’s neighbours, want Delhi to contribute a lot more
to regional peace and security. ___________________________________________________ (1 Mark)
A. India is the laggard in the negotiations on the ASEAN framework for Regional Comprehensive
Economic Partnership (RCEP).
B. On his part, the PM has signalled India’s intent to promote a rules-based architecture in Asia.
C. If Trump has shocked the region by pulling America out of the Trans-Paci c Partnership (TPP)
talks and China is claiming the leadership of Asian regionalism, Delhi is paralysed.
D. The conversation between the four capitals on reviving the quad has been taking place for
some time.
E. That the meeting took place in Manila on the margins of the annual ASEAN-led meetings was
just a matter of diplomatic convenience.
Your Answer: Not Attempted

Correct Answer: B. On his part, the PM has signalled India’s intent to promote a rules-based
architecture in Asia. <br>
for more join
Solution
https://t.me/currentAffairscLuB
The paragraph is about the quadrilateral security dialogue between senior o cials of India, United
States, Japan and Australia. Among the given options, there is only sentence (b) which nds some
connection with the paragraph and at the same time it concludes the paragraph in the best
manner, adding meaning to it. Other options are not feasible enough to make the paragraph
complete and conclusive. Hence (b) is the correct choice.
Purchased by roshnipanwar025@gmail.com
Q105.
https://store.adda247.com/#!/myTestAnalysis/printsolution/mappingId=25329/packageId=359/lang=ENGLISH 94/121
12/31/2017 Adda247 Store | Adda247 Store
Each of the following questions has a paragraph from which one sentence has been deleted. From
the given options, choose the one that completes the paragraph in the most appropriate way.

The report presents a two-pronged challenge for policymakers. The large-scale variation in the
disease patterns across the country means that one health policy and uniform health-related
schemes are unlikely to work in all the states. ______________________________________. One nugget of
information in the report throws light on the enormity of this problem: Kerala had the lowest
disease burden due to malnutrition in India, but even that was 2.7 times higher per person than in
China. That said, policymakers in the country should see the report as an opportunity. After all, it
addresses their longstanding grievance about the paucity of data on India’s disease burden. (1
Mark)
A. Since then, several studies have tried to understand the contours of this shift in the country’s
disease burden.
B. But worryingly, the study upturns the widespread perception that states performing well on
economic yardsticks are also doing well on health indicators.
C. Major national surveys, such as the National Family Health Survey and the Annual Health
Survey, have provided valuable data on key health indicators, and several states have generated
data on non-communicable diseases such as diabetes and heart ailments.
D. But at the same time, the persistence of communicable diseases and malnutrition means that
e orts to tackle these maladies have to be scaled up.
E. However, a comprehensive assessment of every major disease across all states of the country,
providing estimates over an extended period, has eluded policymakers.
Your Answer: Not Attempted

Correct Answer: D. But at the same time, the persistence of communicable diseases and
malnutrition means that e orts to tackle these maladies have to be scaled up. <br>

Solution
The paragraph is all about the new report that addresses need for data on the country’s disease
burden and identi es challenges for policymakers. Among the given options, only sentence (d) ts
perfectly into the blank space as it follows the sentence prior to the space as well as the one
for more join
following it. It brings the continuity into the meaning of the paragraph. Other options are not
relevant enough to bring about a similar meaning to the paragraph. Hence (d) is the correct
https://t.me/currentAffairscLuB
choice.
Purchased by roshnipanwar025@gmail.com
Q106.
Five statements are given below, labelled a, b, c, d and e. Among these, four statements are in
logical order and form a coherent paragraph. From the given options, choose the option that does
not t into the theme of the paragraph. (1 Mark)

https://store.adda247.com/#!/myTestAnalysis/printsolution/mappingId=25329/packageId=359/lang=ENGLISH 95/121
12/31/2017 Adda247 Store | Adda247 Store
A. To make matters worse, the imposition of an ill-designed Goods and Services Tax (GST), whose
impact fell disproportionately on small manufacturing units and traders, has compounded the
crisis.
B. A year later, the e ects of the monumental blunder, undertaken in the name of ghting the
“black money” menace, still linger.
C. Livelihoods in huge swathes of the economy that were uprooted in the wake of demonetisation
have still not recovered.
D. It was clear that the troika of Modi, the Finance Ministry and the RBI was responsible for the
crisis.
E. November 8, 2017, marks the rst anniversary of demonetisation, an exercise that shocked,
numbed and grievously wounded livelihoods on a scale unprecedented since Independence.
Your Answer: Not Attempted

Correct Answer: D. It was clear that the troika of Modi, the Finance Ministry and the RBI was
responsible for the crisis. <br>

Solution
Sentences ebca form a coherent paragraph as it talks about the impact of demonetization and
then GST on the economy that created crises in livelihoods, small manufacturing units and traders
which are still to be recovered. Sentence (d) fails to connect with the other sentences as it tells
about the people who are responsible for this and mentions the word ‘clear’ as no other sentences
are pointing towards this conclusion. Hence option (d) is the correct choice.
Purchased by roshnipanwar025@gmail.com
Q107.
Five statements are given below, labelled a, b, c, d and e. Among these, four statements are in
logical order and form a coherent paragraph. From the given options, choose the option that does
not t into the theme of the paragraph. (1 Mark)
A. China will become a “moderately prosperous society” by 2021, the year that will mark 100 years
of the CPC’s formation.
B. From then onwards, China will be on the home stretch to become an advanced socialist
country, and the goal will be accomplished by 2050.
C. In his marathon speech that lasted over three hours, Xi said his grand mid-century vision would
for more join
be realised in three distinct phases.
https://t.me/currentAffairscLuB
D. China will become a leading developed nation, having accomplished the unprecedented goal of
becoming the world’s rst advanced socialist country, he said.
E. China’s new stage of development will be consolidated by 2035.
Your Answer: Not Attempted

Correct Answer: D. China will become a leading developed nation, having accomplished the
unprecedented goal of becoming the world’s rst advanced socialist country, he said. <br>
https://store.adda247.com/#!/myTestAnalysis/printsolution/mappingId=25329/packageId=359/lang=ENGLISH 96/121
12/31/2017 Adda247 Store | Adda247 Store

Solution
Sentences caeb a coherent paragraph whereas sentence (d) is not a part of the paragraph as it is
about accomplishing the goal and making China as a leading developed nation. You will notice that
the three phases which are being talked about in the sentence (c) are following a chronological
order and are talking about di erent timelines. So while (d) can be a part of the paragraph but
when it comes to coherence ‘caeb’ is the correct order.
Purchased by roshnipanwar025@gmail.com
Q108.
Five statements are given below, labelled a, b, c, d and e. Among these, four statements are in
logical order and form a coherent paragraph. From the given options, choose the option that does
not t into the theme of the paragraph. (1 Mark)
A. Cuban Foreign Minister Bruno Rodriguez Parrilla sharply criticised the U.S.’ move and accused
the U.S. of not “su ciently cooperating” with the Cuban government’s e orts to investigate the
“health attacks”.
B. Previously, he had only referred to the cases as “incidents”.
C. The Obama administration had relaxed restrictions on travel to Cuba.
D. In his recent statement on the subject, Tillerson said that there was an “attack” on his country’s
diplomats based in Havana.
E. The U.S. authorities have not provided access to those a ected or to the doctors examining
them.
Your Answer: Not Attempted

Correct Answer: C. The Obama administration had relaxed restrictions on travel to Cuba. <br>

Solution
Sentences aedb form a coherent paragraph talking about criticism made by Cuba against U.S.
move of not supporting them. Sentence (c) is about relaxing restriction during Obama’s
administration which is not making a connection with the other sentences . Hence option (c) is the
right choice.
Purchased by roshnipanwar025@gmail.com
Q109.
for more join
Five statements are given below, labelled a, b, c, d and e. Among these, four statements are in
https://t.me/currentAffairscLuB
logical order and form a coherent paragraph. From the given options, choose the option that does
not t into the theme of the paragraph. (1 Mark)
A. Illegal drugs—such as cocaine and heroin—continue to pose a challenge, but the real threat is
from prescription opioids such as fentanyls of one kind or another.
B. The declaration of an emergency means that state funds should go post-haste to help stem the
crisis.

https://store.adda247.com/#!/myTestAnalysis/printsolution/mappingId=25329/packageId=359/lang=ENGLISH 97/121
12/31/2017 Adda247 Store | Adda247 Store
C. In early November, President Donald Trump declared a national emergency against opioid
abuse.
D. It is not clear, however, if this will be enough.
E. “The opioid is a tremendous emergency,” he said in his characteristic style.
Your Answer: Not Attempted

Correct Answer: A. Illegal drugs—such as cocaine and heroin—continue to pose a challenge, but
the real threat is from prescription opioids such as fentanyls of one kind or another. <br>

Solution
Going through the sentences we nd that sentences cebd form a coherent paragraph and are
discussing about the emergency declared by Donald Trump against opioid abuse whereas
sentence (a) is about threat from the drugs making it unrelated to the other sentences. Hence
option (a) is the correct choice.
Purchased by roshnipanwar025@gmail.com
Q110.
Five statements are given below, labelled a, b, c, d and e. Among these, four statements are in
logical order and form a coherent paragraph. From the given options, choose the option that does
not t into the theme of the paragraph. (1 Mark)
A. There are issues with encroachment on the western face of the ranges (in Kanyakumari district)
where there are signi cant human population pressures.
B. Coconut plantations, irrigated with borewells, are sprouting in semi-arid areas.
C. Change has been a key aspect of the observations—the forest has recovered from being a
degraded plantation to now hosting a plethora of rare and endangered Western Ghats species.
D. The challenges in Naraikadu and the wider KMTR protected area in the future will come from a
multitude of sources.
E. The eastern edges of the hills are now facing development pressures as land is bought up for
investment with little thought for the ecological impact.
Your Answer: Not Attempted

Correct Answer: C. Change has been a key aspect of the observations—the forest has recovered
from being a degraded plantation to now hosting a plethora of rare and endangered Western
for more join
Ghats species. <br>
https://t.me/currentAffairscLuB
Solution
Sentences daeb form a coherent paragraph and discusses about the sources from which the
challenges in protected area will arise that are encroachment on western part, development
pressures on eastern edges of the hills and sprouting of coconut plantations whereas sentence (c)
talks about the recovery of forests making it unrelated to other sentences. Hence option (c) is the
correct choice.
https://store.adda247.com/#!/myTestAnalysis/printsolution/mappingId=25329/packageId=359/lang=ENGLISH 98/121
12/31/2017 Adda247 Store | Adda247 Store
Purchased by roshnipanwar025@gmail.com
Q111.
In each of the given questions an inference is given in bold which is then followed by three
statements. You have to nd the statement(s) from where it is inferred. Choose the option with
the best possible outcome as your choice.

Nobody gave much thought about those innocent who were at the receiving end of
demonetization at the advent.

(I) Indians are a very emotional lot and tend to quickly come to conclusions without delving deep
into issues. The public outcry against demonetization — especially by those with few solutions at
their hand to face the problem of cash crunch. The main criticism against the government is that
demonetisation has in icted more pain on the poor and innocent while opening a window for the
rich to cleanse their unaccounted income, of course, after paying a handsome share to the
exchequer.
(II) If demonetisation was a war, then those who lost their lives outside bank queues are true
martyrs. These ordinary citizens sincerely believed the government’s intentions of bringing in
transparency in the economy. Which is why they were willing to put their lives at risk to withdraw
their hard-earned money in new currency. People waiting in endless queues outside banks to
exchange old notes reportedly resulted in even deaths during the rst few weeks after
demonetisation. Honouring the sacri ce of these true martyrs would be a tting tribute on the
rst anniversary of demonetisation.
(III) A year after Prime Minister NarendraModi announced the ban on Rs 1000 and Rs 500 old
banknotes, his government described demonetisation as a historic and multi-dimensional success.
Huge advertisements could be seen where Modi lauded 125 crore Indians for ghting a decisive
battle against black money and corruption. Unfortunately despite tall claims by BJP, the debate still
rages whether the move has bene tted the nation or done more harm than good. (1 Mark)
A. Both (I) & (III)
B. Both (II) & (III)
C. Only (I)
D. Only (II)
E. All of the above for more join
Your Answer: Not Attempted
https://t.me/currentAffairscLuB
Correct Answer: D. Only (II) <br>

Solution
The inference drawn is about the people who faced severities during the period starting with
implementation of Demonetisation till situations eased out. Statement (I) talks about the
emotional outcry of people and their criticism about government's step but nowhere presents any
https://store.adda247.com/#!/myTestAnalysis/printsolution/mappingId=25329/packageId=359/lang=ENGLISH 99/121
12/31/2017 Adda247 Store | Adda247 Store
fact or example of unattended people's su ering . Statement (III) talks about only the introduction
of the campaign and the related debates about its righteousness. Statement (II) presents soundly
the fact that ordinary innocent people who believed in government intentions su ered the
hardships and not much was done or thought about easing their su ering at the start of the
process.
Purchased by roshnipanwar025@gmail.com
Q112.
In each of the given questions an inference is given in bold which is then followed by three
statements. You have to nd the statement(s) from where it is inferred. Choose the option with
the best possible outcome as your choice.

Extremism has spread even among Hindus, some of whom believe not in satyamevajayate but in
might is right.

(I) The Parivar proclaims an ideology of “Hindutva,” aimed at ensuring the predominance of
Hinduism in Indian society, politics, and culture, which it promotes through tactics that include
violence and terror. Its agenda includes subjugating or driving out people of other faiths, who total
some 17 percent of the population. It castigates them as foreign faiths, imposed by foreign
conquerors.
(II) All wanton violence and religious fundamentalism is wrong, be it of the right or the left, or of
any religion, and needs to be identi ed, countered and condemned. In this context, the real
question for Hindus is who, and for what reason, is today deliberately fanning this fanatical
violence among them? According to established Hindu practice, disagreements should be resolved
through debate, dialogue and discussion, yet one has only to remember that it was a fanatical
Hindu who killed one of the greatest messiahs of peace – Mahatma Gandhi.
(III)There is no denying that fringe right wing groups have created an atmosphere of intolerance to
outspoken writers and academics who question religious practices and myths, thereby putting
pressure on freedom of speech and expression. The event of killing of veteran writer
MalleshappaMadivalappaKalburgi demands the government not to go soft on Hindu
fundamentalism and to “crack down” on these fringe elements in the same way it would deal with
other “religion and ideology based extremist groups.” (1 Mark)
A. Both (I) & (II) for more join
B. Both (I) & (III)
C. Both (II) &(III) https://t.me/currentAffairscLuB
D. Only (I)
E. All of the above
Your Answer: Not Attempted

Correct Answer: B. Both (I) &amp; (III) <br>

https://store.adda247.com/#!/myTestAnalysis/printsolution/mappingId=25329/packageId=359/lang=ENGLISH 100/121
12/31/2017 Adda247 Store | Adda247 Store
Solution

The inference is drawn is that Hindu out ts can no longer deny the existence of extremism among
their ranks. Statement (II) though mentions the killing of Mahatama Gandhi by a fanatical Hindu, it
fails to state that it has been backed the community. Statement (I) & (III) support the inference
based on facts that fringe Hindus radicals have created an atmosphere of intolerance for writers,
academics speaking against them and predominance of Hinduism in Indian society, politics, and
culture, which it promotes through tactics that include violence and terror respectively.
Purchased by roshnipanwar025@gmail.com
Q113.
In each of the given questions an inference is given in bold which is then followed by three
statements. You have to nd the statement(s) from where it is inferred. Choose the option with
the best possible outcome as your choice.

It is important for the courts to examine disability as a ground for the grant of bail.

(I) The deplorable conditions in Indian prisons are well known. It is settled law now that prisoners
may be deprived of personal liberty according to procedure established by law, but that does not
include a derogation of their right to dignity. How do we begin to understand the sanctity of life,
dignity and bodily integrity for a person with disabilities? If handcu ng is an extraordinary and
excessive restraint on an ordinary prisoner, what constitutes excessive restraint beyond the writ of
law for a person with disabilities? Placing him in solitary con nement with no support violates his
right to life, bodily integrity and autonomy even when conviction only imposes restraints on
personal liberty.
(II) The Indian Constitution gives pride of place to the fundamental rights of a citizen, including the
right to life and liberty. One would assume that it implies two basic legal principles: one, that the
accused is innocent until proven guilty; and two, that bail is the norm and jail the exception. The
stories of thousands of undertrials- including those with disabilities-languishing in jails across the
country, however, run against the grain of the constitutional promise that “no person shall be
deprived of his life or personal liberty except according to procedure established by law”
(III) The Law Commission has done well to recommend a complete overhaul in the way courts
grant bail. Bail must be the rule rather than the exception, given that every person charged with a
for more join
crime is presumed innocent until proven guilty. Reform in bail jurisprudence that includes fast
https://t.me/currentAffairscLuB
disposal of bail applications, easier surety requirements and minimising pretrial detention is
overdue. Courts must deny bail only under three conditions. One, the person charged with the
crime is likely to ee. Two, the accused is likely to tamper with evidence or in uence witnesses.
Three, the person is likely to repeat the same crime if granted bail. (1 Mark)
A. Only (III)
B. Only (I)
C. Only (II)
https://store.adda247.com/#!/myTestAnalysis/printsolution/mappingId=25329/packageId=359/lang=ENGLISH 101/121
12/31/2017 Adda247 Store | Adda247 Store
D. Both (I) & (II)
E. Both (II) & (III)
Your Answer: Not Attempted

Correct Answer: B. Only (I) <br>

Solution

The inference drawn is based upon the fact that if the court nds out a circumstance where in
there is need for special physical assistance for the prisoner daily life activities i.e. with a physical
disability, he should be allowed bail on this ground. Statement (II) and (III) beat around the bush
with references to the Indian Constitution stating right to life and liberty but it doesn't speci es it
particularly for the disabled accused. Statement (I) alone provides for consideration of a person's
physical condition as a ground for bail provision.
Purchased by roshnipanwar025@gmail.com
Q114.
In each of the given questions an inference is given in bold which is then followed by three
statements. You have to nd the statement(s) from where it is inferred. Choose the option with
the best possible outcome as your choice.

Police investigators should not jump to conclusions, in uenced by public outrage

(I) That the police had to extract a false confession is downright disgraceful, but it is not an isolated
case in a country known for its primitive investigative methods. In cases of demand of a result
from the masses, putting a statement which is refutable isn't justi ed. Studies on police reforms
have highlighted the need to make the investigation process more scienti c and more rooted in
forensic analysis, but custodial torture and extracted confessions continue to be reported. When
two narratives emerge from di erent police agencies for a heinous murder, a sense of disquiet
among the public is inevitable.
(II) Regimes come and regimes go, a new party and a new leadership adds its own new pages to
the annals of India’s political history. But India’s khaki tales continue with the same old narrative –
hackneyed, violent and tragic, both for the people of the country as well as for the rank and le
for more join
cops. Not trained properly for the job that requires mental strength to withstand public pressure
they are often rounded up in botched up https://t.me/currentAffairscLuB
investigations. The strong arm of the state is severely
weak within and the politicians want it that way; bereft of choice, cops take solace in venting their
‘powerless power’ where they can.
(III) Police bumbling closure in Pradyuman murder case is no exception. It points to systemic
failures. It is often di cult to distinguish police investigation, especially carried out in the face of
public and political pressure, from the witch-hunt. Police with only rudimentary training in modern

https://store.adda247.com/#!/myTestAnalysis/printsolution/mappingId=25329/packageId=359/lang=ENGLISH 102/121
12/31/2017 Adda247 Store | Adda247 Store
investigation, using beatings and threats as their main truth-seeking tools, have a long record of
securing wrongful convictions, as well as letting the guilty walk free, sometimes wearing the halo
of martyrdom. To explain this away as the failure of individuals is inadequate. (1 Mark)
A. Both (I) & (II)
B. Both (I) & (III)
C. Both (II) & (III)
D. All of the above
E. Only (III)
Your Answer: Not Attempted

Correct Answer: B. Both (I) &amp; (III) <br>

Solution

The inference drawn centers around the fact that police not able to withstand public pressure
jump to conclusions that may be later refuted. Supporting this inferences are Statement (I) & (III)
which clearly mentions police investigations carried in wake of public pressure have resulted into
debatable conclusions showing inadequacy. Statement (II) makes a mention of the public pressure
but then deviates to badly carried out investigations. It doesn't show that a public statement was
issued on the basis of any conclusion.
Purchased by roshnipanwar025@gmail.com
Q115.
In each of the given questions an inference is given in bold which is then followed by three
statements. You have to nd the statement(s) from where it is inferred. Choose the option with
the best possible outcome as your choice.

More and more bilateral series have taken away the excitement from the game.

(I) Players often display more cricketing brilliance on the eld as they would be watched by more
viewers than in a bilateral ODI series. The most interesting feature of these tournaments is that
they seldom get boring. On the other hand, a seven-match bilateral ODI series can become
monotonous if one team holds inordinate sway in the opening matches.
for more join
(II) With repeated bilateral clashes between the same opponents, the fans hardly get to speculate
as much as they liked to do when morehttps://t.me/currentAffairscLuB
number of teams brought more number of players and
hence more erce competition from the players. The near same combinations of the two teams
makes the scope of watching the game for some innovative batting or bowling battles, narrowed
down to negligible. Moreover, Due to the bilateral nature of busy cricket schedules and the
mushrooming of Twenty20 cricket leagues, the space for triangular cricket tournaments has been
squeezed.
(III) The ODI tournaments were more keenly followed than bilateral ODI face-o s in general. This is
https://store.adda247.com/#!/myTestAnalysis/printsolution/mappingId=25329/packageId=359/lang=ENGLISH 103/121
12/31/2017 Adda247 Store | Adda247 Store
for many reasons. In multilateral tournaments, one gets to see many teams participate and make
a bid for the same trophy. The permutations and combinations of match results creates a unique
interest, allowing fans to constantly examine ways in which a particular team can reach the knock-
out stage or clinch the title. (1 Mark)
A. Both (I) & (II)
B. Only II
C. Both (II) & (III)
D. Both (III) & (I)
E. All of the above
Your Answer: Not Attempted

Correct Answer: E. All of the above <br>

Solution

The inference drawn on the basis of increasing bilateral faceo s in ODI cricket to be making game
watching boring is supported by all the Statements (I) (II) & (III) as all statements mention the facts
: Number of bilateral series have increased with decrease in more than 2 team series tournaments
hence resulting in increase of monotonous nature in these clashes.
Purchased by roshnipanwar025@gmail.com
Q116.
The Reserve Bank of India opened a branch in which city to make transactions of the Uttarakhand
State government more convenient?
A. Mussoorie
B. Rishikesh
C. Nainital
D. Haridwar
E. Dehradun
Your Answer: Not Attempted

Correct Answer: E. Dehradun <br>


for more join
Solution
https://t.me/currentAffairscLuB
The Reserve Bank of India opened a branch in the Dehradun city to make transactions of the
Uttarakhand State government more convenient. Uttarakhand Chief Minister Trivendra Singh
Rawat inaugurated the RBI's Branch. Earlier, the state government had to do nancial transactions
through the Kanpur branch of the RBI.
Purchased by roshnipanwar025@gmail.com
Q117.

https://store.adda247.com/#!/myTestAnalysis/printsolution/mappingId=25329/packageId=359/lang=ENGLISH 104/121
12/31/2017 Adda247 Store | Adda247 Store
The Centre has recently set up a high-level committee for proper management of water resources
in the North-Eastern region. The Committee will be headed by __________________.
A. NHAI Chairman Deepak Kumar
B. Chairman of National Water supply and Drainage Board K.Alahudeen Ansar
C. NITI Aayog Vice Chairman Rajiv Kumar
D. Chairman of Central Water Commission Narendra Kumar
E. None of these
Your Answer: Not Attempted

Correct Answer: C. NITI Aayog Vice Chairman Rajiv Kumar <br>

Solution
Centre has set up a high-level committee for proper management of water resources in the North-
Eastern region. The Committee will be headed by NITI Aayog Vice Chairman Rajiv Kumar.
Purchased by roshnipanwar025@gmail.com
Q118.
Name the insurance company that was conferred with the 21st Asia Insurance Industry Awards in
the "General Insurance Company of the Year" category.
A. Bajaj Allianz General Insurance
B. Bharti AXA Life Insurance
C. IDBI Federal Health Insurance
D. Apollo Munich Health Insurance Company
E. Sriram Health Insurance
Your Answer: Not Attempted

Correct Answer: D. Apollo Munich Health Insurance Company <br>

Solution
Apollo Munich Health Insurance Company, a joint venture between integrated healthcare
provider, Apollo Hospitals Group, and the Munich Re Group was conferred with the 21st Asia
Insurance Industry Awards in the "General Insurance Company of the Year" category.
Purchased by roshnipanwar025@gmail.com
Q119.
for more join
The minimum paid-up equity capital forhttps://t.me/currentAffairscLuB
small nance banks shall be----------
A. Rs. 100 crore
B. Rs. 200 crore
C. Rs. 300 crore
D. Rs. 400 crore
E. Rs. 500 crore
Your Answer: Not Attempted
https://store.adda247.com/#!/myTestAnalysis/printsolution/mappingId=25329/packageId=359/lang=ENGLISH 105/121
12/31/2017 Adda247 Store | Adda247 Store

Correct Answer: A. Rs. 100 crore <br>

Solution
The minimum paid-up equity capital for small nance banks shall be Rs. 100 crore.
Purchased by roshnipanwar025@gmail.com
Q120.
NSDP links are intended to provide quick access to a single comprehensive source of economic
and nancial data consistent with the data categories and components described in the
subscriber's metadata. NSDP stands for-
A. National Summary District Page
B. National Summary Development Page
C. National Summary Department Page
D. National Summary Division Page
E. National Summary Data Page
Your Answer: Not Attempted

Correct Answer: E. National Summary Data Page <br>

Solution
The National Summary Data Page (NSDP) links are intended to provide quick access to a single
comprehensive source of economic and nancial data consistent with the data categories and
components described in the subscriber's metadata. Additionally, many of the NSDPs include
further links to additional data or information on other national Internet data sites.
Purchased by roshnipanwar025@gmail.com
Q121.
Which of the following is the parent organization of UPI?
A. RBI
B. GOI
C. NPCI
D. PMMY
E. SEBI
Your Answer: Not Attempted
for more join
https://t.me/currentAffairscLuB
Correct Answer: C. NPCI <br>

Solution
National Payments Corporation of India (NPCI) is the parent organisation of UPI (Uni ed Payment
Interface).
Purchased by roshnipanwar025@gmail.com

https://store.adda247.com/#!/myTestAnalysis/printsolution/mappingId=25329/packageId=359/lang=ENGLISH 106/121
12/31/2017 Adda247 Store | Adda247 Store
Q122.
The International Consortium of Investigative Journalists (ICIJ) leaked a massive trove of secret
nancial data named "Paradise Papers". Its headquarter is in ___________________.
A. New York
B. Singapore
C. Washington DC
D. Brussels
E. Geneva
Your Answer: Not Attempted

Correct Answer: C. Washington DC <br>

Solution
ICIJ is a global network of more than 200 investigative journalists in 70 countries who collaborate
on in-depth investigative stories.It was founded in 1997 by an American journalist Chuck Lewis.It is
Headquartered in Washington, DC, USA.
Purchased by roshnipanwar025@gmail.com
Q123.
BCSBI was registered as a society under the _____________ in February 2006.
A. Companies Act, 1956
B. Societies Registration Act, 1860
C. Banking Regulation Act, 1949
D. Reserve Bank of India Act, 1934
E. None of these
Your Answer: Not Attempted

Correct Answer: B. Societies Registration Act, 1860 <br>

Solution
The Banking Codes and Standards Board of India was registered as a society under the Societies
Registration Act, 1860 in February 2006. It functions as an independent and autonomous body.
Membership of BCSBI is voluntary and open to scheduled banks.
for more join
Purchased by roshnipanwar025@gmail.com
Q124. https://t.me/currentAffairscLuB
Name the Bank, which has launched India's rst payments bank?
A. Fino Payments Bank Ltd
B. Paytm Payments Bank
C. Airtel Payments Bank Ltd
D. India Post Payments Bank
E. Reliance Industries
https://store.adda247.com/#!/myTestAnalysis/printsolution/mappingId=25329/packageId=359/lang=ENGLISH 107/121
12/31/2017 Adda247 Store | Adda247 Store
Your Answer: Not Attempted

Correct Answer: C. Airtel Payments Bank Ltd <br>

Solution
Purchased by roshnipanwar025@gmail.com
Q125.
The launch of India's rst voice-based international remittance service that would enable
nonresident Indian to send money to any bank in India has been announced by
___________________.
A. Bank of Baroda
B. CITI Bank
C. HDFC
D. SBI
E. ICICI
Your Answer: Not Attempted

Correct Answer: E. ICICI <br>

Solution
It is rst-of-its-kind cross-border remittance service by a bank in the country.With this new feature
in the ICICI Bank's Money2India app, an NRI customer can instantly initiate a remittance to his/her
existing payees in India with just a simple voice command to Apple's virtual voice assistant, Siri, on
their iPhone/iPad.
Purchased by roshnipanwar025@gmail.com
Q126.
Kundan Shah was passed away recently. He was?
A. scientist and politician
B. philosopher and geographer
C. dancer and cameraman
D. singer and producer
E. director and writer
Your Answer: Not Attempted
for more join
https://t.me/currentAffairscLuB
Correct Answer: E. director and writer <br>

Solution
Purchased by roshnipanwar025@gmail.com
Q127.

https://store.adda247.com/#!/myTestAnalysis/printsolution/mappingId=25329/packageId=359/lang=ENGLISH 108/121
12/31/2017 Adda247 Store | Adda247 Store
An illustrative book containing inspiring words of Dr A.P.J. Abdul Kalam was released online by
Indian women cricket team captain Mithali Raj. The book has been titled as ___________________.
A. Dreamnation: Uniting a Country with Handwritten Dreams
B. Kalam's Inspirational Words
C. The Magical Words of Kalam
D. The Words of APJ
E. None of these
Your Answer: Not Attempted

Correct Answer: A. Dreamnation: Uniting a Country with Handwritten Dreams <br>

Solution
An illustrative book containing inspiring words of Dr A.P.J. Abdul Kalam was released online by
Indian women cricket team captain Mithali Raj on the 86th birth anniversary of the former
President. Titled "Dreamnation: Uniting a Country with Handwritten Dreams", the book includes a
collection of 200-plus handcrafted and handwritten postcards from across India. The book is Co-
authored by Saji Mathew and Jubie John and published by Bloomsbury.
Purchased by roshnipanwar025@gmail.com
Q128.
Indian Hockey team had beaten ___________ in the nale to win the 2017 Asia Cup Hockey
tournament held in Dhaka, Bangladesh.
A. Malaysia
B. Bangladesh
C. China
D. Pakistan
E. Sri Lanka
Your Answer: Not Attempted

Correct Answer: A. Malaysia <br>

Solution
Purchased by roshnipanwar025@gmail.com
Q129.
for more join
https://t.me/currentAffairscLuB
An Asset Reconstruction Company (ARC) in India is associated with-
A. UCPDC
B. DICGC
C. NPA
D. Home Loan
E. None of these
Your Answer: Not Attempted
https://store.adda247.com/#!/myTestAnalysis/printsolution/mappingId=25329/packageId=359/lang=ENGLISH 109/121
12/31/2017 Adda247 Store | Adda247 Store

Correct Answer: C. NPA <br>

Solution
Asset Reconstruction Company (ARC) also known as Securitization company in India helps to
unlock the value of non-performing assets (NPA) in banks through Securitization and
Reconstruction of Financial Assets and Enforcement of Security Interest (SARFAESI) Act 2002.ARC is
regulated by RBI as a Non-Banking Financial Company (NBFC).
Purchased by roshnipanwar025@gmail.com
Q130.
Who has won the Nobel Literature Prize 2017?
A. Rainer Weiss
B. Barry Barish
C. Richard Thaler
D. Bob Dylan
E. Kazuo Ishiguro
Your Answer: Not Attempted

Correct Answer: E. Kazuo Ishiguro <br>

Solution
Kazuo Ishiguro wins the Nobel prize in literature 2017. The British author Kazuo Ishiguro said he
was both honoured and “taken completely by surprise” after he was named this year's winner of
the 2017 Nobel prize in literature
Purchased by roshnipanwar025@gmail.com
Q131.
India and _____________ are going to conduct the 7th edition of joint Military Training Combat
Exercise 'Sampriti-7'.
A. Sri Lanka
B. Nepal
C. Bangladesh
D. Myanmar
E. Singapore
for more join
Your Answer: Not Attempted https://t.me/currentAffairscLuB
Correct Answer: C. Bangladesh <br>

Solution
Army personnel of India and Bangladesh are going to take part in a 13-day joint training exercise
'Sampriti-7' at Joint Training Node at Umroi in Meghalaya and India's Jungle Warfare School at
https://store.adda247.com/#!/myTestAnalysis/printsolution/mappingId=25329/packageId=359/lang=ENGLISH 110/121
12/31/2017 Adda247 Store | Adda247 Store
Vairengte in Mizoram. This is the 7th edition of the joint training exercise.
The main objective of the exercise is to conduct counter-insurgency and counter-terrorism
operations in peace-keeping activities in the United Nations (UN).
Purchased by roshnipanwar025@gmail.com
Q132.
Which of the following is not available to depositors of NBFCs?
A. Deposit Insurance facility
B. Acquisition of shares
C. Loans and Advances
D. All of the Above
E. None of these
Your Answer: Not Attempted

Correct Answer: A. Deposit Insurance facility <br>

Solution
Deposit insurance facility of Deposit Insurance and Credit Guarantee Corporation is not available
to depositors of NBFCs, unlike in case of banks.
Purchased by roshnipanwar025@gmail.com
Q133.
The International Day of Non-Violence is celebrated across the world on ____________.
A. 01 October
B. 02 October
C. 03 October
D. 04 October
E. 05 October
Your Answer: Not Attempted

Correct Answer: B. 02 October <br>

Solution
The International Day of Non-Violence is observed on October 2, the birthday of Mahatma Gandhi.
This day is referred to in India as Gandhi Jayanti.
for more join
https://t.me/currentAffairscLuB
Purchased by roshnipanwar025@gmail.com
Q134.
The Ministry of Shipping has issued a noti cation renaming Kandla Port Trust as Deendayal Port
Trust with e ect from 25th of September, 2017. Kandla Port located in-
A. Kerala
B. Andhra Pradesh
C. Odisha

https://store.adda247.com/#!/myTestAnalysis/printsolution/mappingId=25329/packageId=359/lang=ENGLISH 111/121
12/31/2017 Adda247 Store | Adda247 Store
D. Gujarat
E. Maharashtra
Your Answer: Not Attempted

Correct Answer: D. Gujarat <br>

Solution
The Kandla Port, one of the top 12 major ports in the country, has been rechristened as Deendayal
Port from today in the name of Hindutva icon Pandit Deendayal Upadhyay. Kandla Port Trust is a
seaport in Kutch District of Gujarat state in western India, near the city of Gandhidham. Located
on the Gulf of Kutch, it is one of major ports on west coast.
Purchased by roshnipanwar025@gmail.com
Q135.
As per the recommendations of the 7th Pay Commission, the government has raised the
maximum amount that a central government employee can borrow from the government to Rs
________________ for new construction/purchase of new house/ at.
A. 20 lakh
B. 25 lakh
C. 15 lakh
D. 30 lakh
E. 10 lakh
Your Answer: Not Attempted

Correct Answer: B. 25 lakh <br>

Solution
The total amount of advance that a central government employee can borrow from government
has been revised upwards. The employee can borrow upto 34 months of the basic pay subject to a
maximum of Rs 25 lakh, or cost of the house/ at, or the amount according to repaying capacity,
whichever is the least for new construction/purchase of new house/ at. Earlier this limit was only
Rs 7.50 lakh.
Purchased by roshnipanwar025@gmail.com
Q136.
for more join
https://t.me/currentAffairscLuB
RIDF is maintained by the National Bank for Agriculture and Rural Development (NABARD). RIDF
was set up by the Government in 1995-96 for nancing ongoing ____________ projects.
A. Road projects
B. Infrastructure projects
C. Electricity projects
D. Food projects
E. Health Projects
https://store.adda247.com/#!/myTestAnalysis/printsolution/mappingId=25329/packageId=359/lang=ENGLISH 112/121
12/31/2017 Adda247 Store | Adda247 Store
Your Answer: Not Attempted

Correct Answer: B. Infrastructure projects <br>

Solution
The RIDF was set up by the Government in 1995-96 for nancing ongoing rural Infrastructure
projects. The Fund is maintained by the National Bank for Agriculture and Rural Development
(NABARD).
Purchased by roshnipanwar025@gmail.com
Q137.
The International Campaign to Abolish Nuclear Weapons (ICAN) is a coalition of non-government
organizations in one hundred countries advocating for a strong and e ective nuclear weapon ban
treaty. Where is the headquarter of ICAN?
A. Vienna, Austria
B. Nairobi, Kenya
C. Geneva, Switzerland
D. New York, USA
E. Washington DC, USA
Your Answer: Not Attempted

Correct Answer: C. Geneva, Switzerland <br>

Solution
Purchased by roshnipanwar025@gmail.com
Q138.
As per the amendment, the pecuniary jurisdiction of the Banking Ombudsman to pass an award
has been doubled to _____ rupees.
A. 20 lakh rupees
B. 10 lakh rupees
C. 25 lakh rupees
D. 50 lakh rupees
E. 5 lakh rupees
Your Answer: Not Attempted
for more join
https://t.me/currentAffairscLuB
Correct Answer: A. 20 lakh rupees <br>

Solution
The central bank has extended the scope of Banking Ombudsman Scheme. As per RBI, under the
amended scheme, a customer would also be able to lodge a complaint against the bank for its
non-adherence to RBI instructions with regard to mobile banking/ electronic banking services in
https://store.adda247.com/#!/myTestAnalysis/printsolution/mappingId=25329/packageId=359/lang=ENGLISH 113/121
12/31/2017 Adda247 Store | Adda247 Store
India. As per the amendment, the pecuniary jurisdiction of the Banking Ombudsman to pass an
award has been doubled to 20 lakh rupees. The Reserve Bank extended the scope of Banking
Ombudsman Scheme under which banks could be penalized for mis-selling third-party products
like insurance and mutual funds via mobile or electronic banking.
Purchased by roshnipanwar025@gmail.com
Q139.
The World's highest bridge has been inaugurated by Indian Railways in Jammu & Kashmir's Reasi
district. The bridge has been constructed on which river?
A. Sutlej
B. Ravi
C. Shingo
D. Tawi
E. Chenab
Your Answer: Not Attempted

Correct Answer: E. Chenab <br>

Solution
The Indian Railways has launched the main arch of the world's highest bridge(359 mts) on the
River Chenab in Jammu and Kashmir's Reasi district which will provide direct connectivity to the
Kashmir Valley. The iconic bridge is being constructed on River Chenab on the Katra-Banihal
section as part of the agship Udhampur-Srinagar-Baramulla Rail Link (USBRL) Project.
Purchased by roshnipanwar025@gmail.com
Q140.
___________ a/an electronic trading platform, operated by the Reserve Bank of India, used to
facilitate the exchange of government securities and other money market instruments.
A. Delivery versus Payment (DvP)
B. Negotiated Dealing System (NDS)
C. Risk Mitigation
D. Competition Commission of India (CCI)
E. None of these
Your Answer: Not Attempted
for more join
Correct Answer: B. Negotiated Dealing https://t.me/currentAffairscLuB
System (NDS) <br>

Solution
Till 2002, the Government securities market was mainly a telephone market. Buyers and sellers
traded over telephone and submitted physical Subsidiary General Ledger (SGL) transfer forms for
transfer of the Government securities and cheques for settlement of the funds to the Reserve
Bank of India. These manual operations were ine cient and often resulted in delays. In order to
https://store.adda247.com/#!/myTestAnalysis/printsolution/mappingId=25329/packageId=359/lang=ENGLISH 114/121
12/31/2017 Adda247 Store | Adda247 Store
improve e ciency in the market, the Reserve Bank of India took steps to automate the process of
trading and settlement of Government securities transactions and the Negotiated Dealing System
(NDS) was introduced in February 2002.
Purchased by roshnipanwar025@gmail.com
Q141.
NAV is the value per share of a mutual fund or an exchange-traded fund (ETF) on a speci c date or
time. NAV stands for-
A. Net Asset Voice
B. Nominal Asset Value
C. Non Asset Value
D. Net Agency Value
E. Net Asset Value
Your Answer: Not Attempted

Correct Answer: E. Net Asset Value <br>

Solution
Net asset value (NAV) is value per share of a mutual fund or an exchange-traded fund (ETF) on a
speci c date or time.
Purchased by roshnipanwar025@gmail.com
Q142.
Currency is also printed by Reserve Bank of India. It has two printing presses which are owned by
Bhartiya Reserve Bank Note Mudran Private Limited. These printing presses located in Mysore and
another is-
A. Nasik
B. Dewas
C. Salboni
D. Noida
E. Hyderabad
Your Answer: Not Attempted

Correct Answer: C. Salboni <br>


for more join
Solution https://t.me/currentAffairscLuB
Purchased by roshnipanwar025@gmail.com
Q143.
The Wushu World Championship 2017 was held recently in _____________.
A. Kazan, Russia
B. New Delhi ,India
C. Seoul , China

https://store.adda247.com/#!/myTestAnalysis/printsolution/mappingId=25329/packageId=359/lang=ENGLISH 115/121
12/31/2017 Adda247 Store | Adda247 Store
D. Paris,France
E. None of these
Your Answer: Not Attempted

Correct Answer: A. Kazan, Russia <br>

Solution
Purchased by roshnipanwar025@gmail.com
Q144.
In which state is Jog falls located.......?
A. Maharastra
B. Uttar Pradesh
C. Tamil nadu
D. Kerala
E. Karnataka
Your Answer: Not Attempted

Correct Answer: E. Karnataka <br>

Solution
Located on the border of Shimoga and Uttara Kannada districts of Karnataka, Jog waterfalls is the
highest waterfall In the state and is regularly mistaken as the highest waterfalls of India. Also
known as Gerosappa Falls or the Joga Falls, these falls are one of the most important attractions in
all of South India.
Purchased by roshnipanwar025@gmail.com
Q145.
Name the Prime Minister of Italy who is on a two day visit to India.
A. Sergio Mattarella
B. Paolo Gentiloni
C. Matteo Renzi
D. Enrico Letta
E. Silvio Berlusconi
Your Answer: Not Attempted
for more join
https://t.me/currentAffairscLuB
Correct Answer: B. Paolo Gentiloni <br>

Solution
Prime Minister of Italy Paolo Gentiloni arrived in New Delhi on a two day visit to India. The visit is
aimed at strengthening the bilateral political and economic relations between the two countries.
The last visit of the Italian Prime Minister to India was in February 2007.
https://store.adda247.com/#!/myTestAnalysis/printsolution/mappingId=25329/packageId=359/lang=ENGLISH 116/121
12/31/2017 Adda247 Store | Adda247 Store
Purchased by roshnipanwar025@gmail.com
Q146.
The Deen Dayal SPARSH Yojana has been launched by Minister of Communications to promote
Philately. In SPARSH, A stands for ______________
A. Academic
B. Adolescent
C. Awareness
D. Application
E. Aptitude
Your Answer: Not Attempted

Correct Answer: E. Aptitude <br>

Solution
Under the scheme of SPARSH (Scholarship for Promotion of Aptitude & Research in Stamps as a
Hobby) children with good academic record and also pursuing Philately as a hobby will be selected
through a competitive selection process in all postal circles.
Purchased by roshnipanwar025@gmail.com
Q147.
Stepping up its ght against tax evasion, the Centre has made it mandatory to quote the Aadhaar
number for opening bank accounts and for transactions exceeding-
A. Rs.10,000
B. Rs.80,000
C. Rs.50,000
D. Rs.1,00,000
E. Rs.1,50,000
Your Answer: Not Attempted

Correct Answer: C. Rs.50,000 <br>

Solution
Stepping up its ght against tax evasion, the Centre has made it mandatory to quote the Aadhaar
for more join
number for opening bank accounts and for transactions exceeding Rs.50,000. With the
https://t.me/currentAffairscLuB
amendments to the Prevention of Money Laundering (Maintenance of Records) Rules, 2005
noti ed earlier this month, banks will have to demand both Aadhaar and the Permanent Account
Number (PAN) for veri cation of identity.
Purchased by roshnipanwar025@gmail.com
Q148.
Sushil Kumar Solanki is famous in which of the following eld?
A. Politics

https://store.adda247.com/#!/myTestAnalysis/printsolution/mappingId=25329/packageId=359/lang=ENGLISH 117/121
12/31/2017 Adda247 Store | Adda247 Store
B. Art and Living
C. Film and Television
D. Author
E. None of the above
Your Answer: Not Attempted

Correct Answer: E. None of the above <br>

Solution
Sushil Kumar Solanki is an Indian freestyle wrestler. Competing in the 66 kg weight division he won
the 2010 world title, a silver medal at the 2012 London Olympics and a bronze medal at the 2008
Beijing Olympics, which made him the only Indian to win two individual Olympic medals.
Purchased by roshnipanwar025@gmail.com
Q149.
Which of the following country will host the next United Nations (UN) global wildlife conference in
2020?
A. Malaysia
B. Indonesia
C. Russia
D. India
E. France
Your Answer: Not Attempted

Correct Answer: D. India <br>

Solution
India will host the next United Nations (UN) global wildlife conference in 2020. It was o cially
announced at the closing CMSCOP12 plenary, in Manila, Philippines. The six-day 12th Meeting of
the Conference of the Parties to the Convention on Migratory Species (CMS COP12) is the only
international treaty devoted exclusively to migratory animal species. The theme of the Conference
was ‘Their Future is Our Future - Sustainable Development for Wildlife and People’.
Purchased by roshnipanwar025@gmail.com
Q150.
for more join
https://t.me/currentAffairscLuB
Orang National Park is also known as Rajiv Gandhi National park, located in-?
A. Assam
B. Tripura
C. Meghalaya
D. Arunachal Pradesh
E. Sikkim
Your Answer: Not Attempted
https://store.adda247.com/#!/myTestAnalysis/printsolution/mappingId=25329/packageId=359/lang=ENGLISH 118/121
12/31/2017 Adda247 Store | Adda247 Store

Correct Answer: A. Assam <br>

Solution
The Orang National Park is a beautiful place, located in the Darrang and Sonitpur districts of the
state of Assam.
Purchased by roshnipanwar025@gmail.com
Q151.
Name the temple in Tamil Nadu that has won UNESCO Asia Paci c Award of Merit 2017 for the
cultural heritage conservation program.
A. Bhimbetka Rock Shelters
B. Airavatesvara Temple
C. Brihadeeswar Temple
D. Ramappa Temple
E. Sri Ranganathaswamy Temple
Your Answer: Not Attempted

Correct Answer: E. Sri Ranganathaswamy Temple <br>

Solution
The Sri Ranganathaswamy Temple of Srirangam became the rst temple in Tamil Nadu to win
UNESCO Asia Paci c Award of Merit 2017 for the cultural heritage conservation program.The two
key parameters that made the temple most eligible to win the award are its traditional method of
renovating temple premises as well as re-establishment of rainwater harvesting and historic
drainage system in preventing ooding.
Purchased by roshnipanwar025@gmail.com
Q152.
Who is current Human Resource Development Minister in Narendra Modi government?
A. Dr Harsh Vardhan
B. Kalraj Mishra
C. Jual Oram
D. Prakash Javadekar
E. Thawar Chand Gehlot
for more join
Your Answer: Not Attempted https://t.me/currentAffairscLuB
Correct Answer: D. Prakash Javadekar <br>

Solution
Prakash Javadekar is an Indian Bharatiya Janata Party (BJP) politician and is the incumbent Union
Minister of Ministry of Human Resource Development (MHRD), Government of India. He was
https://store.adda247.com/#!/myTestAnalysis/printsolution/mappingId=25329/packageId=359/lang=ENGLISH 119/121
12/31/2017 Adda247 Store | Adda247 Store
elected to the Rajya Sabha as a Member of Parliament from Maharashtra in 2008 and re-elected
from Madhya Pradesh in 2014.
Purchased by roshnipanwar025@gmail.com
Q153.
Kudremukh National Park is situated in which of the following Indian state?
A. Karnataka
B. Kerala
C. West Bengal
D. Uttar Pradesh
E. Meghalaya
Your Answer: Not Attempted

Correct Answer: A. Karnataka <br>

Solution
Kuduremukha National Park is a mountain range and name of a peak located in Chikkamagaluru
district, in Karnataka.
Purchased by roshnipanwar025@gmail.com
Q154.
Who is the governor of Sikkim?
A. Keshari Nath Tripathi
B. Acharya Dev Vrat
C. Shriniwas Dadasaheb Patil
D. V.P. Singh Badnore
E. Tathagata Roy
Your Answer: Not Attempted

Correct Answer: C. Shriniwas Dadasaheb Patil <br>

Solution
Shriniwas Patil is the present Governor of Sikkim, India. He was a member of the 13th and 14th
Lok Sabha of India. He represented the Karad constituency of Maharashtra and is a member of
for more join
the Nationalist Congress Party (NCP). He is a former Indian Administrative Service o cer.
https://t.me/currentAffairscLuB
Purchased by roshnipanwar025@gmail.com
Q155.
Pradhan Mantri Suraksha Bima Yojana (PMSBY) is one of three social security schemes that the
government had announced in the 2015 Budget. What is PMSBY?
A. An accident insurance scheme
B. Life insurance scheme
C. Health insurance scheme

https://store.adda247.com/#!/myTestAnalysis/printsolution/mappingId=25329/packageId=359/lang=ENGLISH 120/121
12/31/2017 Adda247 Store | Adda247 Store
D. Future money insurance scheme
E. Motor vehicles insurance scheme

Your Answer: Not Attempted

Correct Answer: A. An accident insurance scheme <br>

Solution
Pradhan Mantri Suraksha Bima Yojana (PMSBY) is one of three social security schemes that the
government had announced in the 2015 Budget. The other two being Pradhan Mantri Jeevan Jyoti
Bima Yojana (PMJJBY) and the Atal Pension Yojana (APY). An accident insurance scheme, PMSBY
o ers a one-year accidental death and disability cover, which can be renewed annually.

for more join


https://t.me/currentAffairscLuB

https://store.adda247.com/#!/myTestAnalysis/printsolution/mappingId=25329/packageId=359/lang=ENGLISH 121/121

You might also like